Sie sind auf Seite 1von 50

!"#$!

%&%'$"(%)#*%&%+'%+,-+%
%
THE NATURE OF SALE
1. GAITE v FONACIER Gross inadequacy of price does NOT affect the validity
of sale, unless it indicates either (1) a vice of consent or
FACTS: Fonacier was the holder of 11 iron lode mineral (2) that the parties intended a donation or some other
claims. By a Deed of Assignment, he appointed Gaite as contract. No evidence suggests such circumstances. The
his attorney-in-fact for the purpose of operating the price need not be the exact value of the property. In
same. Gaite then executed a general assignment fact, all the parties to the sale believed that they
conveying the right to develop and exploit the mining received the commutative value of what they paid for.
claim to Larap Iron Mines, owned by him, and then
started to develop the same. Fonacier then decided to
revoke the authority granted to Gaite; the latter 3. CELESTINO & CO. v COLLECTOR
acceded and transferred the claims back to Fonacier but
for considerationroyalties and a sum of P75,000, FACTS: Celestino & Co. (Oriental Sash Factory) was
P10,000 of which was already paid. A balance of P65,000 paying 7% taxes based on gross receipts for the
remained for which Fonacier issued 2 sureties, good for manufacture and sale of sash products. It now seeks to
a year. There was a stipulation that the P65,000 balance pay only the 3% tax imposable upon contracts for piece
will be paid from the 1st shipment of ores and its local of workas opposed to the 7% tax on salesclaiming
sale. Eventually, the sureties expired and Fonacier that they do not manufacture ready-made doors for the
defaulted in settling his debt. He now alleges that the public but only upon special order of the customers.
payment of the balance was subject to a suspensive
conditionbeing the 1st shipment and sale of iron ores. ISSUE: W/N Celestino & Co. is a contractor (piece of
st
work)
ISSUE: W/N the 1 shipment and sale of iron ores are
considered suspensive condition HELD: NO. The fact that the sash products are made
only upon the order of the customers does NOT change
HELD: NO. It was only a SUSPENSIVE TERM. What took the nature of the establishment. Timing is not the
place between Gaite and Fonacier, regarding the controlling factor but the nature of the work done. They
transfer of the mining rights, was a sale. A contract of habitually make sash products and can easily duplicate
sale is normally ONEROUS and COMMUTATIVE. Each and mass-produce the same. The bulk of their sales
party anticipates performance form the very start. Since come from standard ready-made productsspecial
a sale is essentially onerous, any doubts must be settled orders are the exception and come only occasionally. If
in favor of the greatest reciprocity of rightsin this the goods are manufactured specifically upon special
case, that a period, and not a condition, was order of the customer and requires extraordinary
contemplated. Had it been a suspensive condition, service, then that would be the time when it can be
Fonacier would have been able to postpone payment classified as piece of work. But such is not the case
indefinitely. here. Oriental Sash is clearly a manufacturer and mass-
producer of doors.

2. BUENAVENTURA v CA
4. COMMISSIONER OF INTERNAL REVENUE v
FACTS: Joaquin spouses sold 6 subdivision lots to some ENGINEERING EQUIPMENT & SUPPLY CO.
of their 9 children evidenced by corresponding Deeds of
Sale. The other children, interested in protecting their FACTS: Engineering Equipment & Supply (EES) was
inheritance, sought to have the deeds of sale declared engaged in the business of designing and installing
null and void for prejudicing their legitimes, lack of central air-conditioning systems. It was assessed by the
consideration, and gross inadequacy of price. CIR for 30% advanced sales tax, among other penalties
pursuant to an anonymous complaint filed before the
ISSUE: W/N the contract of sale is valid BIR. EES vehemently objected and argued that they are
contractors and not manufacturers, and thus, should
HELD: YES. At the onset, their rights to the legitimes are only be liable for the 3% tax on sales of services or
merely inchoate and vest only upon the death of their pieces of work.
parents; thus they have no legal interest thereof.
Payment of the price has nothing to do with the ISSUE: W/N EES is a contractor (piece of work)
perfection of the contract of sale; it was perfected by
mere consent. Failure to pay consideration cannot be HELD: YES. EES was NOT a manufacturer of air-
equated with lack of consideration, which prevents the conditioning units. While it imported such items, they
existence of a valid contract. The former only results in were NOT for sale to the general public and were used
the right to demand payment or rescission. There was as mere components for the design of the centralized
already a meeting of the minds as to the price which air-conditioning system, wherein its designs and
was reflected in the Deed of Saleand that was specifications are different for every client. Various
sufficient. In fact, evidence suggests that the purchase technical factors must be considered and it can be
process have indeed been paid. The sales are thus valid. argued that no 2 plants are the same; all are engineered

ANTONIO | HIPOLITO | IMPERIAL | ZARAGOSA 1


!"#$!%&%'$"(%)#*%&%+'%+,-+%
%
separately and distinctly. Each project requires careful Three years later, Arco discovered that the prices
planning and meticulous layout. Such central air- quoted to them by GPS with regard to their first 2 orders
conditioning systems and their designs would not have mentioned were not the net prices, but rather the list
existed were it not for the special order of the party price, and that it had obtained a discount from Starr
desiring to acquire it. Thus, EES is not liable for the Piano. Moreover, Arco alleged that the equipment were
sales tax of 30%. overpriced. Thus, being its agent, GPS had to reimburse
the excess amount it received from Arco.

5. QUIROGA v PARSONS ISSUE: W/N there was a contract of agency, not of sale

FACTS: Quiroga and Parsons Hardware entered into a HELD: NO. The letters containing Arco's acceptance of
contract where the former granted the latter the the prices for the equipment are clear in their terms
exclusive right to sell Quiroga Beds in the Visayas. It and admit no other interpretation that the prices are
provided for a discount of 25% as commission for the fixed and determinate. While the letters state that GPS
sales, among other conditions. Quiroga alleged that was to receive a 10% commission, this does not
Parsons breached its contractual obligations by selling necessarily mean that it is an agent of Arco, as this
the beds at a higher price, not having an open provision is only an additional price which it bound itself
establishment in Iloilo, not maintaining a public to pay, and which stipulation is not incompatible with
exhibition, and for not ordering beds by the dozen. Only the contract of sale.
the last imputation was provided for by the contract, It is GPS that is the exclusive agent of Starr Piano in
the others were never stipulated. Quiroga argued that the Philippines, not the agent of Arco. it is out of the
since there was a contract of agency between them, ordinary for one to be the agent of both the seller and
such obligations were necessarily implied. the buyer. The facts and circumstances show that Arco
entered into a contract of sale with GPS, the exclusive
ISSUE: W/N the contract between them was one of agent of Starr Piano. As such, it is not duty bound to
agency, not sale reveal the private arrangement it had with Starr Piano
relative to the 25% discount.
HELD: NO. The agreement between Quiroga and Parsons Thus, GPS is not bound to reimburse Arco for any
was that of a simple purchase and salenot an agency. difference between the cost price and the sales price,
Quiroga supplied beds, while Parsons had the obligation which represents the profit realized by GPS out of the
to pay their purchase price. These are characteristics of transaction.
a purchase and sale. In a contract of agency (or order to
sell), the agent does not pay its price yet, and sells the
products, remitting to the principal its proceeds. Unsold 7. LO v KJS ECO-FORMWORK SYSTEM PHIL., INC.
products must also be returned to the principal. The
provisions on commission and the use of the word FACTS: KJS Inc was engaged in the sale of steel
agency in the contract as well as the testimonies in scaffolding. Sonny Lo, a contractor, purchased
court do not affect its nature. Contracts are what the scaffolding equipment worth P540,000. He made a
law defines it to be, not what the parties call it. deposit of P150,000, the balance payable within 10
months. Due to financial difficulties, Lo defaulted after
paying only 2 installments. A debt of some P335,000
6. PUYAT v ARCO AMUSEMENT CO. remained. Thus, Lo assigned in favor of KJS all his
receivables from Jomero Realty Corp. which refused to
FACTS: Arco Amusement was engaged in the business of pay and raised the defense of compensationclaiming
operating cinematopgraphs. Gonzalo Puyat & Sons Inc that Lo also had debts in its favor. KJS thus again sought
(GPS) was the exclusive agent in the Philippines for the to collect from Lo who them averred that his debts have
Starr Piano Company. Desiring to equip its already been extinguished by the said assignment.
cinematograph with sound reproducing devices, Arco
approached GPS, through its president, GIl Puyat, and ISSUE: W/N the assignment of credit extinguished the
an employee named Santos. After some negotiations, it debts
was agreed between the parties that GPS would order
sound reproducing equipment from Starr Piano Company HELD: NO. The assignment of credit made by Lo in favor
and that Arco would pay GPS, in addition to the price of of KJS was in the nature of dacion en pago, which is
the equipment, a 10% commission, plus all expenses governed by the law on sales. It is as if KJS bought the
such as freight, insurance, etc. When GPS inquired Starr credit from Lo, the payment of which is to be charged
Piano the price (without discount) of the equipment, the upon the latters debt. Lo, as vendor not good faith,
latter quoted such at $1,700 FOB Indiana. Being shall be liable for the existence and legality of the
agreeable to the price (plus 10% commission plus all credit at the time of the sale (but not for the solvency
other expenses), Arco formally authorized the order. of the debtor). He is bound by certain warranties. In this
The following year, both parties agreed for another case, since the assignment he made in favor of KJS has
order of sound reproducing equipment on the same already been compensated, he should still be liable to
terms as the first at $1,600 plus 10% plus all other pay KJS for his indebtedness. He should make good the
expenses. warranty and pay the obligation.

ANTONIO | HIPOLITO | IMPERIAL | ZARAGOSA 2


!"#$!%&%'$"(%)#*%&%+'%+,-+%
%

PARTIES TO A CONTRACT OF SALE


1. PARAGAS v HEIRS OF DOMINADOR BALACANO properties to each other; the same prohibitions apply
to a couple living in as husband and wife without the
FACTS: Balancano, married to Lorenza, owned 2 parcels benefit of marriage. As public interests dictate, to rule
of land. He was already 81 years old, very weak, could otherwise would put the persons in guilt at better
barely talk, and had been battling with liver disease for position than those legally married.
over a month. On his deathbed, barely a week before he
died, he allegedly signed a Deed of Absolute Sale over
the lots in favor of Paragas Spouses, accompanied by 3. RUBIAS v BATILLER
Atty. De Guzman who proceeded to notarize the same,
alleging that it was a mere confirmation of a previous FACTS: Militante claimed ownership over a parcel of
sale and that Gregorio had already paid P50,000 as land and applied for the registration of the same with
deposit. The Paragas driver was also there to take a the CFI; his counsel was his son-in-law, Atty. Rubias. His
picture of Gregorio signing said deed with a ballpen in claim was dismissed by the trial court, thus he
his hand. There was nothing to show that the contents appealed. Pending appeal, he sold the lot to Atty.
of the deed were explained to Balacano. Paragas then Rubias for P2,000. Batiller, on the other hand, claimed
sold a portion of the disputed lot to Catalino. The to have inherited the same lot from his ancestors who
grandson of Gregorio, Domingo, sought to annul the sale have been in open, public, peaceful, and actual
and the partition. There was no sufficient evidence to possession thereof under a claim of title. Atty. Rubias
support any prior agreement or its partial execution. filed an ejectment suit against Batiller who assailed the
validity of the sale to Rubias. Given the dismissal of
ISSUE: W/N Balacano is incapacitated to enter into a Militantes application, he had thus no right over the
contract of sale said land that he may have validly transferred to Atty.
Rubias.
HELD: YES. A person is not rendered incompetent merely
because of old age; however, when such age has ISSUE: W/N the sale to Atty. Rubias is valid
impaired the mental faculties as to prevent a person
from protecting his rights, then he is undeniably HELD: NO. Even assuming he had title thereto, the sale
incapacitated. He is clearly at a disadvantage, and the of the lot to Atty. Rubias would be null and void for
courts must be vigilant for his protection. In this case, being expressly prohibited by the Civil Code. Lawyers
Balacanos consent was clearly absenthence the sale cannot acquire by purchase the property or rights under
was null and void. The circumstances raise serious litigation over which they take part by virtue of their
doubts on his capacity to render consent. Considering profession. The same rule applies to judges, clerks of
that the Paragas spouses are not owners of the said court, and other judicial officers with respect to the
properties, it only follows that the subsequent sale to same. The purchase in violation of the above provision is
Catalinowho was not in good faithis likewise void. not merely voidable as Atty. Rubias contends; it is VOID
Furthermore, the lots pertained to the conjugal and INEXISTENT from the very beginning. The right to
partnershiphaving been inherited by Balacano during set up the defense of its illegality cannot be waived
his marriage to Lorenza. Thus, it cannot be sold without and, unlike cases involving agents, guardians, or
the latters consent. administrators with respect to the properties under
their charge, it is not susceptible to compromise or
ratification. It is likewise contrary to public policy
2. CALIMLIM-CANULLAS v FORTUN

FACTS: Mercedes and Fernando were married and had 5 4. PHIL. TRUST CO. v ROLDAN
children. Fernando inherited the land upon which their
house was built. Fernando left his family to live with FACTS: Mariano Bernardo, a minor, inherited among
his concubine Corazon. He then sold the said lot with others 17 parcels of land from his deceased father.
the house in favor of Corazon for P2,000. Corazon, Soccoro Roldan was appointed as his guardian. Soccoro
unable to take possession of the house and lot, filed a sought and was granted authority to sell the lots to her
complaint for quieting of title. Mercedes objected brother-in-law Ramos for P14,700. Very shortly after,
alleging that the properties pertained to their conjugal Ramos sold back to Soccoro the same properties for
partnership. P15,000. She then sold 4 parcels to Emilio Cruz. Phil.
Trust Co. replaced Soccoro as guardian and sought to
ISSUE: W/N the sale to Corazon was valid annul all the aforesaid sales.

HELD: NO. The properties pertained to the conjugal ISSUE: W/N the sale to Ramos was valid
partnership of Mercedes and Fernando, thus the sale is
null and void for lack of Mercedes consent and for HELD: NO. Guardianship is the trust of the highest order.
being contrary to morals and public policy. The law In this case, for all intents and purposes, it was as if
generally prohibits spouses from selling or donating

ANTONIO | HIPOLITO | IMPERIAL | ZARAGOSA 3


!"#$!%&%'$"(%)#*%&%+'%+,-+%
%
Soccoro herself purchased the properties of her ward. This falls within the prohibition under Art. 1459 of the
Civil Code. She indirectly sold the properties to herself. the houseallowing Atty. Murillo the option to occupy or
The same applies even though there was no actual lease 40% of the said house and lot. A compromise
malice or collusion proven. Since the sale to Soccoro agreement was entered into where Florencio acquired
was null and void, it only follows that the sales made by both properties. Atty. Murillo installed a tenant in the
Soccoro to Cruz were likewise void. One cannot sell Pugahanay Property; later on, Florencio claimed
what is not his property. exclusive rights over the properties invoking Art. 1491 of
Soccoro tried to correct the problem by allowing the CC. Florencio and Atty. Murillo both died and were
Mariano to re-purchase the said properties for P15,000. succeeded by their respective heirs.
However, the child would still be at a losing end
because it would not entitle him to the fruits of the ISSUE: W/N contingent fees agreed upon are valid
property during the time when he was not in possession
thereof. The SC annulled the sale. HELD: YES. Contingent fees are not contemplated by the
prohibition in Art. 1491 disallowing lawyers to purchase
CLV: Bad ruling because W/N ward is benefited is properties of their clients under litigation. The said
IMMATERIAL. Advantage to ward can easily be forged. prohibition applies only during the pendency of the
litigation. Payment of the contingent fee is made after
the litigation, and is thus not covered by the
5. FABILLO v IAC prohibition. For as long as there is no fraud or undue
influence, or as long as the fees are not exorbitant, the
FACTS: Florencio Fabillo contracted the services of Atty. same as valid and enforceable. It is even recognized by
Murillo to revive a lost case over his inheritance from his the Canons of Professional Ethics.
deceased sister Justinia. He sought to acquire the San However, considering that the contract is vague on
Salvador and Pugahanay Properties that his sister left the matter of division of the shares if Florencio occupies
behind against the latters husband. They entered into a the property; the ambiguity is to be construed against
contract where a contingent fee in favor of Atty. Murillo Atty. Murillo being the one who drafted the contract and
in case the case won was agreed upon. The fee was 40% being a lawyer more knowledgeable about the law. The
of the value of whatever benefit Florencio may derive Court thus invoking the time-honored principle that a
from the suitsuch as if the properties were sold, lawyer shall uphold the dignity of the legal profession,
rented, or mortgaged. It was vague, however, regarding ordered only a contingent fee of P3,000 as reasonable
the fee in case Florencio or his heirs decide to occupy attorneys fees.

SUBJECT MATTER OF SALE


1. POLYTECHNIC UNIVERSITY v CA grasp the whole gamut of transfers where ownership of
a thing is ceded for consideration. Further, judging from
FACTS: The National Development Corp. (NDC) owned the conduct of the parties in this case, all the elements
the NDC Compound, a portion of which was leased to of a valid sale attend. Consent is manifested by the
Firestone Ceramics, which built several warehouses and Memo Order No. 214, the cancellation of liabilities
facilities therein. Since business between NDC and constituted consideration; the subject matter was of
Firestone went smooth, the lease was twice renewed course the property subject of the dispute.
this time conferring upon Firestone a right of first Since a sale was involved, the right of first refusal in
refusal should NDC decide to dispose of the property. favor of Firestone must be respected. It forms an
Also, under the contract, Firestone was obliged to integral part of the lease and is supported by
introduce considerable improvements thereon. considerationFirestone having made substantial
Eventually though, Memo Order No. 214 was issued investments therein. Only when Firestone fails to
ordering the transfer of NDC Compound to the exercise such right may the sale to PUP proceed.
government in consideration of the cancellation of
NDCs P57M debt. Pursuant thereto, NDC transferred the
property to Polytechnic University (PUP). Firestone sued 2. ATILANO v ATILANO
for specific performance invoking its right of first
refusal, and sought to enjoin NDC and PUP from FACTS: Eulogio Atilano I purchased Lot 535 and had it
proceeding with the sale. Both PUP and NDC aver that subdivided into 5 parts (A to E). He occupied Lot A; his
there was no sale involved since ownership of the brother, Eulogio II, occupied Lot E. He then sold lots B,
property remained with the governmentboth C, and D to other persons. He then sold Lot E to his
companies being GOCCs. brother Eulogio II. Both brothers died and their heirs
found out after a survey that Eulogio I actually occupied
ISSUE: W/N there was a sale Lot E and Eulogio II occupied Lot A. Thus, the heirs of
Eulogio II offered to exchange the properties. However,
HELD: YES. The argument of PUP and NDC was the heirs of Eulogio I refused because Lot E was bigger
untenable. GOCCs have personalities separate and than Lot A.
distinct from the government. Sale brings within its

ANTONIO | HIPOLITO | IMPERIAL | ZARAGOSA 4


!"#$!%&%'$"(%)#*%&%+'%+,-+%
%
ISSUE: W/N an exchange of the properties was proper HELD: NO. The subject matter was not yet determinate.
The sugar agreed upon has yet to be segregated from all
HELD: NO. What took place was a simple mistake in other articles. That being the case, there was merely an
drafting the instrument evidencing the agreement executory agreementa promise of sale, and not a
between the brothers. One sells or buys property as he contract of sale itself.
sees it in actual setting and not by the mere lot number Moreover, there was no stipulation that the sugar was
in the certificate of title. The brothers remained in to be derived from his crop; he was at liberty to get it
possession of their respective portions throughout their from whatever source he could find. The obligation he
lives unaware of the mistake in the designation of the incurred was for the delivery of the generic thing. Thus,
lots. In this case, the instrument simply failed to reflect he cannot invoke force majeure under the maxim genus
the true intention of the parties; thus, an exchange of never perishes. His obligation to deliver the sugar is not
the properties is unnecessary. All the heirs should do is extinguished.
to execute mutual deeds of conveyance. Yu Tek is thus entitled to rescind the contract and
recover the money in addition to the stipulated P1,200
as indemnity for losses.
3. MELLIZA v CITY OF ILOILO DD: This rule no longer holds true. Generic things may
now be the subject matter of a contract of sale provided
FACTS: Meliza owned Lot 1214, 9,000 sqm of which she that they have the quality of being DETERMINABLE at
donated to the Mun. of Iloilo for the use of the site of the perfection of the contract.
the Mun. Hall. However, the donation was revoked
because it was inadequate to meet the requirements of
the Arellano Plan. Lot 1214 was later divided into 4 5. NGA v IAC
lots. Meliza then sold Lots C and D to the Municipality;
Lot B was not mentioned in the sale. However, the FACTS: National Grains Authority (now National Food
contract stipulated that the area to be sold to the Authority, NFA) is a government agency created under
Municipality would include such areas needed for the PD 4. One of its incidental functions is the buying of
construction of the City Hall according the Arellano palay grains from qualified farmers. In 1979, Leon
Plan. She then sold the remaining portions of the lots to Soriano offered to sell palay grains to the NFA, through
Villanueva, who then sold the same to Pio. The sale was its Provincial Manager, William Cabal. He submitted the
for such lots not included in the sale to the Mun. of documents required by the NFA for pre-qualifying as a
Iloilo. The City of Iloilo, assuming that Lot B has been seller, which were processed and accordingly, he was
sold in its favor pursuant to the Arellano Plan, then given a quota of 2,640 cavans of palay. The quota noted
donated Lot B to UP. Pio objected and sought to recover in the Farmers Information Sheet represented the
the lots stating that Lot B was not included in the initial maximum number of cavans of palay that Soriano may
sale made by Meliza to the Municipalityand that the sell to the NFA. On 23 and 24 August 1979, Soriano
subject matter of sale should be a determinate thing. delivered 630 cavans of palay. The palay delivered were
not rebagged, classified and weighed. When Soriano
ISSUE: W/N there was a determinate/determinable demanded payment of the 630 cavans of palay, he was
subject matter informed that its payment will be held in abeyance since
Mr. Cabal was still investigating on an information he
HELD: YES. The requirement for the subject matter to received that Soriano was not a bona fide farmer and
be determinate is satisfied in this case. Simple the palay delivered by him was not produced from his
reference to the Arellano Plan would indicate that it farmland but was taken from the warehouse of a rice
could determine what portions of the contiguous land trader, Ben de Guzman. On 28 August 1979, Cabal wrote
(lot B) were needed for the construction of the City Soriano advising him to withdraw from the NFA
Hall. There was no need for a further agreement to warehouse the 630 cavans stating that NFA cannot
establish the lots covered by the sale; thus, the sale is legally accept the said delivery on the basis of the
valid. Besides, the portions of Lot B covered by the sale subsequent certification of the BAEX technician
were practically at the heart of the City Hall site. (Napoleon Callangan) that Soriano is not a bona fide
farmer.
Instead of withdrawing the 630 cavans of palay,
4. YU TEK & CO. v GONZALES Soriano insisted that the palay grains delivered be paid.
He then filed a complaint for specific performance
FACTS: Gonzales received P3,000 from Yu Tek and and/or collection of money with damages against the
obligated himself in favor of the latter to deliver 600 NFA and William Cabal. Meanwhile, by agreement of the
piculs of sugar of the 1st and 2nd grade within 3 months. parties and upon order of the trial court, the 630 cavans
He failed to deliver the sugar and refused to return the of palay in question were withdrawn from the
moneythus Yu Tek sued him. Gonzales, in seeking to warehouse of NFA. In 1982, RTC ruled in favor of Soriano
evade liability, invokes fortuitous event, alleging the and in 1986, CA affirmed decision of RTC.
total failure of his crop.
ISSUE: W/N there was a perfected contract of sale
ISSUE: W/N there was perfected contract of sale
HELD: YES. In the present case, Soriano initially offered

ANTONIO | HIPOLITO | IMPERIAL | ZARAGOSA 5


!"#$!%&%'$"(%)#*%&%+'%+,-+%
%
to sell palay grains produced in his farmland to NFA. 7. NOOL v CA
When the latter accepted the offer by noting in
Sorianos Farmers Information Sheet a quota of 2,640 FACTS: One lot formerly owned by Victorio Nool has an
cavans, there was already a meeting of the minds area of 1 hectare. Another lot previously owned by
between the parties. The object of the contract, being Francisco Nool has an area of 3.0880 hectares. Both
the palay grains produced in Sorianos farmland and the parcels are situated in San Manuel, Isabela. Spouses
NFA was to pay the same depending upon its quality. Conchita Nool and Gaudencio Almojera (plaintiffs)
The contention that since the delivery were not alleged that they are the owners of the subject land as
rebagged, classified and weighed in accordance with the they bought the same from Victorio and Francisco Nool,
palay procurement program of NFA, there was no and that as they are in dire need of money, they
acceptance of the offer thus this is a clear case of obtained a loan from DBP, secured by a real estate
policitation or an unaccepted offer to sell, is untenable. mortgage on said parcels of land, which were still
The fact that the exact number of cavans of palay to registered in the names of Victorino and Francisco Nool,
be delivered has not been determined does not affect at the time, and for the failure of the plaintiffs to pay
the perfection of the contract. Article 1349 of the New the said loan, including interest and surcharges, totaling
Civil Code provides that the fact that the quantity is P56,000.00, the mortgage was foreclosed; that within
not determinate shall not be an obstacle to the the period of redemption, the plaintiffs contacted
existence of the contract, provided it is possible to Anacleto Nool for the latter to redeem the foreclosed
determine the same, without the need of a new properties from DBP, which the latter did; and as a
contract between the parties. In the present case, result, the titles of the 2 parcels of land in question
there was no need for NFA and Soriano to enter into a were transferred to Anacleto; that as part of their
new contract to determine the exact number of cavans arrangement or understanding, Anacleto agreed to buy
of palay to be sold. Soriano can deliver so much of his from Conchita the 2 parcels of land under controversy,
produce as long as it does not exceed 2,640 cavans. for a total price of P100,000.00, P30,000.00 of which
price was paid to Conchita, and upon payment of the
balance of P14,000.00, the plaintiffs were to regain
6. JOHANNES SCHUBACK & SONS PHIL. TRADING possession of the 2 hectares of land, which amounts
CORP. v CA spouses Anacleto Nool and Emilia Nebre (defendants)
failed to pay, and the same day the said arrangement
FACTS: SJ Industrial, through Ramon San Jose, was made; another covenant was entered into by the
approached Schuback & Sons Phil. Trading (SSPT) to parties, whereby the defendants agreed to return to
purchase bus spare parts. He submitted the list of parts plaintiffs the lands in question, at anytime the latter
he wanted and SSPT coordinated with its Germany have the necessary amount; that latter asked the
Office to quote the prices, and forwarded its formal defendants to return the same but despite the
offer to SJ Industrial, containing the prices, item intervention of the Barangay Captain of their place,
numbers, descriptions, etc. SJ informed SSPT of his defendants refused to return the said parcels of land to
desire to purchase such items and promised to submit plaintiffs; thereby impelling the plaintiffs to come to
the quantity per unit. SJ then submitted such quantities court for relief. On the other hand, defendants
needed to SSPTs GM, Mr. Reichert. San Jose indicated theorized that they acquired the lands in question from
the same in the Purchase Order with the inscription the DBP, through negotiated sale, and were misled by
this will serve as our initial purchase order. PO will plaintiffs when defendant Anacleto Nool signed the
include 3% discount. SSPT immediately ordered the private writing, agreeing to return subject lands when
products from Germany to avail of the old prices plaintiffs have the money to redeem the same;
partial deliveries of which were made. Then, for his defendant Anacleto having been made to believe, then,
failure to secure letters of credit, SJ failed to purchase that his sister, Conchita, still had the right to redeem
the same and alleged that there was no perfected the said properties.
contract of sale. Thus, SSPT sought damages. It should be stressed that Manuel S. Mallorca,
authorized officer of DBP, certified that the 1-year
ISSUE: W/N there was a perfected contract of sale redemption period and that the mortgagors right of
redemption was not exercised within this period. Hence,
HELD: YES. Quantity is immaterial in the perfection of a DBP became the absolute owner of said parcels of land
contract of sale. What is important is the meeting of the for which it was issued new certificates of title. About 2
minds as to the object and cause of the sale. There was years thereafter, DBP entered into a Deed of Conditional
already a meeting of the minds in this case from the Sale involving the same parcels of land with Anacleto
moment SJ manifested that he will order the parts, Nool as vendee. Subsequently, the latter was issued new
although he will communicate quantities later on. In certificates of title in 1988.
fact, he indeed communicated such needed quantities RTC ruled in favor of Anacleto Nool. CA affirmed.
this goes to the execution of the contract of sale
already. By ordering the parts, SJ acceded to the prices ISSUE: W/N there was a valid contract of sale between
offered by SSPT. On the other hand, SSPT acceded to Anacleto and Conchita
SJs request for discount by immediately ordering the
parts. SJ Industrial is thus liable for damages HELD: NO. Article 1459 of the Civil Code provides that
the vendor must have a right to transfer the ownership

ANTONIO | HIPOLITO | IMPERIAL | ZARAGOSA 6


!"#$!%&%'$"(%)#*%&%+'%+,-+%
%
thereof [object of the sale] at the time it is delivered. to the goods than the seller had, unless the owner of the
Here, delivery of ownership is no longer possible. The goods is by his conduct precluded from denying the
sellers can no longer deliver the object of the sale to sellers authority to sell. In the present case, there is
the buyers, as the buyers themselves have already no allegation at all that petitioners were authorized by
acquired title and delivery thereof from the rightful DBP to sell the property to the private respondents.
owner, the DBP. Thus, such contract may be deemed to Further, the contract of repurchase that the parties
be inoperative and may thus fall, by analogy, under item entered into presupposes that petitioners could
5 of Article 1409 of the Civil Code: Those which repurchase the property that they sold to private
contemplate an impossible service. respondents. As petitioners sold nothing, it follows
Article 1505 of the Civil Code provides that where that they can also repurchase nothing. In this light,
goods are sold by a person who is not the owner thereof, the contract of repurchase is also inoperative and by the
and who does not sell them under authority or with same analogy, void.
consent of the owner, the buyer acquires no better title

PRICE AND OTHER CONSIDERATION


1. MAPALO v MAPALO debt. Apparently, it was a Deed of Sale.
In 1980, Rongavilla went to Dela Cruz' place and asked
FACTS: Miguel and Candida Mapalo were illiterate them to vacate the lot. Suprised by this, Dela Cruz
farmers and owned a parcel of land. Since Maximo discovered the misrepresentation her niece made when
Mapalo was to be married, they donated to him the she signed the document. She the filed an action with
eastern half of the land. Maximo, however, deceived the RTC to have the purported Deed of Sale declared
them by making them sign an instrument donating the null and inexistent for lack of consent and
entire lot. There was a consideration for P5,000 stated consideration.
in the deed, but the spouses never received anything.
Miguel built a fence to divide the lot and continued to ISSUE: W/N there was a valid sale
occupy the western part. Maximo then registered the
entire lot and 13 year after, sold the same to the HELD: NO. Rongavilla and Jimenez were able to secure
Narcisos who took possession only of the eastern half. the signature of Dela Cruz in the Deed of Absolute Sale
Later on, the Narcisos sought to be declared owners of through fraud and there was no consideration
the entire land; the spouses claimed that the sale to the whatsoever for the alleged sale. The consent was not
Narcisos was void for lack of consideration. The CA only vitiated, but it was not given at all. Since there was
declared that the sale was merely voidable and the no consent, the deed of absolute sale is null and void ab
action by the spouses was barred by prescription, being initio.
filed after 4 years from the discovery of the fraud.

ISSUE: W/N there was a valid contract of sale 3. MATE v CA

HELD: Consideration was totally absent; the P5,000 FACTS: Josefina approached Fernando asking for help.
price stipulated was never received/delivered to the Her family was to be sued by Tan for issuing rubber
spouses. Thus, the sale to the Narcisos was VOID ab checks; thus she asked him to cede his 3 lots to Tan and
initio for want of consideration. The inexistence of the it will be Josefina who will repurchase them for him. He
contract is permanent and cannot be the subject of initially rejected her offer. Then, Josefina issued him 2
prescription. The Narcisos are also in bad faiththey checks, one for P1.4M, pertaining to the value of the
had knowledge of the true nature and extent of lot, and another for P420,000 corresponding to 6
Maximos right over the land. months interests. He agreed, drafted the instrument
himself, and ceded his properties to Tan. Later, both
checks bounced; he sued Tan for annulment of the sale
2. RONGAVILLA v CA for lack of consideration since he never received
anything. He also sued Josefina criminally, but
FACTS: Both spinsters and unschooled in English, absconded.
Mercedes and Florencia dela Cruz are the aunts of
Rongavilla. Dela Cruz co-owned a parcel of land (1/2 ISSUE: W/N there was a valid contract of sale
pro-indiviso) in Las Pinas with another niece named
Juanita Jimenez (elder sister of Rongavilla), who kept HELD: YES. There was consideration in the form of the
the OCT, as well as the TCT after it was subdivided. check for P420,000. It was his fee for executing the
In 1976, Dela Cruz borrowed P2,000 from Rongavilla sale. It was not only kindness that impelled him to cede
for the repair of their dilapidated rooftop. A month his property, it was also his interest for profit.
later, Rongavilla and Jimenez visited their aunts' home That he never received money is of no moment; a sale
and brought with them a document for the signature of is a consensual contract. He also tacitly admitted to the
their aunts. While the document was in English and upon sale when he filed criminal charges against Josefina.
inquiry by Dela Cruz what it was about, Rongavilla Fernando, being a lawyer, has no one else to blame but
answered that it was merely evidencing the P2,000

ANTONIO | HIPOLITO | IMPERIAL | ZARAGOSA 7


!"#$!%&%'$"(%)#*%&%+'%+,-+%
%
himself for the loss. He acted negligently our of desire
for profit. HELD: YES. There was consideration. Its apparent
inadequacy is of no moment since the usual practice in
deeds of conveyance is to place a nominal amount
4. YU BUN GUAN v CA although there is more valuable consideration given.
Consideration is presumed to exist. He who alleges
FACTS: Yu Bun Guan and Ong are married since 1961 and otherwise assumes the burden of proof. The one peso
lived together until she and her children were was not the consideration, but rather the other valuable
abandoned by him in 1992, because of his incurable considerations.
promiscuity, volcanic temper, and other vicious vices.
In 1968, out of her personal funds, Ong purchased a
parcel of land (Rizal Property) from Aurora Seneris. 6. BAGNAS v CA
Also, during their marriage, they purchased a house and
lot out of their conjugal funds. FACTS: Hilario died with no will and was survived only
Before their separation in 1992, she reluctantly agreed by collateral relatives. Bagnas (et al) were the nearest
to execute a Deed of Sale of the Rizal Property on the kin. Retonil (et al) were also relatives but to a farther
promise that Yu Bun Guan would construct a commerical extent. They claimed ownership over 10 lots from the
building for the benefit of the children. He suggested estate of Hilario presenting notarized and registered
that the property should be in his name alone so that Deeds of Sale (in Tagalog) where the consideration for
she would not be involved in any obligation. The the lands was P1 and services rendered, being rendered,
consideration for the sale was the execution of a Deed and to be rendered. Bagnas argued that the sales were
of Absolute Sale in favor their children and the payment fictitious, while Retonil claimed to have done many
of the loan he obtained from Allied Bank. things for Hilariosuch as nursing him on his deathbed.
However, when the Deed of Sale was executed in
favor of Yu Bun Guan, he did not pay the consideration ISSUE: W/N there was a valid contract of sale
of P200K, supposedly the "ostensible" valuable
consideration. Because of this, the new TCT issued in his HELD: NO. At the onset, if a contract has no
name was not delivered to him by Ong. consideration, it is not merely voidable, but VOIDand
Yu Bun Guan then filed for a Petition for Replacement even collateral heirs may assail the contract. In this
of the TCT, with an Affidavit of Loss attached. Ong, on case, there was no consideration. Price must be in
the other hand, executed an Affidavit of Adverse Claim money or its equivalent; services are not the equivalent
and asked that the sale be declared null and void . of money insofar as the requirement of price is
RTC ruled in favor of Ong. CA affirmed. concerned. A contract is not one for sale if the
consideration consists of services. Not only are they
ISSUE: W/N there was a valid contract of sale vague, they are unknown and not susceptible of
determination without a new agreement between the
HELD: NO. It is clear from the findings of the lower parties.
courts that the Deed of Sale was completely simulated
and thus, VOID without effect. No portion of the
P200,000 consideration stated in the Deed was ever 7. REPUBLIC v PHIL. RESOURCES DEV. CORP.
paid. And, from the facts of the case, it is clear that
neither party had any intention whatsoever to pay that FACTS: The Republic brought an action against Apostol
amount. Instead, the Deed of Sale was executed merely for the collection of sums owing to it for his purchase of
to facilitate the transfer of the property to petitioner Palawan Almaciga and other logs. His total debt
pursuant to an agreement between them to enable him amounted to some P34,000. PRDC intervened claiming
to construct a commercial building and to sell the Juno that Apostol, as President of the company, without prior
property to their children. Being merely a subterfuge, authority, took goods (steel sheets, pipes, bars, etc)
that agreement cannot be taken as a consideration for from PRDC warehouse and appropriated them to settle
the sale. his personal debts in favor of the government. The
Republic opposed the intervention of PRDC, arguing that
price is always paid in money and that payment in kind
5. ONG v ONG is no payment at all; hence, money and not the goods of
PRDC are under dispute.
FACTS: For an in consideration of P1 and other valuable
considerations, Imelda Ong transferred through a Deed ISSUE: W/N payment in kind is equivalent to price paid
of Quitclaim her rights over a ! portion of a parcel of in money
land to Sandra. Later on, she revoked the Deed and
donated the whole property to her son, Rex. Sanda, HELD: YES. Price may be paid in money or ITS
through her guardian, sought to recover ownership and EQUIVALENTin this case, the goods. Payment need not
possession thereof. Imelda alleged that the sale was be in the form of money. The prices for the goods have,
void for lack of consideration. in fact, been assessed and determined. PRDC thus has a
substantial interest in the case and must be permitted
ISSUE: W/N there was a valid contract of sale

ANTONIO | HIPOLITO | IMPERIAL | ZARAGOSA 8


!"#$!%&%'$"(%)#*%&%+'%+,-+%
%
to interveneits goods paid out without authority being mortgage over another set of properties of RRRC.
under dispute in this case. Likewise, it defaulted and the properties were
foreclosed. However, RRRC was able to negotiate with
the Bank for the redemption of the properties by was of
8. NAVARRA v PLANTERS DEV. BANK a concession whereby the Bank allowed RRRC to refer to
it would-be buyers of the properties who would remit
FACTS: Navarra spouses are the owners of 5 parcels of their payments directly to the Bank, which would then
land in BF Homes, Paranque. In 1982, they obtained a be considered as redemption price for RRRC. Eventually,
loan of P1.2M from Planters Bank, secured by a these were sold and payments made directly to the Bank
mortgage over these parcels of land. Unfortunately, were in excess by P300K for the redemption price.
they defaulted to pay their obligation and thus, Planters In the meantime, Jorge Navarra requested that they
Bank foreclosed the property. They were not able to repurchase their house and lot for P300K, which the
redeem the property as well. Bank agreed. Accordingly, Jorge Navarra requested
On the other hand, RRRC Dev. Corp. is a real estate further that the excess payment of RRRC be applied as
company owned by the parents of Carmelita Navarra. It down payment for their repurchase. For his failure to
obtained a loan from Planters Bank secured by a
submit a board resolution from RRRC authorizing such, this fact cannot lead to the conclusion that a contract of
the Bank refused to apply the excess to his repurchase. sale had been perfected. Before a valid and binding
In 1988, a portion of the lots was sold to Gatchalian contract of sale can exist, the manner of payment of the
Realty. Navarra spouses filed for specific performance purchase price must first be established since the
against Planters Bank, alleging that there was a agreement on the manner of the payment goes into the
perfected contract of sale (P1.8M, with P300K price such that a disagreement on the manner of
downpayment). payment is tantamount to a failure to agree on the
RTC ruled in favor of Navarra spouses. CA reversed. price.
Moreover, the letter/offer failed to specify a definite
ISSUE: W/N there was a valid contract of sale (consider amount of the purchase price for the sale/repurchase of
the repurchase as a sale) the properties. It merely stated that it will be based on
the redemption value plus accrued interest at the
HELD: NO. While the letters indicate the amount of prevailing rate up to the date of the sales contract.
P300K as downpayment, they are completely silent as to Clearly, the lack of a definite offer on the part of the
how the succeeding installment payment shall be made. Navarra spouses could not possibly serve as the basis of
At most, the letters merely acknowledge that the their claim that the sale was perfected.
downpayment was agreed upon by the parties. However,

FORMATION OF CONTRACT OF SALE


1. MANILA METAL CONTAINER CORP. v PNB less the P725K deposit. Manila Metal, again, rejected
this offer and filed a complaint against PNB for the
FACTS: Manila Metal was the owner of a parcel of land in annulment of foreclosure or specific performance,
Mnadaluyong. To secure a P900K loan it obtained from contending that there was a valid contract of sale
PNB, Manila Metal executed a real estate mortgage over between Manila Metal and SAMD.
the lot. PNB later granted Manila Metal a new credit In 1993, while the case was pending, Manila Metal
accommodation of P1M. Manila Metal secured another offered to repurchase at P3.5M, but PNB rejected
loan of P653K from PNB. because the market value of the property was at P30M.
In 1982, PNB sought to have the property foreclosed Manila Metal offered again at P4.25M but was rejected
and sold at a public auction. PNB was the highest again.
bidder. Manila Metal requested an extension of time to
redeem the property and to repurchase such on ISSUE: W/N there was a valid contract of sale
installment.
The Special Assets Management Department (SAMD) HELD: NO. There was no perfected contract of sale
prepared a statement of account and as of 1984, Manila between PNB and Manila Metal because there was no
Metal's obligation amounted to P1.6M, which includes agreement as to the price certain. The Statement of
the bid price, interests, advances of insurance Account prepared by SAMD cannot be classified as a
premiums, advances on realty taxes, etc. When apprised counter-offer. It is simply a recital of its total monetary
of the statement of account, Manila Metal remitted claims against Manila Metal. The amount stated therein
P725K to PNB as deposit to repurchase. could not be considered as a counter-offer since it was
In the meantime, SAMD recommended that Manila only a recommendation subject to PNB's Board of
Metal be allowed to repurchase for P1.6M. PNB, Directors' approval. Neither can the receipt of P725K by
however, rejected the recommendation and offered the SAMD be regarded as evidence of a perfected contract
property at P2.66M, its minimum market value. Manila of sale. The amount is merely an acknowledgment of the
Metal refused and reiterated that it already acceded to receipt of P725K as deposit to repurchase the property.
SAMD's offer, to which it remitted P725K. It was accepted by respondent on the condition that the
In 1985, PNB accepted the offer but for P1.9M cash purchase price will still be approved by the Board of

ANTONIO | HIPOLITO | IMPERIAL | ZARAGOSA 9


%
Directors. Pending such approval, Manila Metal cannot
legally claim that PNB is already bound by any contract HELD: NO. The Deeds of Assignment were not option
of sale with it. contracts, which may be enforced by Tayag. Not being
the legal owners of the property, the tenants had no
right to confer upon Tayag the option, more so, the
2. CARCELLER v CA exclusive right to buy the property.

FACTS: Carceller leased 2 parcels of land owned by


State Investment Houses (SIHI), the period being 18 4. VILLAMOR v CA
months at P10,000/month rent. Under the lease, SIHI
guaranteed Carceller the exclusive right and option to FACTS: The Villamors purchased from Macaria ! of the
purchase the said lots within the lease period for the latters land for a price considerably higher than the
aggregate amount of P1.8M. Around 3 weeks before the prevailing market price. They then executed a Deed of
end of the lease period, SIHI informed Carceller of the Option stating that the only reason why the Villamors
impending termination of the lease and the short period agreed to purchase the said lot is because Macaria
left for him to purchase. He begged for an extension, agreed to confer upon them the exclusive right to
but SIHI refused. Nevertheless, SIHI offered the property purchase the other half of the land. Such sale under the
to him for lease for another year, but this time, it also deed may be imposed whenever the need for the sale
offered it for sale to the public. Carceller thus sued SIHI arises on the part of either party. Macaria sought to
for specific performance to compel SIHI to execute a repurchase the land, but the Villamors refused. Instead,
Deed of Sale in his favor. the Villamors exercised their option to purchase the
other half of the property. Macaria refused, thus the
ISSUE: W/N Carceller may still exercise the option to Villamors filed a case for specific performance. Macaria
purchase the property averred that the option is void for lack of consideration.

HELD: YES. Even if Carceller failed to purchase the ISSUE: W/N the option contract is void for lack of
property within the said period, still equity must consideration
intervene. He had introduced substantial improvements
thereon; to rule against him would cause damage to HELD: NO. The Option Contract is supported by a
himand SIHI does not stand to gain much therefrom. considerationthat being the difference of the agreed
SIHI clearly intended to sell the lot to him considering price and the market price of the other half of the land,
that it was under financial distress, that is constantly which was sold to the Villamors. Thus, it is valid and
reminded him of the option and the impending deadline. may be enforced by the Villamors. The consideration
The delay of 18 days is not substantial. Carcellers letter may consist of anything of value.
to SIHI expressing his intent to purchase the lot is fair The option was, in fact, the only reason why they
notice of intent to exercise the option despite the purchased the other half for an expensive price. Since
request for extension. Carceller should thus be allowed the Villamors exercised their option, this is tantamount
to buy the lots. to an acceptance of the offera valid and obligatory
contract of sale was thus perfected.

3. TAYAG v LACSON
5. SANCHEZ v RIGOS
FACTS: Angelica Lacson and her children were registered
owners of agricultural lands. Tiamzon and others were FACTS: Sanchez and Rigos executed an Option to
their farmer-tenants. The tenants executed a Deed of Purchase where Rigos agreed, promised, and committed
Assignment in favor of Tayagassigning to the latter to sell to Sanchez a parcel of land in Nueva Ecija for
their rights to purchase the lands as tenant-tillers of the P1,510. In spite of the repeated tenders made by
landholdings possessed by them at P50.00 per sqm. This Sanchez, Rigos refused to sell the same. Thus, Sanchez
was subject to the conditions that (1) Lacson, the consigned the amounts and filed a case for specific
landowner, would agree to sell the same parels and (2) performance. Rigos alleged that the contract between
that there are no more legal impediments to the them was a unilateral promise to sell, which is not
assignment. Tayag invited the tenants to a meeting to supported by any consideration, hence, it is not binding.
discuss the agreement, but the latter did not attend and
wrote Tayag that they have decided to sell their rights ISSUE: W/N there was a valid option contract
to the Lacsons instead because he allegedly betrayed
their trust by filing a certain lawsuit. Tayag thus filed a HELD: NO. The promisee (Sanchez) cannot compel the
Complaint before the RTC asking that the court fix the promissor (Rigos) to comply with the promise unless the
period for the payment; he also asked for a Writ of former can establish that the promise was for a
Preliminary Injunction against Lacson and the tenants to consideration. The burden of proof to establish the
enjoin them from accepting any offers for sale made by existence of the consideration lies with Sanchez.
the tenants. Therefore, there was no valid option contract in this
case. However, an option without consideration is a
ISSUE: W/N the assignment was in the form of an option mere offer, which is not binding until accepted. But
contract from the moment it is accepted before it is withdrawn,
ANTONIO | HIPOLITO | IMPERIAL | ZARAGOSA 10
%
a valid contract of sale arises. In this case, even though exercise of the option need not be coupled with actual
there was no option contract, there was nevertheless an payment so long as such payment is made upon the
offer and acceptance enough to constitute a valid fulfillment of the owners undertaking to deliver the
contract of sale. property. This is based on the principle that such option
contracts involve reciprocal obligationsand one does
not incur delay if the other party fails or refuses to
6. VASQUEZ v CA comply with his respective obligation. That being the
case, there was no need for Nietes to deposit the said
FACTS: The Vallejera spouses sought to recover from amountsand his withdrawal thereof does not affect his
Vasquez an agricultural lot, which they previously sold right.
to him. Along with the previous execution of a Deed of
Sale, the parties also executed a Right of Repurchase
allowing Vallejera to repurchase the said estate. 8. ANG YU ASUNCION v CA
Vasquez resisted the redemption arguing that the option
to buy was not supported by any considerationand thus FACTS: The Unijeng spouses owned certain residential
not binding upon him. and commercial spaces leased by Ang Yu. They offered
to sell the said units to Ang Yu on several occasions and
ISSUE: W/N there was a valid option contract for P6M. Ang Yu made a counter offer for P5M. The
Unijeng spouses asked Ang Yu to specify his terms in
HELD: NO. It is apparent that the Right to Repurchase writing but the latter failed to do so. They failed to
was not supported by any consideration. Thus, in order arrive at any definite agreement. When Ang Yu
for the doctrine under Sanchez v Rigos to apply, giving discovered that the spouses were planning to sell the
rise to a valid contract of sale, it must be shown that property to others, he sued them for specific
the promissee (Vallejera) accepted the right of performance. While the case was pending, the spouses
repurchase before it was withdrawn by Vasquez. In this sold the units to Buen Realty for P15M.
case, no such acceptance was made. The vendor a retro
(Vallejera) must make actual and simultaneous tender ISSUE: W/N there was a perfected contract of sale
of payment and consignation. Mere expressions of between Unijeng and Ang Yu
readiness and willingness to repurchase are insufficient.
Their ineffectual acceptance allowed Vasquez to HELD: NO. There was no perfected contract of sale yet
withdraw the offer through his refusal to sell the lot. since there was yet any meeting of the minds. Thus,
Vasquez thus cannot be compelled to sell the lot. there is no ground for specific performance. During the
negotiation stage, any party may withdraw the offer
madeespecially if it was not supported by any
7. NIETES v CA consideration.
An Option Contract of a Right of First Refusal is
FACTS: Nietes leased from Dr. Garcia the Angeles separate and distinct from the actual contract of sale
Educational Institute; the contract contained an Option which is the basis for specific performance. The remedy
to Buy the land and school buildings within the period of available to Any Yu, in case the withdrawal was made
the lease. It also stipulated that the unused payment capriciously and arbitrarily, would be to sue on the basis
will be applied to the purchase price of the school. of abuse of right. In case there was an option contract,
Nietes paid Garcia certain sums in excess of the rent, timely acceptance would create an obligation to sell on
which Garcia acknowledged as forming partial payment the part of the vendor; but no such circumstance
of the purchase price of the property. Later on, Garcia, attends in this case.
through counsel, wrote Nietes informing him of his
decision to rescind the contract due to certain violations
of the contractsuch as poor maintenance, lack of 9. EQUATORIAL REALTY DEV. INC. v MAYFAIR
inventory of school equipment, and the use of another THEATER INC.
name for the said school. Nietes replied by informing
Garcia that he decided to exercise his Option to Buy, FACTS: For its theaters, Mayfair was leasing a portion of
but Garcia refused to sell. Nietes thereafter deposited the property in CM Recto, which Carmelo owns. Under
the balance of the price to Agro-Industrial Bank, but he the lease agreement, if Carmelo should decide to sell
later withdrew the said amounts. CA ruled in favor of the leased premises, Mayfair shall be given 30 days
Garcia stating that the full purchase price must be paid exclusive option to purchase the same. Carmelo,
before the Option to Buy may be exercised. Thus, Nietes through Henry Yang, informed the president of Mayfair
brought the matter to the SC. that the former is interested in selling the whole CM
Recto propertyand that Araneta offered to purchase
ISSUE: W/N actual payment is needed before one may the same for $1.2M. Mayfair twice replied through a
exercise the option to buy letter of its intention to exercise its right to
repurchasebut Carmelo never replied. Thereafter,
HELD: NO. There is nothing in the contract that required Carmelo sold the entire property to Equatorial Realty
Nietes to pay the full price before he could exercise the for some P11M. Thus, Mayfair instituted an action for
option. It was sufficient that he informed Garcia of his specific performance and annulment of the sale.
choice and that he was at that time ready to pay. The
ANTONIO | HIPOLITO | IMPERIAL | ZARAGOSA 11
%
Carmelo alleges that the right, being an option contract, the MOA, Ayala was to undertake the development of
is void for lack of consideration. the lands except the retained area. Under Par. 5.15 of
the MOA, Ayala agreed to give Vasquez a first option to
ISSUE: W/N the right to repurchase is an option contract purchase the 4 adjacent lots to the retained area at the
and void for lack of consideration prevailing market price at the time of the purchase. A
case was filed by one of the former sub-contractors of
HELD: NO. The clause in the lease agreement was NOT Conduit against Ayala causing a 6-year delay in the
an option contract, but a RIGHT OF FIRST REFUSAL. It development of the project. Now, Vasquez comes
was premised on Carmelos decision to sell the said forward invoking Par. 5.15 claiming that it was a valid
property. It also did not contain a stipulation as to the option contract, and that Ayala should sell to him the
price of said property. The requirement of separate said property at the 1984 prevailing price. Ayala offered
consideration does not apply to a right of 1st refusal to sell the said properties to Vasquez at the prevailing
because consideration is already an integral part of the prices (1990); but the latter refused to accept. Ayala
lease. Carmelo violated such right by not affording discounted the price from P6,500/sqm to P5,000/sqm,
Mayfair a fair chance to negotiate. It abandoned the but still, Vasquez refused.
negotiations arbitrarily.
Equatorial was likewise in bad faith; it was well aware ISSUE: W/N there was a valid option contract given to
of the right conferred upon Mayfair because its lawyers Vasquez
had ample time to review the contract. That being the
case, the contract between Carmelo and Equatorial is HELD: NO. Par. 5.15 was NOT an option contract, but a
rescissible. Mayfair should be allowed to purchase the RIGHT OF FIRST REFUSAL. It was predicated upon
entire property for the price offered by Equatorial. Ayalas decision to sell the said properties. The price
Rights of First Refusal are also governed by the law on was also not specified. It was also not supported by any
contracts, not the amorphous principles on human independent consideration. By twice refusing to accept
relations. Ayalas offers, Vasquez lost his right to repurchase.
Ayala did not breach its obligation.

10. PARANAQUE KINGS ENTERPRISES INC v CA


12. RIVERA FILIPINA INC v CA
FACTS: Catalina owned 8 parcels of land leased to Chua,
who assigned its rights thereto to Lee Ching Bing, who, FACTS: In 1982, Reyes executed a 10-year (renewable)
in turn, assigned said rights to Paranaque King Contract of Lease with Riivera Filipina over a parcel of
Enterprises, which introduced significant improvements land in EDSA. Under such contract, the lessee is given a
on the premises. Under the lease agreement, in case of right of first refusal should the lessor decide to sell the
sale, the lessee shall have the option or priority to buy property during the terms of the lease.
the said properties. Catalina, in violation of the said Such property was subject of a mortgage executed by
stipulation, sold the lot to Raymundo for P5M. Reyes in favor of Prudential Bank. Since Reyes failed to
Paranaque King notified her of the said breach, and she pay the loan with the bank, it foreclosed the mortgage
immediately had the lots reconveyed. She then offered and it emerged as the highest bidder in the auction sale.
the lot to Paranaque King for P15M; but the latter Realizing that he could not redeem the property, Reyes
refused claiming that the offer was ridiculous. decided to sell it and offered it to Riviera Filipina for
Catalina thereafter sold it again to Raymundo for P9M. P5,000/sqm. However, it bargained for P3,500/sqm.
Reyes rejected such offer. After 7 months, it again
ISSUE: W/N there was compliance with the Right of First bargained for P4,000/sqm, which again was rejected by
Refusal assigned to Paranaque King Reyes who asked for P6,000/sqm price. After 2 months,
it again bargained for P5,000/sqm, but since Reyes
HELD: NO. In a Right of First Refusal, the seller cannot insisted on P6,000/sqm price, he rejected Riviera's
offer the property to another for a lower price or under offer.
terms more favorable. It must be offered under the Nearing the expiry of the redemption period, Reyes
same terms & conditions to Paranaque King; otherwise, and Traballo (his friend) agreed that the latter would
the right of first refusal becomes illusory. Only if buy the same for P5,300. But such deal was not yet
Paranaque King fails to meet the offer may the property formally concluded and negotiations with Riviera Filipina
be offered for sale to another buyerand under the once again transpired but to no avail.
same terms and conditions as well. The Right of First In 1989, Cypress and Cornhill Trading were able to
Refusal may also be validly transferred or assignedas in come up with the amount sufficient to cover the
this case. redemption money, with which Reyes paid to Prudential
Bank to redeem the property. Subsequently, a Deed of
Absolute Sale was executed in favor of Cypress and
11. VASQUEZ v AYALA CORP. Cornhill for P5.4M. Cypress and Cornhill mortgaged the
property in favor of Urban Dev. Bank for P3M.
FACTS: In 1984, Ayala Corp. entered into a Memorandum Riviera Filipina filed a suit against Reyes, Cypress and
of Agreement with Dr. Vasquez buying the latters Cornhill on the ground that they violated its right of first
shares with Conduit Developmentwhich constitute refusal under the lease contract. RTC ruled in favor of
some 50 hectares of the land in Ayala Alabang. Under
ANTONIO | HIPOLITO | IMPERIAL | ZARAGOSA 12
%
Reyes, Cypress, and Cornhill. On appeal, CA affirmed Dela Vida took possession of it and promptly built an
the decision of the RTC. edifice worth P800,000. However, on the said date, the
sale did not materialize. Consequently, Macion filed a
ISSUE: W/N Riviera Filipina lost its right of first refusal complaint for unlawful detainer against Dela Vida, while
Dela Vida countered with a complaint for reformation of
HELD: YES. As clearly shown by the records and the contract to sell. These differences were eventually
transcripts of the case, the actions of the parties to the settled.
contract of lease, Reyes and Riviera, shaped their In 1992, both parties entered into a compromise
understanding and interpretation of the lease provision agreement where Macion will give Dela Vida 5 months to
"right of first refusal" to mean simply that should the raise P2.06M and in case of failure to do so, Dela Vida
lessor Reyes decide to sell the leased property during would vacate the premises. After 2 months, Dela Vida
the term of the lease, such sale should first be offered alleged that they had negotiated a loan from BPI and
to the lessee Riviera. And that is what exactly ensued requested Macion to execute the contract to sell in its
between Reyes and Riviera, a series of negotiations on favor. However, Macion refused, which prompted Dela
the price per square meter of the subject property with Vida to file an urgent motion for an order to direct
neither party, especially Riviera, unwilling to budge Macion to execute the contract to sell. In return, Macion
from his offer, as evidenced by the exchange of letters filed a motion for execution of judgment alleging that
between the two contenders. after 5 months, Dela Vida was not able to settle their
It can clearly be discerned from Rivieras letters that obligations with Macion. RTC ruled in favor of Dela Vida.
Riviera was so intractable in its position and took
obvious advantage of the knowledge of the time ISSUE: W/N it was proper to execute a contract to sell in
element in its negotiations with Reyes as the favor of Dela Vida
redemption period of the subject foreclosed property
drew near. Riviera strongly exhibited a "take-it or leave- HELD: YES. Although the compromise agreement (par. 7)
it" attitude in its negotiations with Reyes. It quoted its does NOT give Dela Vida the right to demand from
"fixed and final" price as Five Thousand Pesos Macion the execution of the contract to sell in its favor.
(P5,000.00) and not any peso more. It voiced out that it From this paragraph, it is clear that Macion is obliged to
had other properties to consider so Reyes should decide execute a Deed of Sale and not a Contract to Sell upon
and make known its decision "within fifteen days." payment of the full price of P2.06M. Thereafter, Macion
Riviera even downgraded its offer when Reyes offered will turn over to Dela Vida the TCT.
anew the property to it, such that whatever amount HOWEVER, a review of the facts reveals that even
Reyes initially receives from Riviera would absolutely be prior to the signing of the compromise agreement, both
insufficient to pay off the redemption price of the parties had entered into a contract to sell, which was
subject property. Naturally, Reyes had to disagree with superseded by a compromise agreement. This
Rivieras highly disadvantageous offer. compromise agreement must be interpreted as
Nary a howl of protest or shout of defiance spewed bestowing upon Dela Vida the power to demand a
forth from Rivieras lips, as it were, but a seemingly contract to sell from Macion. Where Macion promised to
whimper of acceptance when the counsel of Reyes execute a deed of absolute sale upon completing
strongly expressed in a letter dated December 5, 1989 payment of the price, it is a contract to sell. In the case
that Riviera had lost its right of first refusal. Riviera at bar, the sale is still in the executory stage since the
cannot now be heard that had it been informed of the passing of title is subject to a suspensive condition--that
offer of Five Thousand Three Hundred Pesos (P5,300.00) if Dela Vida is able to secure the needed funds to
of Cypress and Cornhill it would have matched said purchase the properties from Macion. A mere executory
price. Its stubborn approach in its negotiations with sale, one where the sellers merely promise to transfer
Reyes showed crystal-clear that there was never any the property at some future date, or where some
need to disclose such information and doing so would be conditions have to be fulfilled before the contract is
just a futile effort on the part of Reyes. Reyes was converted from an executory to an executed one, does
under no obligation to disclose the same. Pursuant to not pass ownership over the real estate being sold. It
Article 1339 of the New Civil Code, silence or cannot be denied that the compromise agreement,
concealment, by itself, does not constitute fraud, unless having been signed by both parties, is tantamount to a
there is a special duty to disclose certain facts, or unless bilateral promise to buy and sell a certain thing for a
according to good faith and the usages of commerce the price certain. Hence, this gives the contracting parties
communication should be made. The general rule is rights in personam, such that each has the right to
applicable in the case at bar since Riviera failed to demand from the other the fulfillment of their
convincingly show that either of the exceptions are respective undertakings. Demandability may be
relevant to the case at bar. exercised at any time after the execution of the Deed.

13. MACION v GUIANI 14. VILLONCO v BORMAHECO

FACTS: Macion and Dela Vida Institute entered into a FACTS: Cervantes and his wife owned 3 parcels of land
contract to sell, where the latter assured the former along Buendia where he buildings of Bormaheco Inc
that it will buy the 2 parcels of land in Cotabato City on were situated. Beside their property were lots owned by
or before July 31, 1991 at P1.75M. In the meantime, Villonco Realty. Cervantes entered into several

ANTONIO | HIPOLITO | IMPERIAL | ZARAGOSA 13


%
negotiations with Villonco for sale of the Buendia signatories
property. Cervantes made a written offer of P400/sqm
with a downpayment of P100,000 to serve as earnest HELD: NO. It is true that the signatures of the 5 siblings
money. The offer also made the consummation of the did not confer authority on Ernesto as agent to sell their
sale dependent upon the acquisition by Bormaheco of a respective shares in the properties, because such
Sta. Ana property. Villonco made a counter-offer stating authority to sell an immovable is required to be in
that the earnest money was to earn 10% interest p.a. writing. However, those signatures signify their act of
The check was enclosed with the reply letter. Cervantes directly (not through an agent) selling their personal
accepted and cashed the check. The Sta. Ana Property shares to Paraiso Dev. Corp.
was awarded to Bormaheco; the transfer was also duly In the case at bar, the Contract to Sell was perfected
approved. However, Cervantes sent the check back to when the petitioners consented to the sale to the
Villonco with the interest thereonstating that he was respondent of their shares in the subject parcels of land
no longer interested in selling the property. He also by affixing their signatures on the said contract. Such
claims that no contract was perfected; Villonco sues for signatures show their acceptance of what has been
specific performance. stipulated in the Contract to Sell and such acceptance
was made known to respondent corporation when the
ISSUE: W/N there was a perfected contract of sale duplicate copy of the Contract to Sell was returned to
the latter bearing petitioners signatures.
HELD: YES. There was a perfected contract of sale. The As to petitioner Enriquetas claim that she merely
alleged changes made in the counter-offer are signed as a witness to the said contract, the contract
immaterial and are mere clarifications. The changes of itself does not say so. There was no single indication in
the words Sta. Ana property to another property as the said contract that she signed the same merely as a
well as the insertion of the number 12 in the date, witness. The fact that her signature appears on the
and the words per annum in the interest are trivial. right-hand margin of the Contract to Sell is
There is no incompatibility in the offer and counter- insignificant. The contract indisputably referred to the
offer. Cervantes assented to the interest and he, in Heirs of Bibiano and Encarnacion Oesmer, and since
fact, paid the same. Also, earnest money constitutes there is no showing that Enriqueta signed the document
prood of the perfection of the contract of sale and in some other capacity, it can be safely assumed that
forms part of the consideration. The condition regarding she did so as one of the parties to the sale.
the acquisition of the Sta. Ana property was likewise In the instant case, the consideration of P100,000.00
fulfilled; there is thus no ground for the refusal of paid by respondent to petitioners was referred to as
Cervantes to consummate the sale. option money. However, a careful examination of the
words used in the contract indicates that the money is
not option money but earnest money. Earnest money
15. OESMER v PARAISO DEV CORP. and option money are not the same but distinguished
thus: (a) earnest money is part of the purchase price,
FACTS: Oesmers are co-owners of undivided shares of 2 while option money is the money given as a distinct
parcels of agricultural and tenanted land in Cavite, consideration for an option contract; (b) earnest money
which are unregistered and originally owned by their is given only where there is already a sale, while option
parents. When their parents died, they acquired the lots money applies to a sale not yet perfected; and, (c)
as heirs by right of succession. when earnest money is given, the buyer is bound to pay
In 1989, Paular, a resident and former Mun. Sec. of the balance, while when the would-be buyer gives
Carmona Cavite, brought Ernesto Oesmer (one of the option money, he is not required to buy, but may even
heirs) to meet with Lee, President of Paraiso forfeit it depending on the terms of the option.
Development Corp, in Manila for the purpose of
brokering the sale of Ernesto's properties to Paraiso Dev.
Corp. A contract to sell was entered into between 16. FULE v CA
Paraiso Dev. Corp and Ernesto as well as Enriqueta. A
check in the amount of P100,000 payable to Ernesto was FACTS: Fule, a banker and a jeweler, acquired a 10-
given as option money. Eventually, Rizalino, Leonora, hectare property in Rizal (Tanay Property), which used
Bibiano Jr, and Librado also signed the Contract to Sell. to be under the name of Fr. Antonio Jacobe, who
However, 2 of their brothers, Adolfo and Jesus, refused mortgaged it to Rural Bank of Alaminos to secure a loan
to sign the document. of P10,000. However, the mortgage was foreclosed.
A couple of months after, the Oesmers informed In 1984, Fule asked Dichoso and Mendoza to look for a
Paraiso (through a letter) that it is rescinding the buyer of the Tanay property. They found one in the
Contract to Sell and returning the option money. person of Cruz, who owns a pair of diamond earrings.
However, Paraiso did not respond and thus, Oesmers Fule was interested to buy these earrings, but Cruz
filed a complaint for declaration of nullity of the refused to sell them to him for the price he offered.
Contract to Sell with the RTC, which ruled in favor of Subsequently, negotiations for the barter between the
Paraiso Dev. Corp. On appeal, CA modified by declaring earrings and the property ensued. But it turned out that
that the Contract to Sell is valid and binding as to the the redemption period for the property has not yet
undivided shares of the six signatories of the document. expired. Thus, Fule executed a deed of redemption on
behalf of Fr. Jacobe in the amount of P16,000, and on
ISSUE: W/N the Contract to Sell is valid as to all even date, Fr. Jacobe sold the property to Fule for

ANTONIO | HIPOLITO | IMPERIAL | ZARAGOSA 14


%
P75,000. The Deed of Sale was notarized ahead of the 17. DAILON v CA
Deed of Redemption.
Subsequently, a Deed of Sale over the earrings was FACTS: Sabesaje sues to recover ownership of a parcel
executed and when it was delivered, Fule contends that of land based on a private document of absolute sale
the earrings were fake, even using a tester to prove executed by Dailon. Dailon denies the fact of the sale
such allegation. Thereafter, they decided to Dimayuga, alleging that the same being embodied in a private
a jeweler, to have the earrings tested. After a glance, instrument, the same cannot convey title under Art.
Dimayuga declared them fake. 1358 of the Civil Code which requires that contracts
Fule filed a complaint with the RTC against Cruz and which have for their object the creation, transmission,
her lawyer, Belarmino, praying that the contract of sale modification, or extinction of real rights over immovable
over the Tanay property be declared null and void on property must appear in a public instrument.
the ground of fraud and deceit. RTC ruled in favor of
Cruz and Belarmino. ISSUE: W/N there was a valid/perfected contract of sale

ISSUE: W/N the Deed of Sale over the Tanay Property is HELD: YES. The necessity of a public instrument is only
valid for conveniencenot for validity and enforceability.
Such is not a requirement for the validity of a contract
HELD: YES. It is evident from the facts of the case that of sale, which is perfected by mere consent. Dailon
there was a meeting of the minds between petitioner should thus be compelled to execute the corresponding
and Dr. Cruz. As such, they are bound by the contract deed of conveyance in a public instrument in favor of
unless there are reasons or circumstances that warrant Sabesaje. If the sale is made through a public
its nullification. The records, however, are bare of any instrument, it amounts to constructive delivery.
evidence manifesting that private respondents employed
such insidious words or machinations to entice
petitioner into entering the contract of barter. Neither 18. SECUYA v VDA DE SELMA
is there any evidence showing that Dr. Cruz induced
petitioner to sell his Tanay property or that she cajoled FACTS: Caballero owned certain friar lands. She entered
him to take the earrings in exchange for said property. into an Agreement of Partition where she parted with
On the contrary, Dr. Cruz did not initially accede to 1/3 of the said property in favor of Sabellona. Sabellona
petitioner's proposal to buy the said jewelry. Rather, it took possession thereof and sold a portion to Dalmacio
appears that it was petitioner, through his agents, who Secuya through a private instrument that is already lost.
led Dr. Cruz to believe that the Tanay property was Secuya, along with his many relatives took possession of
worth exchanging for her jewelry as he represented that the said land. Later on, Selma bought a portion of the
its value was P400,000.00 or more than double that of said land, including that occupied by Secuya; she bought
the jewelry which was valued only at P160,000.00. If it from Caesaria Caballero. She presented a Deed of
indeed petitioner's property was truly worth that much, Absolute Sale and a TCT. Secuya filed a case for quieting
it was certainly contrary to the nature of a businessman- of title. CA upheld Selmas title considering that she had
banker like him to have parted with his real estate for a TCT and a Deed of Sale.
half its price. In short, it was in fact petitioner who
resorted to machinations to convince Dr. Cruz to ISSUE: Who has a better right, Secuya or Selma?
exchange her jewelry for the Tanay property.
Furthermore, petitioner was afforded the reasonable HELD: The Secuyas have nothing to support their
opportunity required in Article 1584 of the Civil Code supposed ownership over the parcel of land. The best
within which to examine the jewelry as he in fact evidence they could have had was the private
accepted them when asked by Dr. Cruz if he was instrument indicating the sale to their predecessor-in-
satisfied with the same. By taking the jewelry outside interest. But the instrument is lost. Even so, it is only
the bank, petitioner executed an act which was more binding as between the parties and cannot prejudice 3rd
consistent with his exercise of ownership over it. This persons since it is not embodied in the public document.
gains credence when it is borne in mind that he himself Selma, on the other hand, has all the supporting
had earlier delivered the Tanay property to Dr. Cruz by documents necessary; she also acted in good faith and
affixing his signature to the contract of sale. That after thought that the Secuyas were merely tenants. They did
two hours he later claimed that the jewelry was not the not even pay realty taxes and did not have their claim
one he intended in exchange for his Tanay property, annotated to the certificate of sale.
could not sever the juridical tie that now bound him and
Dr. Cruz. The nature and value of the thing he had taken
preclude its return after that supervening period within
19. YUVIENGCO v DACUYCUY
which anything could have happened, not excluding the
alteration of the jewelry or its being switched with an
FACTS: Yuvienco entered into a contract with Yao King
inferior kind.
Ong and the other occupants, wherein the former will
sell to the latter the Sotto property in Tacloban City for
P6.5M provided that the latter made known their
decision to buy it or not later than July 31, 1978. When
Yuvienco's representative went to Cebu with a prepared
and duly signed contract for the purpose of perfecting
ANTONIO | HIPOLITO | IMPERIAL | ZARAGOSA 15
%
and consummating the transaction, Yao King Ong and asked if it was possible to pay on terms. The bank
other occupants found variance between the terms of officials stated that there was no harm in trying to ask
payment stipulated in the document and what they had for payment on terms because in previous transactions,
in mind. Thus, it was returned unsigned. Thus, the the same had been allowed. It was the understanding,
action for specific performance. however, that should the term payment be disapproved,
then the price shall be paid in cash. It was Albano who
ISSUE: W/N the claim for specific performance of Yao dictated the terms under which the installment payment
King Ong is enforceable under the Statute of Frauds may be approved, and acting thereon, Alfonso Lim
wrote BPI through Merlin Albano embodying the
HELD: YES. It is nowhere alleged in the complaint that payment initially of 10% and the remaining 90% within a
there is any writing or memorandum, much less a duly period of 90 days. 2 or 3 days later, LSM learned that its
signed agreement to the effect, that the price of offer to pay on terms had been frozen. Alfonso Lim went
P6,500,000 fixed by petitioners for the real property to BPI and tendered the full payment of P33,056,000.00
herein involved was agreed to be paid not in cash but in to Albano. The payment was refused because Albano
installments as alleged by Yao King Ong. The only stated that the authority to sell that particular piece of
documented indication of the non-wholly-cash payment property in Pasig had been withdrawn from his unit. The
extant in the record is the deeds already signed by same check was tendered to BPI Vice-President Nelson
Yuvienco and taken to Tacloban by Atty. Gamboa for the Bona who also refused to receive payment.
signatures of the respondents. In other words, the 90- LSM filed an action for specific performance with
day term for the balance of P4.5 M insisted upon by damages against BPI. In the course of the trial, BPI
respondents choices not appear in any note, writing or informed the trial court that it had sold the property
memorandum signed by either the petitioners or any of under litigation to National Book Store (NBS) in 1989.
them, not even by Atty. Gamboa. Hence, looking at the The complaint was thus amended to include NBS. RTC
pose of respondents that there was a perfected ruled in favor of LSM, holding that there was a perfected
agreement of purchase and sale between them and contract of sale between LSM and BPI. CA reversed,
petitioners under which they would pay in installments holding that no contract of sale was perfected because
of P2 M down and P4.5 M within ninety 90) days there was no concurrence of the three requisites
afterwards it is evident that such oral contract involving enumerated in Article 1318 of the Civil Code.
the "sale of real property" comes squarely under the
Statute of Frauds (Article 1403, No. 2(e), Civil Code.) ISSUE: W/N there was a valid contract of sale
In any sale of real property on installments, the
Statute of Frauds read together with the perfection HELD: YES. There was a meeting of the minds between
requirements of Article 1475 of the Civil Code must be the buyer and the bank in respect to the price of
understood and applied in the sense that the idea of P1,000/sqm. The requirements in the payment of the
payment on installments must be in the requisite of a purchase price on terms instead of cash were suggested
note or memorandum therein contemplated. While such by BPI Vice-President Albano. Since the authority given
note or memorandum need not be in one single to broker Revilla specified cash payment, the possibility
document or writing and it can be in just sufficiently of paying on terms was referred to the Trust Committee
implicit tenor, imperatively the separate notes must, but with the mutual agreement that if the proposed
when put together', contain all the requisites of a payment on terms will not be approved by our Trust
perfected contract of sale. To put it the other way, Committee, Limketkai should pay in cash, the amount
under the Statute of Frauds, the contents of the note or was no longer subject to the approval or disapproval of
memorandum, whether in one writing or in separate the Committee, it is only on the terms. The record
ones merely indicative for an adequate understanding of shows that if payment was in cash, either broker Revilla
all the essential elements of the entire agreement, may or Aromin had full authority. But because LSM took
be said to be the contract itself, except as to the form. advantage of the suggestion of Vice-President Albano,
the matter was sent to higher officials. Immediately
upon learning that payment on terms was frozen and/or
20. LIMKETKAI SONS MILLING INC v CA denied, Limketkai exercised his right within the period
given to him and tendered payment in full, thus
FACTS: In 1976, Philippine Remnants Co., Inc. complying with their agreement.
constituted the Bank of the Philippine Islands (BPI) as its The negotiation or preparation stage started with the
trustee to manage, administer, and sell its real estate authority given by Philippine Remnants to BPI to sell the
property, one of which was the disputed lot in Pasig. In lot, followed by the authority given by BPI and
1988, Pedro Revilla, Jr., a licensed real estate broker, confirmed by Philippine Remnants to broker Revilla to
was given formal authority by BPI to sell the lot for sell the property, the offer to sell to Limketkai, the
P1,000/sqm. Broker Revilla contacted Alfonso Lim of inspection of the property and the negotiations with
Limketkai Sons Milling (LSM) who agreed to buy the land. Aromin and Albano at the BPI offices. The perfection of
LSM asked that the price of P1,000/sqm. be reduced to the contract took place when Aromin and Albano, acting
P900.00 while Albano stated the price is to be for BPI, agreed to sell and Alfonso Lim with Albino
P1,100.00. The parties finally agreed that the lot would Limketkai, acting for LSM, agreed to buy the disputed
be sold at P1,000/sqm. to be paid in cash. lot at P1,000/sqm. Aside from this there was the earlier
Notwithstanding the final agreement to pay agreement between LSM and the authorized broker.
P1,000/sqm. on a cash basis, Alfonso Lim (LSM official)

ANTONIO | HIPOLITO | IMPERIAL | ZARAGOSA 16


%
There was a concurrence of offer and acceptance, on partial performance, which takes the verbal agreement
the object, and on the cause thereof. out of the operation of the Statute of Frauds.

21. ORTEGA v LEONARDO 22. CLAUDEL v CA

FACTS: Ortega occupied a parcel of land. After the FACTS: Cecilio Claudel acquired a lot from the Bureau of
liberation, the government assigned the lot to the Rural Lands. He occupied the same, declared it in his name
Progress Admin. She asserted her right thereto; but was and dutifully paid his taxes. After his death, his heirs
disputed by Leonardo. Ortega and Leonardo agreed to a and siblings contested each other claiming ownership
compromise. The agreement was for Ortega to desist thereof. It was his heirs who were in possession of the
from pressing her claim, and Leonardo, upon getting the property. They partitioned it amongst themselves,
lot, would sell to her a portion thereof provided she registered each portion under the Torrens System, and
paid for the surveying of the lot. If he acquired title, she each paid their respective taxes. The siblings filed a
could stay as tenant. Ortega thus desisted from her case for cancellation of titles and reconveyance arguing
claim, paid for the surveying of the lot and the that there was a verbal sale between Cecilio and their
preparation of the plan, and regularly paid him a parents over the lot. As evidence, they presented a
monthly rental. When she remodeled her sons house subdivision plan. CA ordered the cancellation of the
beside the lot, it extended over the subject lot. When TCTs in favor of the heirs.
Leonardo acquired title, he refused to sell the portion
agreed upon. He claims that the contract is ISSUE: W/N there was a valid sale between Cecilio and
unenforceable based on the Statute of Frauds. his siblings

ISSUE: W/N the contract is unenforceable HELD: NO. As a rule, a sale of land is valid regardless of
the form it may have been entered into. However, in
HELD: NO. The contract is enforceable because there the event that a 3rd party disputes the ownership, there
was partial performance. Ortega made substantial is no such proof in support of the ownership. As such, it
improvements on the lot, desisted from her claim, cannot prejudice 3rd personssuch as the heirs in this
continued possession, and paid for the surveying, and case. Also, the heirs had a right to rely upon their
also paid the rentals. All these put together amount to Torrens titles, which, as opposed to the subdivision
plans, are definitely more credible.
Further, the subsequent buyers were in bad faith
because Armando & Adelia registered their adverse
23. ALFREDO v BORRAS claimthis amounts to constructive notice, which
negates good faith.
FACTS: Godofredo & Carmen mortgaged their land to The Statute of Frauds likewise does not apply
DBP for P7,000. To pay their debt, they sold the land to considering that Godofredo & Carmen had already
Armando & Adelia for P15,000. The latter also assumed derived the benefits from the salesuch as the money to
to pay the loan. Carmen issued Armando & Adelia a pay for the loan. The receipt also suffices to constitute
receipt for the sale. They also delivered to Armando & the memorandum required by the Statute of Frauds.
Adelia the Original Certificate of Title, tax declarations, Assuming that the sale was voidable because it was
and tax receipts. They also introduced Armando & conjugal property, the same was ratified by Godofredo
Adelia to the Natanawans, the tenants of the said by introducing Armando & Adelia to the Natanawans as
property as the new lessors. They thereafter took the new lessors. Also, even though titled as Specific
possession of the said land. Later, they found out that Performance, the complaint was one for reconveyance
Godofredo & Carmen sold the land again to other buyers and prescription does not lie of one who is in actual
by securing duplicate copies of the OCTs upon petition possession of the property.
with the court. Thus, they filed for specific
performance. Godofredo & Carmen claimed that the
sale, not being in writing, is unenforceable under the 23. TOYOTA SHAW INC v CA
Statute of Frauds.
FACTS: Luna Sosa wanted to buy a Toyota Lite Ace. He
ISSUE: W/N the contract of sale is unenforceable under went to Toyota Shaw where he met Popong Bernardo, a
the Statute of Frauds. sales rep. Sosa explained that he needed the Lite Ace by
June 17, otherwise, he would become a laughing stock.
HELD: NO. The Statute of Frauds is applicable only to Bernardo guaranteed that the vehicle would be
executory contracts, not those that have already been delivered. They executed a document entitled
partially or completely consummated. In this case, the Agreements between Sosa & Popong Bernardo of Toyota
sale of the land to Armando & Adelia had already been Shaw where a P100K downpayment was stipulated and
consummated. The ownership of the land was also that the Lite Ace would be available at a given date.
transferred to Armando & Adelia when they were When the day of reckoning arrived, the Lite Ace was
introduced to the Natanawans and took possession unavailablethe explanation of Bernardo being nasulot
thereof. Therefore, when Godofredo & Carmen sold the ng ibang malakas. However, according to Toyota, the
land to other buyers, it was no longer theirs to sell. true reason was that BA Finance, which was supposed to
ANTONIO | HIPOLITO | IMPERIAL | ZARAGOSA 17
%
answer for the balance of the purchase price, did not essential to the perfection of the sale. It was also clear
approve Sosas application. Toyota also returned the that Bernardo signed the document in his personal
downpayment. Thus, Sosa sued for damages amounting capacity and it was up to Sosa to inquire as to the
to P1.2M due to his humiliation, hurt feelings, sleepless extent of the formers capacity. Sosa did not even sign
nights, and so on. it. It was nothing but a mere proposal, which did not
mature into a perfected contract of sale in lieu of the
ISSUE: W/N there was a perfected contract of sale subsequent events. In fact, it made no specific
reference to the sale of a vehicle. No obligations could
HELD: NO. Toyota Shaw should NOT be held liable for thus arise therefrom. Sosa has no one else to blame but
damages because there was no perfected contract of himself for his humiliation for bragging about something
sale in the first place. There was no agreement as to the he does not own yet.%
price and the manner of paymentwhich are both

CONSUMMATION/PERFECTION OF CONTRACT
1. SANTOS v SANTOS ISSUE: W/N Wilfredo, as mortgagor, can sell the tractor
subject of a mortgage
FACTS: Jesus and Rosalia owned a lot with a 4-door
apartment. They sold through a public instrument the HELD: YES. The mortgagor (Wilfredo) had every right to
said property to their children, Salvador and Rosawho sell the property subject to mortgageeven without the
sold her share to Salvador as well. Nonetheless, in spite consent of the mortgagee as long as the purchaser
of the sale, Rosalia remained in possession and control assumes the liability of the mortgagor.
over the property. Jesus, Rosalia and Salvador died. In this case, there was constructive delivery already
Zenaida, claiming to be Salvadors heir, demanded rent upon the execution of the public instrumenteven if the
from the tenants. The other children of Jesus and tractor could not yet be delivered. Execution of the
Rosalia filed a case for reconveyance averring that the public instrument and mutual consent of the parties was
sale to Salvador was fictitious and done merely to equivalent to constructive delivery. Therefore, at the
accommodate him. time when the sheriff levied upon the tractor, it was no
longer the property of Wilfredo. Also the clearing of the
ISSUE: W/N the sale to Salvador was fictitious check was not a condition for the consummation of the
sale but only upon the extinguishment of the mortgage.
HELD: YES. While it is true that sale through a public
instrument is equivalent to delivery of the things sold
which has the effect of transferring ownership, the 3. ADDISON v FELIX
delivery can be rebutted by clear and convincing
evidence. The vendors continuous possession makes the FACTS: Addison owned 4 parcels of land, which he sold
sale dubious. Salvador never took possession of the to Felix, through public instrument. The down payment
property. He surrendered the titles to his mother after was made; the final installment to be paid after the
having registered the lots in his name, he never issuance of the certificate of title. Addison sued Felix to
collected rentals, neither has he paid the taxes thereon. compel the latter to pay the last installmentbut Felix
Thus, there was no real transfer of ownership. That refused and sought to rescind the contract due to the
being the case, the action for reconveyance was absolute failure of Addison to deliver the thing sold.
imprescriptible.
ISSUE: W/N there was delivery

2. DY JR v CA HELD: NO. While it is true that execution of a public


instrument is tantamount to delivery of the thing sold,
FACTS: Perfecto and Wilfredo Dy are brothers. Wilfredo in order for such symbolic delivery to have the effect of
purchased a truck and a tractor, both of which were tradition, the vendor should have had control over the
mortgaged to Libra Financing as security for a loan. thing and at the moment of the sale, its delivery could
Perfecto wanted to purchase the tractor, he convinced have been made. In this case, the ownership was
his sister to purchase the truck. Perfecto executed a disputed by the Villafuertes, who were in possession of
public document to evidence the sale. Libra acceded to the land. Addison even failed to show the land to Felix
the sale and agreed that upon the issuance and due to the hostile opposition; he also failed to have it
encashment of the check that they issued for the surveyed. The legal fiction of delivery thus yields to
purpose, the chattels can be released. However, in a realityno delivery was ever made. Felix had every right
case against Wilfredo filed by Gelac Trading, the sheriff therefore to rescind the contract. Had there been an
seized the tractor on levy and sold the same on public agreement that Felix would have to undertake to evict
auction, with Gelac as the highest bidder. Perfecto thus the Villafuertes, the result may have been different, but
sought to recover the truck from Gelac. there is no such agreement.

ANTONIO | HIPOLITO | IMPERIAL | ZARAGOSA 18


%
4. DANGUILAN v IAC 6. POWER COMMERCIAL AND INDUSTRIAL CORP. v CA

FACTS: Domingo owned 2 lots, which he donated FACTS: Power Commercial Corp. entered into a contract
through a private instrument to Danguilan for the of sale with the Quiambao spouses. It agreed to assume
consideration that the latter must take care of him for the mortgages thereon. A Deed of Absolute Sale with
the remainder of his life and manage his burial. Assumption of Mortgage was executed. Power
Domingos daughter, Apolonia, laid claim to the land, Commercial failed to settle the mortgage debt
presenting a public document allegedly executed in her contracted by the spouses, thus it could not undertake
favor, the purchase price being paid for by her mother. the proper action to evict the lessees on the lot. Power
She however failed to take possession of the said Commercial thereafter sought to rescind the contract of
property after the execution of the deed. In fact, she the sale alleging that it failed to take actual and
moved out of the farm when Danguilan started to physical possession of the lotwhich allegedly negated
cultivate the same for as long as she was given a share constructive delivery.
from the harvests. She decided to file a case only after
the deliveries of farm produce have ceased. ISSUE: W/N there was delivery

ISSUE: Who has a better title over the land, Danguilan or HELD: YES. First, such a condition that the Quiambao
Apolonia? spouses would evict the lessees therein was not
stipulated in the contract. In fact, Power Commercial
HELD: DANGUILAN. At the onset, the donation in favor was well aware of the presence of the tenants therein.
of Danguilan was valid even though embodied in a Also in this case, Power Commercial was given control
private instrument, because it was an onerous donation. over the said lot and it endeavored to terminate the
The deed of sale presented by Apolonia was also occupation of the actual tenants.
suspicious. It was only 3 years old and the consideration Control cannot be equated with actual possession.
was paid for by her mother. Assuming that it was valid, Power Commercial, as purchaser, agreed voluntarily to
still the presumptive delivery is overcome by the fact assume the risks involved. The public instrument
that she failed to take possession of the property. executed amounted to symbolic delivery of the property
Ownership, after all, is not transferred by mere sold and authorized the buyer to use the document as
stipulation butby actual and adverse possession. She proof of ownership. Power Commercial was deprived of
even transferred the same to Danguilan possession of ownership only after it failed to remit the
the same. She cannot have a better right in this case amortizations, but not due to failure of delivery.
than Danguilan.

7. CHUA v CA
5. PASAGUI v VILLABLANCA
FACTS: Valdes-Choy is the owner of the subject matter,
FACTS: Pasagui purchased a parcel of land form the when she advertised the property for sale. Chua
Bocar Spouses for P2,800, which was embodied in a responded to the advertisement, and met up with
public instrument. They failed to take possession of the Valdes-Choy. They agreed for the purchase price of
property because the Villablancas illegally took P10,800,000, to be paid on July 15, 1989. This was
possession of the property and harvested the coconuts evidenced by an earnest money for P100,000, which was
therein. Thus, Pasagui filed a case for ejectment before put on a receipt, stating that the money will be
the CFI. The Bocar spouses were likewise impleaded. forfeited upon failure to pay on the dat stipulated. On
The latter contested that the case should be dismissed July 13, Valdes-Choy executed two deeds of absolute
because the CFI did not have jurisdiction over forcible sale, first, pertaining to the house and lot, valued at
entry cases. P8,000,000, and second, pertaining to the movable
properties therein. The next day, Chua issued a check
ISSUE: W/N this is a case of forcible entry worth P485,000 for the purpose paying the capital gains
tax. The value was deducted from the balance, with an
HELD: NO. The case was not for forcible entry because outstanding value of P10,295,000 (additional P80,000 for
there was no allegation that Pasagui was in prior the documentary stamp tax). Chua also showed a check
physical possession of the land and that the worth P10,215,00 to Valdes-Choy, however, he
Villablancas, through force, stealth, or threat, deprived demanded that the TCT should first be transferred to his
them thereof. While the sale was made through a public name before paying the check. Out of anger, Valdes-
document is equivalent to delivery, this presumption Choy tore the deed of absolute sale. On the reckoning
only holds true if there is no impediment to the date, Valdes-Choy tried to make a compromise with
possession of the purchaser. Such is not the case here. Chua, but she did not get any response. Two days later,
Since Pasagui had not yet acquired physical possession Chua filed an action for specific performance, which the
of the land, the case was not one for forcible entry and trial court dismissed. A week later, he filed another
the CFI (not municipal courts) has jurisdiction. action for specific performance, where the court ruled
in favor of him. On appeal, CA reversed.

ANTONIO | HIPOLITO | IMPERIAL | ZARAGOSA 19


%
ISSUE:
ISSUE: W/N VELI is obliged to pay for the expenses to
1. Whether the agreement was a contract of sale transfer the title of the property to Genuino Ice
or contract to sell
2. Whether registration is needed to transfer HELD: YES. Under Art. 1487 of the CC, the expenses for
ownership the registration of the sale should be shouldered by the
vendor (VELI) unless there is a stipulation to the
RULING: It is a contract to sell. First, when the contrary. In the absence of the stipulation of the parties
agreement was made, the earnest money is forfeited in relating to the expenses for the registration of the sale
favor of Valdes-Choy who may then sell the land to and the transfer of the title to the vendee (Genuino
other interested parties. This is the nature of reserving Ice), Art. 1487 shall be applied in a supplementary
the ownership of the property, subject to the full manner.
payment of the purchase price. Second, absent of a Under Art. 1495 of the CC, VELI, as vendor, is obliged
formal deed of conveyance of the property in favor of to transfer title over the property and deliver the same
the buyer shows that there was no intention to transfer to the vendee (Genuino Ice). While Art. 1498 of the CC
ownership immediately. The non-fulfillment of the provides that the execution of a notarized deed of
suspensive condition, which is payment of the full absolute sale shall be equivalent to the delivery of the
purchase price prevents the obligation to sell from property, the same shall not apply if from the deed the
arising, where the owner retains the ownership over the contrary does not appear or cannot clearly be inferred.
property. Art 1482 speaks of earnest money as an In this case, Genuino Ice and VELI agreed that the latter
evidence of a perfected contract of sale. However, in would cause the eviction of the tenants and deliver
this case, the earnest money was paid in part possession of the property. It is clear that at the time
consideration of a contract to sell, and therefore, art the petitioner executed the deed of sale in favor of
1482 does not apply. Genuino Ice, there were tenants in the property. It
Delivery is effected upon execution of the sale in a cannot be concluded that the property was thereby
public instrument. However, registration is not needed delivered to Genuino Ice.
in order to complete the deed of sale. Delivery is what
transfers ownership, and not registration in the Registry
of Property. Registration is only necessary to bind third 9. BHEN MEYER & CO. v YANGCO
persons; it is not a mode of acquiring ownership.
FACTS: Yangco ordered 80 drums of caustic soda
Carabao Brand from Bhen & Meyer. The instrument
8. VIVE EAGLE LAND INC v CA evidencing the agreement made use of the terms FOB
and CIF. The goods were detained by the British
FACTS: In 1987, Spouses Flores, as owners, sold 2 authorities in Penang. Bhen & Meyer alleges that Yangco
parcels of land in Cubao to Tatic Square International had already acquired ownership of the said goods and
Corp for P5.7M. Tatic applied for a loan with Capital should thus pay for the purchase price. However,
Rual Bank of Makati to finance its purchase of the said Yangco refused to accept the same alleging that the
lots, which the bank granted provided that the torrens goods were not Carabao Brand and that the same
title over the lots would be registered under its name as were adulterated.
collateral for the payment of the loan.
In 1988, Tatic sold these parcels of land to Vive Eagle ISSUE: W/N ownership is transferred/delivery is effected
Land Inc (VELI) for P6.3M, although the torrens titles with FOB and CIF from seller to buyer
over the lots were still in the custody of the bank.
During the same year, VELI sold one of these parcels of HELD: NO. The terms FOB and CIF mean that the costs
land to Genuino Ice Co. Inc. for P4M. Also, a deed of of delivery are for the seller. This means that it is the
assignment of rights in which VELI assigned in favor of sellers duty to make sure that the goods are duly
Genuino Ice all rights and interests under the Deed of delivered. Until then, ownership of the goods had not
Sale executed by spouses Flores and the other Deed of yet passed. Had the expenses been for the buyer, the
Sale executed by Tatic in VELI's favor, in so far as that goods are deemed delivered upon delivery to the
lot is concerned. common carrier. In this case, the delivery has not been
effected to the buyer, thus, the latter had every right to
Flores!Tatic (2 lots)!VELI (2 lots)!Genuino Ice (1 lot) rescind the contract of sale.

Genuino Ice demanded that VELI pay its capital gains


tax amounting to P285,000. However, VELI refused 10. GENERAL FOODS v NACOCO
saying that the Spouses Flores and Tobias (broker of the
sale) are responsible to pay the tax. Genuino Ice filed an FACTS:
action for specific performance against VELI, contending General Foods is a foreign corporation licensed to do
that VELI failed to transfer title to and in the name of business in the Philippines.
Genuino Ice, to cause the eviction of the occupants, and National Coconut Corporation (NACOCO) sold to
to pay the tax and other dues to effectuate the transfer General Foods 1500 tons of long copra under the terms:
of the title of the property. RTC ruled in favor of a. Quantity: Seller could deliver 5% more or less than
Genuino Ice, CA affirmed. the contracted quantity, and the
ANTONIO | HIPOLITO | IMPERIAL | ZARAGOSA 20
%
surplus/deficiency shall be paid on the basis of contracted quantity, and the surplus/deficiency
the delivered weight. shall be paid on the basis of the delivered
b. Price: CIF New York. weight.
c. Payment: Buyers to open an Irrevocable Letter of While the risk of loss was apparently placed on
Credit for 95% of invoice value based on shipping General Foods after the delivery of the cargo to the
weight. carrier, it was agreed that the payment of the price was
d. Balance of the price was to be ascertained on the to be according to the net landed weight which is 898
basis of outturn weights and quality of the cargo (weighed in New York) and not 1054 (weighed in the
at the port of discharge. Philippines).
e. Weights: Net landed weights. NACOCO had the burden to prove that the shortage
In the Philippines, the net cargo was weighed at 1054 was due to risks of voyage and not the natural drying up
tons, the alleged weight delivered by NACOCO. NACOCO of copra. In other words, if the weight deficiency was
then withdrew 95% (or $136,000) of the amount in the due to the risks of the voyage, General Food would not
Letter of Credit in favor of NACOCO. have been entitled to any claim in the deficiency.
In New York, the net cargo was reweighed and found The provision on the balance of the price was to be
to weigh only 898 short tons. General Foods demanded ascertained on the basis of outturn weights and quality
the refund of the amount of $24000. of the cargo at the port of discharge should not be
NACOCOs officers-in-charge acknowledged in a letter construed separately from the provision that the net
liability the deficiency and promised payment as soon as landed weight was to control.
funds were available. The manifest intention of the parties was for the total
However, NACOCO was abolished and went into price to be finally ascertained only upon determining
liquidation. The Board of Liquidators refused to pay the the net weight and quality of the goods upon arrival in
claim of General Foods. New York, most likely because the nature of copra is
General Foods then filed to recover $24,000 and 17% that it dries up and diminishes weight during the
exchange tax plus attorneys fees and costs. voyage.
General Foods alleges that although the sale quoted In fact, this intention was shown by the letter of the
CIF New York, the agreement contemplated the officer-in-charge of NACOCO acknowledging NACOCOs
payment of the price according to the weight and liability to General Foods. Though this letter of
quality of the cargo upon arrival in New York (port of acknowledgement should not be construed as an
destination). Therefore, the risk of shipment was upon admission of liability of NACOCO, it is nevertheless
the seller. competent evidence of NACOCOs intention to be bound
NACOCO alleges that the contract is an ordinary CIF, by the net landed weight or outturn weight of the copra
which means that delivery to the carrier is delivery to at the port of discharge.
the buyer. Therefore, the shipment having been
delivered to the buyer and the buyer having paid the
price, the sale was consummated. 11. PACIFIC VEGETABLE OIL CORP v SINGZON

ISSUES: FACTS:
1. Whether the weight in New York should be the basis Petitioner and respondent entered into a contract in
upon payment of the price of copra should be made. the US whereby Singzon agreed to ship 500 tons of
Yes. The weight in New York should be the basis. copra, with the agreement CIF, Pacific Coast
2. Whether what is to be ascertained based upon the Singzon failed to deliver, but the parties entered into
outturn weights and quality at port of discharge was a settlement, whereby Singzon would deliver 300 tons at
only the balance due to be paid. No. The balance due the same terms the contract provided that should
to be paid is not the only basis. Singzon again default, he would pay $10,000 for
damages and the original contract would be revived
HELD: Singzon again failed to ship the copra, and he did not
Under an ordinary CIF agreement, delivery to the pay the fine or ship the 500 tons as originally agreed
buyer is complete upon delivery of the goods to the Pacific filed an action to recover damages
carrier and tender of the shipping and other documents Singzon claims that Pacific had no legal personality to
required by the contract and the insurance policy are sue because it is a foreign corporation
taken in the buyers behalf. However, the parties may,
by express stipulation, modify a CIF contract and throw HELD:
the risk upon the seller until the arrival in the port of The contract was perfected in the US by a broker and
destinations. representatives of the parties payment was made to a
In this case, the terms of the contract indicate and bank in California and delivery undertaken through CIF,
intention that the precise amount to be paid by the Pacific Coast
buyer depended upon the ascertainment of the exact Under that arrangement, the vendor is to pay not only
net weight of the cargo at the point of destination: the cost of goods, but also the freight and insurance
a. Net landed weights were to govern. expenses, and this is taken to indicate that the delivery
b. The balance of the price was to be ascertained on is to be made at the port of destination
the basis of outturn weights and quality of the Since CIF includes both insurance and freight expenses
cargo at the port of discharge. to be paid by the seller, ordinarily, before the vessel
c. The seller could deliver 5% more or less than the arrives at the point of destination the risk of loss be for

ANTONIO | HIPOLITO | IMPERIAL | ZARAGOSA 21


%
the account of the seller. and creates lien upon the land. The spouses acquired
their titles under the Torrens System and they acted in
good faith by exercising due diligence; thus, they have a
12. RUDOLF LIETZ INC v CA better right to the said property.

FACTS: Buriol previously owned a parcel of unregistered


land in Palawan. In 1986, he entered into a lease 14. NAVAL v CA
agreement with Flaviano and Tiziana Turatello and Sani
(Italians) involving a hectare of his property. This FACTS: In 1969, Ildefonso Naval sold a parcel of land to
agreement was for a period of 25 years, renewable for Gregorio; the sale was recorded under Act 3344. Also in
another 25 years. After the paying P10,000 1969, Gregorio sold portions thereof to Balilla, Camalla
downpayment, Turatello and Sani took possession of the and the Moya Spouses, who thereafter took possession of
land. However, this agreement was only reduced into their respective portions. Juanita, a great
writing in 1987. granddaughter of Ildefonso, surfaced and claimed that
After 11 months, Buriol sold the same parcel of land (5 the land was sold to her by the latter in the year 1972;
hec) to Rudolf Lietz Inc for P30,000. Later on, Rudolf she also presented an OCT as evidence. It must be noted
Lietz Inc discovered that Buriol owned only 4 hectares that the property was not yet registered under the
with one hectare covered by the lease; thus, only 3 Torrens System when it was sold to Juanita and
hectares were delivered to it. Rudolf Lietz Inc instituted Gregorio.
a complaint for the annulment of the lease against
Buriol, Sani and the Turatellos before the RTC. RTC and ISSUE: W/N Juanita has a better title (since it is
CA ruled in favor of Buriol, Sani and Turatellos. registered) than Balilla, Camalla and Moya spouses

ISSUE: Whether the sale between Buriol and Rudolf Lietz HELD: NO. Art. 1544 is not applicable because the land
Inc is a lump sum or unit price sale was unregistered under the Torrens System at the time
of the 1st sale. The applicable law is Act 3344. Under
HELD: LUMP SUM SALE. The Deed of Absolute Sale shows said law, registration by the 1st buyer is constructive
that the parties agreed on the purchase price on a notice to the 2nd buyerand as such, the latter cannot
predetermined area of 5 hectares within the specified be deemed to be in good faith. Applying the principle of
boundaries and not based on a particular rate per area. priority in time, priority in rights, Juanita cannot claim
In accordance with Art. 1542, there shall be no to have a better right. The fact that Juanita was able to
reduction in the purchase price even if the area secure a title in her name does not operate to vest
delivered to Rudolf Lietz Inc is less than that states in ownership. The Torrens System cannot be used as a
the contract. In the instant case, the area within the means to protect usurpers.
boundaries as stated in the contract shall control over
the area agreed upon in the contract.
15. CARILLO v CA

13. NAAWAN COMMUNITY RURAL BANK INC v CA FACTS: Gonzales purchased from Priscilla, acting as
agent of Aristotle, the latters land. For failure to
FACTS: Comayas offered to sell to the Lumo Spouses a execute the Deed of Sale, she filed a case for specific
house and lot. The property was already registered performance and impleaded Priscilla (not Aristotle). The
under the Torrens System that time and they made latter defaulted and judgment was rendered against her
appropriate inquiries with the RD; they found out that it ordering the nullification of the OCT of Aristotle and the
was mortgaged for P8,000, paid Comayas to settle the issuance of a new certificate of title in favor of
mortgage, and the release of the adverse claim was Gonzales. The Dabons thereafter surfaced and sought to
annotated in the title. Thereafter, they executed an annul the judgment of the trial court averring that they
Absolute Deed of Sale over the subject property and purchased the property from Aristotle himself and they
registered the same. However, it turns out that it was were not impleaded as the real parties in interest.
already previously sold to Naawan Community Rural
Bank; it was then unregistered. The Bank foreclosed on ISSUE: Who has better title, Gonzales or Dabon?
the property, purchased the same, and registered it
under Act 3344. Thus, the Bank sought to eject the HELD: DABON. The decision of the lower court in favor
spouses. However, the latter countered with an action of Gonzales was void due to extrinsic fraud. The Dabons
for quieting of title. were deprived of their day in court and through
questionable means at thatsuch as the failure to give
ISSUE: Who has a better title, Naawan or Lumo spouses? them appropriate notice of the proceedings, and not
having them impleaded even though they are the parties
HELD: LUMO SPOUSES. Where a person claims to have to be adversely affected. Instead, it was the agent who
superior property rights by virtue of a sheriffs sale, the was impleadednot the principal or the subsequent
benefit of Art. 1544 applies favorably only if the purchasers. The court never acquired jurisdiction.
property is registered under the Torrens Systemnot It must be noted that the property was sold to
under Act 3344. Registration under the Torrens System Gonzales in 1988, while the same was sold to the Dabons
is the operative act that gives validity to the transfer in 1989; nonetheless, the requirements of double-sale
ANTONIO | HIPOLITO | IMPERIAL | ZARAGOSA 22
%
are two-fold: acquisition in good faith and registration
in good faith. Based on the foregoing, the case is
remanded to the lower court for further proceeding. 18. MENDOZA v KALAW

FACTS: In 1919, Federico Canet sold to Kalaw a parcel of


16. CARBONELL v CA land under a Conditional Sale. 2 months after, Canet
sold to Mendoza the same parcel of land under an
FACTS: Poncio, a Batanes native, owned a parcel of Absolute Sale. Mendoza took possession thereof, cleaned
land, which he offered to sell to Carbonell and Infante. and fenced it, and sought to have the same registered
The land was mortgaged to Republic Bank. Poncio and but Kalaw opposed. When Kalaw first tried to register
Carbonell agreed to the sale of the land, and the latter the same, he was denied but an anotacion preventiva
assumed to pay the mortgage in favor of the bank. was annotated in the title.
Poncio and Carbonell executed an instrument where the
latter allowed the former to remain in the premises in ISSUE: Who has a better title, Canet or Kalaw?
spite of the sale for a period of 1 year. Later on, when
the Formal Deed of Sale was to be executed, Poncio told HELD: CANET. While a conditional sale came before the
Carbonell that he could no longer proceed with the sale absolute sale, still the latter must prevail. A conditional
as he had already sold the same to Infante for a better sale, before the happening of the condition, is hardly a
price. Carbonell immediately sought to register adverse sale especially if the condition has yet to be complied
claim; 4 days later, Infante registered the sale with the with. The anotacion preventiva obtained by Kalaw
adverse claim annotated thereto. Infante thereafter cannot create an advantage in his favor as the same was
introduced significant improvements on the property. good for only 30 days. The court ruled in favor of
They now dispute ownership over the said land. Mendoza.

ISSUE: Who has a better title, Carbonell or Infante?


19. ADALIN v CA
HELD: CARBONELL. In order to claim the benefit of Art.
1544, the buyer of realty must register the property in FACTS: Elena Kado and her siblings owned a lot with a 5-
good faith. It is a pre-condition to a superior title. In door commercial building fronting Imperial Hotel. The
this case, Infante was not in good faith, thus the prior units were leased. Elena contracted the services of
sale to Carbonell must prevail. Infante registered her Bautista, who brought Yu and Lim to her for the purpose
claim 4 days after the adverse claim was registered, she of buying the premises. During the meeting, it was
had notice that Carbonell paid off the mortgage debt as agreed that the Yu and Lim would buy the said units
the mortgage passbook was already in his possession. except for the 5th which is to be bought by Adalin. They
She likewise ignored Carbonell and refused to talk to entered into a Conditional Sale where Elena was
here. These are badges of bad faith that taint her obligated to evict the tenants before the full payment
registration. of the purchase price. Elena offered the same for sale to
the lessees but they refused claiming that they could
not afford; thus, she filed a case for ejectment against
17. SAN LORENZO DEV CORP v CA them. Thereafter, the lessees decided to exercise their
right to buy the unitsKalaw ruled that since the sale to
FACTS: Spouses Lu owned 2 parcels of land, which they Yu and Lim was conditional, the subsequent sale to the
purportedly sold to Babasanta. He demanded the lessees must be preferred.
execution of a Final Deed of Sale in his favor so he may
effect full payment of the purchase price; however, the ISSUE: Who has a better title, Yu and Lim or the lessees?
spouses declined to push through with the sale. They
claimed that when he requested for a discount and they HELD: YU AND LIM. While it is true that the Deed was for
refused, he rescinded the agreement. Thus, Babasanta Conditional Sale, examination of the contents thereof
filed a case for Specific Performance. San Lorenzo would show that it was one for the actual sale. During
Development Corp. (SLDC) intervened claiming that the the meeting, the property was already sold; the only
lots have been sold to it by virtue of a Deed of Absolute conditions were that Elena would evict the lessees
Sale with Mortgage and that it was a purchaser in good before the full payment of the price. The choice of to
faith. Both sales were not registered. whom to sell the property had already been decided.
That being the case, since the sale in favor of Yu and
ISSUE: Who has a better title, Babasanta or SLDC? Lim was the prior sale, it must be preferred.
Besides, Elena was guilty of double-dealing, which
HELD: SLDC. There was no double sale in this case cannot be sanctioned in law. It was, after all, her
because the contract in favor of Babasanta was a mere obligation to evict the lessees. The lessees were in bad
contract to sell; hence, Art. 1544 is not applicable. The faith as well for having knowledge of the supposed sale
ownership of the property was not to be transmitted in in favor of Yu and Lim. Their subsequent registration of
his favor until the full payment of the purchase price. the sale cannot shield them in their fraud.
There was neither actual nor constructive delivery as his
title is based on a mere receipt. Based on this alone, the
right of SLDC must be preferred.
ANTONIO | HIPOLITO | IMPERIAL | ZARAGOSA 23
%
20. CHENG v GENATO the simple rule on priority in time, priority in right
would apply. As such, the successors-in-interest of
FACTS: Genato owned 2 parcels of land in Paradise Gamiao and Dayag would have a better right as the sale
Farms. He agreed with the Da Jose spouses to enter into in their favor came ahead of time. Further, Marquez was
a contract to sell over the said parcels; it was embodied not in good faith. He knew that the property was being
in a public instrument annotated to the certificates of claimed by other parties who were in possession thereof
title. They asked for and were granted an extension for instead, he willfully closed his eyes to the possibility of
the payment of the purchase price. Unknown to them, the flaws.
Genato dealt with Cheng regarding the lot, executed an
Affidavit to annul the Contract to Sell, appraised the
latter of his decision to rescind the sale, and received a 22. ESTATE OF LINO OLAGUER v ONGJOCO
down payment from Cheng upon the guarantee that the
said contract to sell will be annulled. By chance, Genato FACTS: (Super detailed, with lots of unimportant facts,
and the spouses met at the RD, where he again agreed so I'll only state whatever is related to the case at hand)
to continue the contract with them. He advised Cheng Petitioners are the children of Lino Olaguer and Olivia
of his decision; the latter countered that the sale had Olaguer. When Lino died, Olivia became the
already been perfected. Cheng executed an Affidavit of administrator and Eduardo Olaguer as co-administrator
Adverse Claim and had it annotated to the TCTs and of his estate. Olivia then got married to Jose Olaguer. As
sued for specific performance. administrators, Olivia and Eduardo sold 12 parcels of
land owned by Lino to Pastor Bacani, including Lot 76
ISSUE: Who has a better title, Cheng or the Da Jose which is the lot in question. A day later, it was sold back
spouses? to them, splitting to them the portion of which 6/13
went to Olivia, while 7/13 went to Eduardo. She then
HELD: DA JOSE SPOUSES. Both agreements involve a made a special power of attorney in favor of Jose, giving
contract to sell, which makes Art. 1544 inapplicable him the power to sell, mortgage, transfer, assign
since neither a transfer of ownership nor a sales endorse and deliver with respect to her share over Lot
transaction took place. A contract to sell is premised 76. The lot was sudivided, having Lots 76-B to 76-G in
upon a suspensive conditionthe full payment of the the name of Olivia. As attorney-in-fact, Jose sold the six
purchase price. That being the case, the elementary parcels of land in favor of his son, Virgilio Olaguer. Lots
principle of first in time, priority in right should apply. 76-B and 76-C was consolidated and further subdivided
As such, the contract in favor of the Da Jose spouses into a proportional share, making them Lots 1 and 2.
must prevail considering that the same had not been Jose, claiming to be the attorney-in-fact of his son, sold
validly rescinded. Besides, Cheng cannot be considered Lots 1 and 2 to Emiliano Ongjoco. He further sold Lots
to have acted in good faith as he had knowledge of the 76-D to 76-G to Ongjoco twice on different dates, this
prior transaction in favor of the spouses. time evidenced by a notarized general power of
attorney. Petitioners moved for the sale made by
Spouses Olivia and Jose Olaguer to be null and void. RTC
21. CONSOLIDATED RURAL BANK INC v CA ruled in favor of petitioners (on the subject lots), but CA
reversed.
FACTS: The Madrid Brothers owned a parcel of land,
which was later subdivided. Rizal Madrid sold his share ISSUE: Whether Ongjoco was a buyer in good faith
to Gamaio and Dayag; the other brothers offered no
objection. The sale was not registered under the RULING: With respect to Lots 1 and 2, he cannot be
Torrens System. Gamaio and Dayag sold the southern considered a buyer in good faith since there was no
half to Teodoro, and the northern half to Hernandez, proof that the sale on both lots was evidenced by a
who thereafter donated the same to his daughter. They written power of attorney. According to Agency Law, a
all maintained possession of the properties. Later on, sale of a piece of land must be coupled with a written
the brothers all sold their shared to Marquez who authority of such agent, else the sale is void. Since the
further subdivided the same, registered the lands, and respondent was not able to show proof that there really
mortgaged portions thereof to Consolidated Bank and was an existing written authority, the sale over such lots
Bank of Cauayan. For failure to settle his debt, CB cannot be considered valid, and must be returned to the
foreclosed the property. The successors-in-interest of Estate of Lino Olaguer.
Gamiao and Dayag sought reconveyance. CB interposed With respect to Lots 76-D to 76-G, there was a notarized
that the mortgage must be respected. general power of attorney to show evidence that
authority had been given by Virgilio to his father to
ISSUE: Who has a better title, Marquez or the dispose the subject lots. Since petitioners was not able
successors-in-interest of Gamiao and Dayag to show any proof that the lots being sold twice to
respondent show bad faith, good faith must be
HELD: The successors-in-interest of Gamiao and Dayag. presumed. Being notarized, the regularity of such
While Marquez was the 1st to register the lands under general power of attorney must also be presumed.
the Torrens System, Art. 1544 does not apply as the
double-multiple sales were not done by the single
vendorin this case by the brothers on the one side and
Gamiao and Dayag on the other. That being the case,

ANTONIO | HIPOLITO | IMPERIAL | ZARAGOSA 24


%
23. ABRIGO v DE VERA 25. CARUMBA v CA

FACTS: By virtue of a compromise agreement judicially FACTS: Canuto sold a parcel of land to Carumba by
approved, Villafania sold to Rosenda and Rosita a house virtue of a Deed of Sale of Unregistered Land. The sale
and lot. Unknown to them, Villafania obtained a free was never registered. Thereafter, Canuto was sued for
patent over the said land and sold it to De Vera. On the collection of money, and the said land was levied upon
other hand, Rosenda and Rosita sold the property to the and sold to Balbuena, who registered it.
spouses Abrigo. Now De Vera and Abrigo dispute
ownership over the propertythe former filing an ISSUE: Who has a better right, Carumba or Balbuena?
ejectment suit against the latter.
HELD: CARUMBA. Art. 1544 does not apply in this case.
ISSUE: Who has a better title, Abrigo or De Vera? Instead, the Rules of Court are applicable. Balbuena,
the later vendee, merely steps into the shoes of the
HELD: DE VERA. Abrigo registered the property under judgment debtor and acquires all the rights and
Act 3344, while De Vera registered the same under the interests of the latter. By the time the lot was sold
Torrens System. Naturally, De Veras right prevails. through the foreclosure proceedings, it was no longer
Registration must be done in the proper registry to bind owned by Canuto by virtue of a prior sale to Carumba
the land. It was also proven that De Vera acted in good who has a better right.
faith considering that there was nothing in the
certificate of title or the circumstances, which would
have aroused suspicion and mandated her to make an 26. ACABAL v ACABAL
inquiry. Registration under Act 3344 does not suffice to
constitute constructive notice in order to negate the FACTS: Sps. Acabal sold their lot to their son Villaner
good faith of the registrant under the Torrens System. Acabal who in turn transferred it to his godson-nephew
De Veras right must be upheld. Leonardo Acabal. This was later on sold to Leonardo and
Ramon Nicolas hence a complaint was filed by Villaner
against them and his nephew arguing that what he
24. DAGUPAN TRADING CO v MACAM signed was a Lease contract and not a sales contract.
The RTC ruled in favor of Nicolas which was reversed by
FACTS: Sammy Maron and his 7 brothers were co-owners the CA thus the case at bar.
of a parcel of land for which they applied for
registration. Pending the proceedings, they sold the ISSUE: W/N there was a valid sale
same to Macam, who thereafter introduced substantial
improvements thereon. Later on, the property was HELD: YES
levied upon and sold in favor of Dagupan Trading, which It is valid only insofar as 5/9 of the land is concerned.
thereafter registered the Sheriffs Final Certificate of This is so because the property in question was bought
Sale during the pendency of the marriage of Villaner
therefore it is presumed to belong to the conjugal
ISSUE: Who has a better right, Macam or Dagupan? partnership. Leonarda failed to prove otherwise.
Nevertheless, when Justiniana (wife) died, her share
HELD: MACAM. In this case, the sale in favor of Macam vested on her 8 children, and her husband vesting him
was executed before the land was registered, while the with 5/9 share on the property. Since it is not yet
sale in favor of Dagupan was made after the partitioned, he cannot yet claim title to any definite
registration. In such a case, the Rules of Court will apply portion of the property but only to his ideal, abstract or
such that the delivery of the Sheriffs Final Certificate spiritual share. He may still dispose of the same for
of Sale in favor of Dagupan merely substitutes the latter every co-owner has absolute ownership over his
into the shoes of the seller Maron and acquires all undivided interest in the co-owned property. However,
rights, interests, and claims of the latter. Considering he cannot dispose of the shares of his co-owners based
that at the time of the levy, Maron was no longer the on nemo dat qui non habet. Since he sold it without the
owner of the land, then no title can thereafter pass in consent of the other co-owners, the sale is still valid
favor of Dagupan. Macams title is thus sustained. only insofar as his shares are concerned.
And the finding that both Leonardo and Villaner were
in pari delicto, the same is irrelevant because the
property concerned is unregistered.

ANTONIO | HIPOLITO | IMPERIAL | ZARAGOSA 25


%

SALE BY NON-OWNER/BY ONE HAVING VOIDABLE TITLE:


LIFE OF A CONTRACT OF SALE
1. PAULMITAN v CA also sell their undivided share in the co-ownership.
Otherwise, the properties sold would be subject to a
FACTS: When Agatona died, she was succeeded by 2 partition, which cannot happen to the properties in this
sons: Pascual and Donato. She left 2 parcels of land. case. School equipment, as well as the buildings, are
Pascual died leaving 7 heirs. The titles remained in the indivisible. Thus, they cannot be subject to partition.
name of Agatona and the lots were never partitioned.
Donato, thereafter, executed an affidavit of Declaration
of Heirshipunilaterally adjudicating one of the lots to 3. BUCTON v GABAR
himself. He thereafter sold the entire lot to his daughter
Juliana. For the failure to pay taxes, the lot was FACTS: Josefina bought a parcel of land from Villarin. By
forfeited and sold at a public auction, but Juliana later verbal agreement, Josefina sold a ! portion thereof to
redeemed the property. The Heirs of Pascual then Nicanora for P3,000. Nicanora paid P1,000 then P400all
surfaced and sought to partition the property. evidence by receiptsthen she loaned Josefina P1,000
and thereafter along with her spouse, took possession of
ISSUE: W/N Juliana became the owner of the entire lot the lot and built their house as well as apartments
upon her redemption of the property thereon. Villarin then issued a Deed of Sale to Josefina,
but the latter refused to execute the corresponding
HELD: NO.From the moment of Agatonas death, her Deed of Sale to Nicanora. Josefina claimed that the
heirs, Pascual and Donato, became co-owners of the amounts paid by Nicanora were in the concept of loans.
undivided lot. When Donato died, his pro-indiviso share Thus, Nicanora filed a case for specific performance.
transferred to his heirs. That being the case, when
Donato sold the entire property to his daughter, he was ISSUE: W/N there was a sale between Josefina and
merely co-owner thereof and transferred only his Nicanora
undivided share.
If a co-owner alienates the entire property without HELD: YES. Assuming that at the time when Josefina
the consent of the other co-owners, the sale will affect sold the lot to Nicanora, she was not yet the owner
only his share. Thus, only ! undivided share passed on thereof. When Villarin executed the Deed of Sale in her
to Juliana. The fact that Juliana redeemed the property favor, title passed to Nicanora by operation of law.
does not operate to terminate the co-ownership. It Although the sale between Josefina and Nicanora was
merely entitles her to reimbursement from the other co- verbal, it was as between them. Considering that
ownersredemption being a necessary expense. Until Nicanora has paid the purchase price, she became
reimbursement, Juliana holds a lien upon the lot for the owner of ! of the lot. Likewise, although the complaint
amount due to her. However, a partition is in order. was titled specific performance it was actually one for
quieting of title, which is imprescriptible so long as the
plaintiff is in possession of the lot.
2. MINDANAO v YAP

FACTS: Rosenda and Sotero were among co-owners of 3 4. CITY OF MANILA v BUGSUK LUMBER
parcels of land, which they sold to Ildefonso Yap for
some P100K without the consent of the other co-owners. FACTS: Bugsuk Lumber had an office in Manila. The City
They included in the sale certain buildings and Treasurer assessed it for license fees and mayors
laboratory and other educational equipment within the permitalleging that Bugsuk sold at wholesale and retail
said properties, which were actually owned by Mindanao to different lumber dealers in Manila. Bugsuk refused to
Academy. Mindanao Academy and the other co-owners pay alleging that the lumber it produced were delivered
assailed the validity of the sale. The trial court declared directly from the shipper to the buyer, that they paid
the sale null and void. Yap contends that Erlinda, one of the appropriate Timber License Fees and that their
the co-owners owning 5/12 share of the co-ownership, Manila Office only received orders and accepted
does not have the standing to challenge the sale for payments. Bugsuk alleges that it is not a dealer and its
being in bad faith. office is not a store to warrant the imposition of the
additional taxes.
ISSUE: W/N the sale is null and void as to its entirety
ISSUE: W/N Bugsuk is liable for the additional taxes
HELD: YES. Although the general rule is that if a co-
owner alienates the entire property without the consent HELD: NO. A dealer buys to sell again; Bugsuk produced
of the other co-owners, the sale will affect only his its own lumber from Palawan. Thus, it is not a dealer.
share, such rule does not apply if the property cannot Its Manila office is not a store as well. A store is a place
be partitioned/subdivided. In this case, aside from the where goods are kept for salewhether for retail or
fact that Rosenda and Sotero cannot sell the entire wholesale. The Manila office only processed the orders
property including the school equipment, they cannot and payments; it did not keep goods therein or act as a

ANTONIO | HIPOLITO | IMPERIAL | ZARAGOSA 26


%
dealer or intermediary between the field office and the Tagatac was NOT unlawfully deprived within the context
customers. Thus, it is not liable for the said taxes. of the Civil Code.
The sale between Feist and Tagatac was merely
voidablevalid until annulled. There was a valid
5. SUN BROS. & CO. v VELASCO transmission of ownership. The fact that Feist did not
pay only gives rise to an action to resolve the contract
FACTS: Sun Brothers sold an Admiral Refrigerator to or demand payment. When Feist sold the car to
Lopez upon the agreement that ownership will only pass Sanchez, the sale between him and Tagatac was still
to the latter upon payment of the full purchase price. valid; therefore, good title passed to Sanchez. As
Lopez paid only the downpayment and sold the same to between 2 innocent parties, the one who made possible
JV Trading (owned by Velasco) and was displayed in the the injury must bear the loss.
latters store. It was thereafter bought by CO Kang Chiu
from JV Trading. Sun Brothers sought to recover the
refrigerator. 7. EDCA PUBLISHING v SANTOS

ISSUE: W/N Sun Brothers may recover the thing FACTS: EDCA sold books to Tomas dela Pena who
fraudulently represented himself to be Prof. Jose Cruz,
HELD: NO. It is true that where a person who is not the a Dean of DLSU. EDCA delivered him the books, the
owner of a thing sells the same, the buyer acquires no check Tomas issued was dishonored because he did not
better title than the seller has. In this case. Lopez have an account at all. Tomas thereafter sold the books
obviously had no title to the goods for having failed to at a discount to Leonor Santos. EDCA, with the aid of
pay the full price. It only follows that JV Trading had no the police, stormed the Santos Bookstore to retrieve the
title thereto as Velasco was not in good faith. He should books.
have inquired if Lopez had good title to itthe same not
being engaged in the business of selling appliances. ISSUE: W/N EDCA may retrieve the books from Santos
HOWEVER, when the refrigerator passed to Co Kang
Chiu, the latter acquired valid title thereto. The HELD: NO. Ownership of the books passed to Tomas
exception to the foregoing rule is the purchase in good upon the delivery thereof. He had the right to transfer
faith in a merchant store or a fair or a market. This rule the same to Santos. The fact that he did not pay for the
fosters stability to commerce and business transactions. books only warrants rescission or an action for payment.
Co Kang Chiu purchased the refrigerator in a merchant EDCA cannot be considered to have been unlawfully
storeand for value and in good faith. Thus, he is deprived under the CC as to warrant recovery of the
protected by the law. Sun Brothers would not be books from Santos. Possession of movable property
entitled to recover the refrigeratornot even if they pay acquired in good faith is equivalent to title. Santos was
its valuesince they were not deprived of the same a buyer in good faith, thus he is protected by the law.
unlawfully. Lopez is the one who should be liable to Sun
Brothers for the full purchase price of the ref.
8. AZNAR v YAPDIANGCO

6. TAGATAC v JIMENEZ FACTS: Teodoro advertised for sale his Ford Fairlane
car. De Dios approached them purporting to be a
FACTS: Tagatac bought a car abroad and brought it to nephew of Marella. Teodoro transacted with Marella
the Philippines. Warner Feist deceived her into believing who agreed to buy the car, agreeing to pay the same
that he was very rich and purchased her car. She only after the car has been registered in his name. The
delivered possession thereof. Levy (another name of Deed was registered in his name, but Marella has yet to
Feist) issued her a postdated check, which was pay so the documents were not delivered to him, he
dishonored. Feist then disappeared with the car. Feist pleaded with Ireneo, Teodoros son, that they proceed
was able to register the car in his name and eventually to Marellas sister to secure the shortage of cash. Ireneo
sold the car to Sanchez, who then sold the same to agreed. They proceeded thereto, Ireneo was
Jimenez. Jimenez even labored to verify the cars accompanied by De Dios and an anonymous person. De
records with Motor Vehicle Office. Jimenez then Dios was able to induce Ireneo to hand over the
delivered the car to California Car Exchange for display. documents under the pretext that he will show them to
Tagatac, upon finding out, sought to recover the car, his lawyer, Ireneo agreed. De Dios made Ireneo wait and
but Jimenez refused. thereafter escaped with the car and the deed. Marella
was then able to sell the car to Aznar. The police
ISSUE: W/N Jimenez may refuse to give the car back thereafter seized the car in Aznars possession. Aznar
countered with a complaint for Replevin.
HELD: YES. Jimenez was a buyer in good faith of the
carhe had no knowledge of any defect in the title of ISSUE: W/N Teodoro may recover the car from Aznar
the seller. It is true that one who has lost any movable
or has been unlawfully deprived thereof may recover HELD: YES. Teodoro was clearly unlawfully deprived of
the same from the possessor. However, in this case, the car. There was no valid delivery to Marella, hence
the latter acquired no title to the car. Delivery must be

ANTONIO | HIPOLITO | IMPERIAL | ZARAGOSA 27


%
coupled with intent. That being the case, Teodoro has Dominador Dizon for the return of the ring pledged but
the right to claim the car not only from the thief, but refused to return the ring thus the case filed by Lourdes.
also from 3rd persons who may have acquired it in good The CFI issued a writ of replevin so Lourdes was able
faith. The buyer would only be entitled to to have possession of the ring during the pendency of
reimbursement if he purchased the same in good faith the case. The CFI also ruled in her favor which was
from a public sale. affirmed by the CA on appeal. Thus the case at bar.

ISSUE: W/N the CA erred in ruling that Lourdes has a


9. CRUZ v PAHATI right to possession of the ring

FACTS: Jose Cruz delivered his car to Belizo for the HELD: NO
latter to sell the same. Belizo forged the letter of Cruz It reiterated the ruling in de Garcia v. CA, that the
to the Motor Section of the Bureau of Public Works and controlling provision is Art. 559 of the CC which states
converted the same into a Deed of Sale. Using the that the possession ofmovable property acquired in
forged deed, he had the car registered in his name. good faith is equivalent to a title. Nevertheless, one
Thereafter, Belizo sold the car to Bulahan, who in turn who has lost any movable or has been unlawfully
sold the same to Pahati. However, the car was deprived thereof may recover it from the person in
impounded by the police, and the sale to Pahati was possession of the same. If the possessor of a movable
cancelled. Bulahan now contends that between 2 lost of which the owner has been unlawfully deprived,
innocent parties (Bulahan and Cruz), the person who has acquired it in good faith at a public sale, the owner
made possible the injury must bear the lossin this cannot obtain its return without reimbursing the price
case, supposedly Cruz. paid therefor.
Lourdes, being unlawfully deprived of her ring thus
ISSUE: W/N Cruz may recover the car from Bulahan she has a right to recover it from the current possessor.
Dizon is engaged in a business where presumably
HELD: YES. It is true that both Bulahan and Cruz acted in ordinary prudence would require him to inquire whether
good faith. One who has lost a movable or had been or not an individual who is offering the jewelry by
deprived of the same may recover it from the possessor. pledge is entitled to do so. The principle of estoppel
This rule applies squarely to this case. Thus, since Cruz cannot help him at all. Since there was no precaution
was unlawfully deprived by Belizo through the latters availed of, perhaps because of the difficulty of resisting
artifice, he is entitled to recover the same even against opportunity for profit, he only has himself to blame and
a subsequent purchaser in good faith. The only should be the last to complain if the right of the true
exception to this rule is if the purchaser acquired the owner of the jewelry should be recognized.
same from a public salein which case, reimbursement
is in order. It was, in fact, Bulahan who acted Other issues raised:
negligently in failing to detect the forged Deed of Sale. 1. Principle of estoppel = has its roots in equity, moral
right and natural justice.
" For estoppel to exist, there must be a
declaration, act or omission by the party who is
10. DIZON v SUNTAY
sought to be bound.
" A party should not be permitted to go against his
FACTS: Lourdes Suntay is the owner of a 3-carat
own acts to the prejudice of another.
diamond ring valued at P5,500. She and Clarita Sison
entered into a transaction wherein the ring would be
Concurring opinion by J. Teehankee:
sold on commission. Clarita received the ring and issued
" Interpretation of the unlawfully deprived in Art. 559
a receipt. After some time, Lourdes made demands for
of the CC. It is understood to include all cases where
the return of the ring but the latter refused to comply.
there has been no valid transmission of ownership. If
When Lourdes insisted on the return, Clarita gave her
our legislature intended interpretation to be that of
the pawnshop ticket which is the receipt of the pledge
the French Code, it certainly would have adopted and
and she found out that 3 days after the ring was
used a narrower term than the broad language of Art.
received by Clarita, it was pledged by Melia Sison, the
559 (formerly 464) and the accepted meaning in
niece of Claritas husband in connivance with Clarita
accordance with our jurisprudence.
with the pawnshop of Dominador Dizon for P2,600.
Lourdes then filed an estafa case. She then asked

ANTONIO | HIPOLITO | IMPERIAL | ZARAGOSA 28


%

LOSS, DETERIORATION, FRUITS AND OTHER BENEFITS


1. ROMAN v GRIMALT 2. LAWYERS COOPERATIVE PUBLISHING v TABORA

FACTS: Grimalt transacted with Roman for the purchase FACTS: Tabora purchased volumes of AmJur from
of a schooner called the Santa Marina. The sale was Lawyers Coop. The agreement was for ownership to
predicted upon the condition that it was seaworthy and remain with Lawyers Coop until payment of the full
that Roman would perfect his title theretothe same price. Loss or damage to the goods after delivery to
being registered to Paulina Giron. Only of the said the buyer is for the account of the latter. The books
conditions were complied with will Grimalt purchase the were delivered to his office; that same night, his office
same. The terms of payment were likewise agreed upon. was razed by fire. Tabora failed to pay the full purchase
Roman did nothing to perfect his title; then due to a price. Now Lawyers Coop sues him for the balance.
severe storm, the vessel sank. Roman now sues Grimalt Tabora invokes force majeure.
for the purchase price of the vessel.
ISSUE: Who bears the loss?
ISSUE: W/N Grimalt is liable for the loss
HELD: TABORA. While it is true that generally, loss is for
HELD: NO. There was yet to be a perfected contract the account of the owner, the same does not apply here
between them for the failure of Roman to perfect his because the parties themselves have expressly
title. That being the case, the loss is for the account of stipulated that loss, after delivery to the buyer, are for
Roman as the owner thereof. the account of the latter. Besides, the stipulation
retaining ownership to the seller is intended merely to
secure payment by the buyer. Likewise, the obligation
of Tabora consists of the delivery not a determinate
thing, but a generic thingmoney. Thus, he is not
absolved from liability.

REMEDIES FOR BREACH OF CONTRACT OF SALE


1. LEVY v GERVACIO balance payable in 24 installments. Title to the property
remained with Delta until the payment of the full
FACTS: Levy Hermanos sold a Packard car to Lazaro purchase price. Under the agreement, failure to pay 2
Gervacio. Gervacio made an initial payment and monthly installments makes the obligation entirely due
executed a promissory note for the balance of P2,400. and demandable. The units were delivered, Niu failed to
He failed to pay the note at maturity date so Levy pay. Thus, Delta filed a complaint for Replevin and
Hermanos foreclosed the mortgage and bought it at the applied the installments paid by Niu as rentals; Niu
public auction for P800. Levy Hermanos then filed a contends that the contractual stipulations are
complaint for the collection of the remaining balance unconscionable.
and interest.
CFI ruled in favor of Gervacio finding that Levy can no ISSUE: W/N the remedy Delta availed of was
longer recover the unpaid balance once he has chosen unconscionable
foreclosure. Thus the case at bar.
HELD: NO. A stipulation in the contract treating
ISSUE: W/N Levy Hermanos can still collect the balance installments as rentals in case of failure to pay is
VALIDso long as they are not unconscionable. The
HELD: YES provision in this case is reasonable.
In order to apply Art. 1454-A of the CC, there must be An unpaid seller has 3 alternative (not cumulative)
(1) a contract of sale of personal property payable in remedies: (1) exact fulfillment of the obligation, (2)
installments and (2) there has been a failure to pay 2 or cancel the sale for default in 2 installments, and (3)
more installments. foreclose the chattel mortgage (if any) but the seller
In the case at bar, although it is a sale of personal cannot anymore claim the unpaid balance of the price.
property, it is not payable in installments. It is payable Delta chose the 2nd remedy. Having done so, it is now
in a straight term in which the balance should be paid in barred from claiming the balance of the purchase price.
its totality at maturity date of the PN, therefore the
prohibition does not apply.
3. TAJANLANGIT v SOUTHERN MOTORS

2. DELTA MOTOR SALES CORP. v NIU KIM DUAN FACTS: Tajanlangit bought 2 tractors and a thresher
from Southern Motors. They executed a promissory note
FACTS: Niu Kim Duan purchased from Delta Motors 3 air in payment thereof; it contained an acceleration clause.
conditioning units. Niu paid the downpayment, the Tajanlangit failed to pay any of the stipulated

ANTONIO | HIPOLITO | IMPERIAL | ZARAGOSA 29


%
installments. Thus, Southern Motors sued him on the PN. HELD: NO. When the unpaid seller forecloses on the
The sheriff levied upon the properties of Tajanlangit mortgage, the law precludes him from bringing further
(same machineries) and sold them at a public auction to actions against the vendee for whatever balance, which
satisfy the debt. Southern Motors now prayed for was not satisfied by the first foreclosure. By choosing to
execution. Tajanlangit sought to annul the writ of foreclose on the Ford sedans, Filinvest renounced all
executionclaiming that since Southern Motors other rights which it might have had under the PN; it
repossessed the machineries (mortgaged), he was must content itself with the proceeds of the sale of the
therefore relieved from liability on the balance of the sedans at the public auction.
purchase price.

ISSUE: W/N Tajanlangit is relieved from his obligation to 6. ZAYAS v LUNETA MOTORS
pay
FACTS: Zayas purchased a Ford Thames Freighter from
HELD: NO. While it is true that the foreclosure on the Escano Enterprises, the dealer of Luneta Motor Co. The
chattel mortgage on the thing sold bars further action unit was delivered and Zayas issued a PN payable in 26
for the recovery of the balance of the purchase price, installments secured by a chattel mortgage over the
this does not apply in this case since Southern did not subject motor vehicle. Zayas failed to pay, thus Luneta
foreclose on the mortgage but insteas sued based on the extra-judicially foreclosed on the mortgage and was the
PNs exclusively. That being the case, it is not limited to highest bidder. However, considering that the proceeds
the proceeds of the sale on execution of the mortgaged of the sale was insufficient to cover the debt, Luneta
goods and may claim the balance from Tajanlangit. filed a case for the recovery of the balance of the
purchase price. Zayas refused to pay.

4. NONATO v IAC ISSUE: W/N Luneta may still recover the balance

FACTS: Nonato spouses purchased from Peoples Car a HELD: NO. When the unpaid seller forecloses on the
Volkwagen car. They issued a PN with chattel mortgage. mortgage, the law precludes him from bringing further
Peoples Car thereafter assigned its rights to the note to actions against the vendee for whatever balance, which
Investors Finance. The Nonatos defaulted, thus Investors was not satisfied from the foreclosure. Luneta contends
Finance repossessed the car and demanded the payment that Escano Enterprises is a different and distinct entity
of the balance of the purchase price. and maintains that its contract with Zayas was a loan.
This is unsubstantiated as the agency relationship
ISSUE: W/N Investors Finance may still demand for the between Luneta and Escano is clear.
payment of the balance when it repossessed the car Nevertheless, assuming that they were distinct
entities, the nature of the transaction remains the
HELD: NO. The remedies contemplated under Art. 1484 same. If Escano assigned its right to Luneta, the latter
are ALTERNATIVEnot cumulative. Investors Finance in merely acquires the rights of the formershence, Art.
effect cancelled the sale and it cannot now claim the 1484 of the CC would likewise be inapplicable.
balance of the purchase price. When it took possession
of the car, it gave the spouses 15 days to redeem the
car. This could mean that their failure to do so would 7. NORTHERN MOTORS v SAPINOSO
constrain the company to retain the permanent
possession of the car. There was no attempt at all the FACTS:
return the carthus, it is untrue that the same was Respondent Casiano Sapinoso purchased from
retained merely for appraisal. petitioner Northern Motors an Opel Kadett car for
P12,171 making a downpayment and executing a
promissory note for the balance of P10,540 payable in
5. RIDAD v FILIPINAS INVESTMENT (Filinvest) installments
To secure the payment of the note, Sapinoso executed
FACTS: Ridad purchased from Supreme Sales 2 Ford in favor of Northern Motors a chattel mortgage on the
Consul Sedans, payable in 24 installments, for which he car; the mortgage provided among others that upon
executed a PN with chattel mortgage over the said Sapinosos default in payment of any part of the
property. Another chattel mortgage was executed this principal or interest, Northern Motors may elect any of
time upon a separate Chevy car, and another one upon the ff. remedies (a) sale of the car by Northern (b)
the franchise to operate taxi cabs. Supreme Sales cancellation of the sale to Sapinoso (c) extrajudicial
thereafter assigned its rights under the PN to Filinvest. foreclosure (d) ordinary civil action for fulfillment of the
Ridad defaulted and Filinvest foreclosed on the mortgage contract; additionally, whichever remedy is
mortgage. It was the highest bidder for the foreclosure chosen, Sapinoso waives his right to reimbursement of
sale of the sedans. But unable to fully satisfy the debt, any and all amounts on the principal and interest
it also foreclosed the Chevy and the franchise. already paid
Sapinoso failed to pay the first 5 installments due from
ISSUE: W/N Filinvest may still foreclose the Chevy and August-November 1965; he made payments though on
the franchise to fully satisfy the debt November and December and on April the next year but
failed to make subsequent payments
ANTONIO | HIPOLITO | IMPERIAL | ZARAGOSA 30
%
Northern Motors filed a complaint stating that it was 8. CRUZ v FILIPINAS INVESTMENT & FINANCE CORP.
availing of the option of extrajudically foreclosing the
mortgage and prayed that (a) a writ of replevin be FACTS: Ruperto Cruz bought a bus from Far East Motor
issued upon its filing of a bond (b) it be declared to have Corp which was payable on installments of
the rightful possession of the car (c) in default of P1,487.20/month for 30 months with 12% interest. Cruz
delivery, Sapinoso be ordered to pay the balance with executed a PN in the sum of the purchase price. To
interest secure the paypent of the PN, Cruz executed a chattel
Subsequent to the commencement of the action but mortgage on the bus. Since no downpayment was made,
before filing of his answer, Sapinoso made 2 payments Far East required Cruz to execute another security and
amounting to P1,250 on the promissory note; in the for the a REM was executed on the land and building of
meantime, a writ of replevin was issued and the car was Mrs. Reyes which at that time was mortgaged to DBP.
turned over to Northern Motors Far East then assigned all its rights and indorsed the
Sapinoso claimed that he withheld payments because PN to Filipinas Investment and Financing Corp. Cruz
the car was defective and Northern Motors failed to fix defaulted in payment of the PN with only P500 being
it despite his repeated demands ever paid. Filipinas had the chattel mortgage foreclosed
and it was the highest bidder at the foreclosure sale.
TRIAL COURT RULING However, the proceeds were not sufficient to cover the
Northern Motors had the right to foreclose the chattel balance so it paid the indebtedness of Mrs. Reyes and
mortgage with Sapinoso failing to pay more than 2 requested that it be sold at foreclosure sale as well.
installments Thus Cruz and Mrs. Reyes filed an action with the CFI to
However, the foreclosure and the recovery of unpaid have the REM constituted on her land cancelled.
balance are alternative remedies which may not be The CFI ruled in favor of Cruz and Reyes finding that
pursued conjunctively; Northern Motors thereby the extrajudicial foreclosure barred further action for
renounced whatever claim it had on the promissory note recovery thus the case at bar.
Ordered Northern Motors to return of the P1,250 which
it had received from Sapinoso after filing the case and ISSUE: W/N recovery from an additional security is
electing to foreclose included in the prohibition thus allowing Filipinas to
recover the balance
ISSUE: W/N as under Article 1484 of the Civil Code,21
plaintiff Northern Motors is barred from recovering HELD: NO
unpaid balance of the debt having elected to foreclose Art. 1484 provides that when in a (1) a sale of
on the chattel mortgage. NO. personal property and (2) payable on installments there
was default in payment of 2 or more installments, the
HELD: remedies of the seller are:
In issuing the writ of replevin and upholding after trial 1. To exact fulfillment of the obligation, should
the right to possession of the car by Northern Motors, the vendor fail to pay
the court below correctly considered the action as one 2. Cancel the sale,
of replevin to secure possession of the car as preliminary 3. Foreclose the chattel mortgage on the thing
step to a foreclosure sale sold, if one has been constituted, should the
The court below however erred in concluding that the vendees failure to pay cover 2 or more
legal effect of the action was to bar Northern Motors installments. In this case, he shall have no
from accepting further payments on the promissory note further action against the purchaser to recover
It is the fact of foreclosure and actual sale of the any unpaid balance of the price. x x x
mortgaged chattel that bars further recovery by the
vendor of any balance on the vendees outstanding It has been held that these remedies are alternative
obligation not satisfied by the sale thus the exercise of one bars the exercise of the others.
In the present case, there is no occasion to apply the This is so to prevent he abuses committed in connection
restrictive provision of Article 1484 as there has not yet with foreclosure mortgages wherein the mortgagees
been a foreclosure sale resulting in a deficiency would seize the mortgaged property and buying them at
A mortgage creditor before the actual foreclosure sale a very low price at the sale and then bringing suit for
is not precluded from recovering the unpaid balance collection of the unpaid balance resulting in the
although he has filed for replevin for the purpose of mortgagor still liable to pay his original debt plus losing
extrajudicial foreclosure the property.
Also, a mortgage creditor who has elected to foreclose To allow Filipinas to recover thru the additional
but subsequently desists from proceeding with the security would result in a circumvention of the law.
auction sale without gaining any advantage and without Should the guarantor be compelled to pay the balance
causing any disadvantage to the mortgagor is not barred then the guarantor would be entitled to recover from
from suing on the unpaid account the debtor-vendee. In the end, it would still be the
And as applicable here, a mortgage creditor is not debtor-vendee who would bear the payment of the
barred from accepting before a foreclosure sale purchase price.
payments voluntarily tendered by the debtor-mortgagor Also, the word action in Art. 1484 covers all types of
who admits indebtedness. legal demand of ones right whether judicial or
extrajudicial thus the barring effect applies to an
extrajudicial foreclosure.

ANTONIO | HIPOLITO | IMPERIAL | ZARAGOSA 31


%
The car was worth 13,371 of which 1,160 was paid
upon delivery, and the remaining balance to be paid in
9. BORBON II v SERVICEWIDE SPECIALISTS INC. 24 monthly installments. A promissory note and chattel
mortgage were executed by the Ridads to ensure
FACTS: Daniel and Francisco Borbon issued a PN in favor fulfillment of their obligation.
of Pangasinan Auto Mart for the purchase of certain The Ridad Spouses failed to pay the last 5
chattels. It was secured by a chattel mortgage. The installments, which prompted Filipinas to file a replevin
rights under the note were assigned to Filinvest, which suit in the city court of Manila, or in the alternative, to
later assigned said rights to Servicewide. The Borbons recover the unpaid balance if delivery could not be
failed to pay, thus the mortgages were foreclosed. The effected.
Borbons aver that the seller delivered chattels not The complaint of Filipinas stated that there was an
strictly in accord with their instructions; nonetheless, unjustifiable failure and refusal of the Ridad Spouses to
they cannot evade liability because the notes have surrender the car for foreclosure. The sheriff was able
passed to holders for value and in good faith. The trial to seize the car and sell it in a public auction.
court sustained the foreclosure but awarded liquidated The Ridad Spouses were declared in default during
damages and attorneys fees in addition to the proceeds these proceedings due to their alleged non-receipt of
of the auction sale. summons; and the order of default included an order to
pay Filipinas 500 for attorneys fees and 163 for
ISSUE: W/N it was proper for the trial court to award expenses incurred in seizing the car. (this is what the
liquidated damages and attorneys fees in addition to case is about)
the proceeds of the auction sale Their motion to set aside the order of default was
denied by the city court of Manila, thus the Ridads
HELD: NO. First, when a person assigns credits to appealed to the CFI Manila. The CFI said that the only
another, the latter is bound under the same law; thus, issue to be resolved was with regard to the attys fees
Art. 1484 is equally applicable. In case of foreclosure, and the expenses incurred due to the seizure of the car.
the legislative intent is not merely to limit the It ruled that Filipinas was entitled to recover both
proscription to collecting the unpaid balance of the debt amounts; for the seizure of the car, and the lowered the
but also to other claims including costs of litigation and attys fees of 300.
attorneys fees. That being the case, the SC struck down This decision was appealed to the SC
the award of liquidated damages, but considering the
facts of the case, the award of attorneys fees is RIDAD SPOUSES CONTENTIONS
reasonable and sustained. Pursuant to Art 148422 (specifically #3) Filipinas, by
opting to foreclose the chattel mortgage renounced all
rights it had under the promissory note, as well as
payment for the unpaid balance, including any amount
10. MACONDRAY & CO v EUSTAQUIO
it would be entitled to under this action, such as attys
fees and costs of suit.
FACTS: Eustaquio bought a De Soto car from Macondray
for which he executed a PN, payable in installments,
FILIPINAS CONTENTIONS:
with a stipulation of attorneys fees, expenses for
They are entitled to an award for attys fees and costs
collection, and other costs. It was secured by a chattel
of suit by virtue of the unjustifiable failure and refusal
mortgage over the said car. As usual Eustaquio failed to
of the Ridad Spouses to comply with their obligations.
pay, and Macondray foreclosed on the mortgage.
What 1484 prohibit is the recovery of the unpaid
However, there remained a balance of some P340 for
balance by means of another replevin suit.
which Macondray sues Eustaquio. Macondray also
contends that at least the stipulated interests and
ISSUE: Whether under Art 1484 (the Recto Law) Fiipinas
attorneys fees must be claimable.
is entitled to the award for attorneys fees and expenses
incurred due to the seizure of the car. Yes, but with
ISSUE: W/N Macondray may still claim the interests and
certain qualifications.
attorneys fees stipulated
HELD: Art 1484 is called the Recto Law, and was created
HELD: NO. If the seller avails of his right to foreclose on
to protect mortgagors from mortgagees who wanted to
the mortgage, he can no longer bring an action against
unjustly enrich themselves. No. 3 of Art 1484, discussing
the buyer for the unpaid balancethis includes all the
the right to foreclose, means that if the vendor opts for
obligations such as attorneys fees, stipulated interests,
this remedy he shall have no further action to recover
expenses of collection and other costs.
any unpaid balance of the same.
The decided case of Macondray & Co. v Estaquio has
almost the exact facts and deals with the same issue of
11. FILIPINAS INVESTMENT & FINANCE CORP v RIDAD this case. In Macondray, it was ruled that the words
unpaid balance in no. 3 of art 1848 refer to the
FACTS: deficiency judgement which the mortgagee may be
The RIDAD SPOUSES purchased a Ford Consul Sedan entitled to, when after the public auction of the
from Supreme Sales and Devt. Corp. Supreme Sales was mortgaged chattel, the proceeds are insufficient to
the assignor-in-interest of FILIPINAS Investment and cover the full amount of the secured obligation, which
Finance, appellees herein.
ANTONIO | HIPOLITO | IMPERIAL | ZARAGOSA 32
%
include attorneys fees and costs of suit. Were it the and turn around to lease it to a client who gets, in
intention of legislature to limit its meaning to the addition, an option to purchase the property at the
unpaid balance of the principal, it would have so expiry of the lease period.
stated. In other words, the mortgagee is limited to the In the case at bar, PCI acquired the office equipments
property mortgaged and is not entitled to attys fees and for their subsequent lease to Giraffe, with the latter
costs of suit. undertaking to pay a monthly fixed rental for the whole
Such doctrine prevents the circumvention of the Recto 36 months. Giraffe made a guaranty deposit. Their
Law. Prior to its enactment, sellers unjustly enriched agreement was that in case Giraffe fails to pay any
themselves at the expense of their buyers; by recovering rental due, PCI will have cumulative remedies, such as,
the goods sold upon default of installment payers, by to recover all rentals for the remaining term of the
retaining the amounts already paid, and by claiming for lease and recover all amounts advanced for Giraffes
damages. Looking at the doctrine of Macondray, it account.
appears that in no instance may the mortgagee recover When PCI demanded for payment of the balance, it
any sum from the mortgagor after the foreclosure of the made a demand for either of the choices. Either to pay
mortgage. the balance hence Giraffe can keep the equipment or
But although the court agrees with the above stated surrender them if he cannot. The so-called monthly
doctrine, it seems that the mortgagees are not rentals were in fact monthly amortizations of the price
protected against perverse mortgagors. Examples of of the leased office equipment.
perverse mortgagors are those who deceitfully hide their The imperatives of equity, the contractual stipulations
mortgaged movables, or upon default of payment, and the actuations of the parties, the SC has treated a
refuse to give up its possession for foreclosure. When purported financial lease as actually a sale of movable
the mortgagor does these acts, the mortgagee has no property on installments and prevented recovery. The
choice but to institute a suit for replevin to recover Lease Agreement is in reality a lease with an option to
possession of the chattel and enforce his rights over purchase the equipment. This has been made manifest
such. It logically follows that the necessary expenses by the actions of PCI itself.
incurred by the mortgagee to regain possession of what In choosing replevin, PCI waived its right to bring an
he had a right to possess should be borne by the action to recover unpaid rentals.
mortgagor. Such recoverable expenses include attys
fees, and expenses incurred in seizing the chattel. In
this case, the court found that the amounts awarded by 13. LEGARDA v SALDANA
the lower court were reasonable.
The ruling in this case, in so far as it conflicts with FACTS: Saldana entered into a contract with Legarda
previously established doctrines, is pro tanto qualified. Hermanos. Legarda agreed to sell 2 equal lots for P1,500
each, payable in 120 equal installments over a period of
10 years at 10% per annum. Saldana paid 95 of the 120
12. PCI LEASING AND FINANCE INC v GIRAFFE-X installments over 8 years, which was recorded in his
CREATIVE IMAGING INC account with Legarda, but without stating as to which
lots the payments were made. The said account stated
FACTS: PCI Leasing and Giraffe entered into a Lease that Saldana still owed 1,311.72 for the 2 lots, although
Agreement whereby PCI Leasing leased several he had already pain more the P1,500, the value of one
machineries for a rent of P116, 878. 21/month for 36 lot.
months and P181, 362/month for 36 months for a total After 5 years, Saldana contacted Legardo Hermanos
of P10, 736, 647.56. Giraffe paid the amount of P3, 120, stating that he was interested in building a house on the
000 as guaranty deposit. However, after 1 year, Giraffe lots, however, he was prevented from doing such
defaulted in its monthly-rental payment obligations. because Hermanos failed to introduce the stipulated
After a 3-month default, PCI demanded a formal pay-or- improvements on the subdivision (roads to his lots). He
surrender-equipment type but the demand went further indicated his intentions to continue his
unheeded thus PCI instituted the instant case and payments.
prayed for the issuance for the writ of replevin. The In his reply, Legarda Hermanos said that since Saldana
trial court issued a writ of replevin. Giraffe filed a failed to complete the 120 payments in time, as they
motion to dismiss arguing that PCI was barred from have previously stipulated, all the amounts paid,
pursuing any other claim since the seizure of the 2 together with the improvements on the premises have
leased equipments because the contract was in reality a been considered as rents paid and as payment for
lease with option to buy. The RTC granted the motion to damages. Furthermore, the sale was cancelled.
dismiss ruling that it was akin to a contract covered by Saldana then filed an action demanding the delivery of
art. 1485 hence can no longer pursue its claim. Hence the 2 lots and for the execution of the corresponding
the case at bar. deed of conveyance after payment of the outstanding
balance.
ISSUE: W/N the contract was covered by Art. 1485 and Subsequently, Legarda Hermanos partitioned the
1484 hence barred PCI from recovering subdivision among the brothers and sisters, and the two
lots were among those allotted to Jose Legarda (co-
HELD: YES respondent).
A financial lease is one where a financing company The lower court sustained Legarda Hermanos
would, in effect, initially purchase a mobile equipment cancellation of the contracts and dismissed Saldanas

ANTONIO | HIPOLITO | IMPERIAL | ZARAGOSA 33


%
complaint. The CA eventually reversed this. The Board of Commissioners of the HLURB only modified
The CA ordered Legarda Hermanos to deliver to the award. The Office of the President adopted the
Saldana one of the two lots, at his option. Furthermore, findings of facts and conclusions of law by the Board
Hermanos was told to execute the deed of conveyance. thus was elevated to the CA which likewise affirmed the
decision of the OP. Hence, the case at bar.
ISSUE: Should the claim of Hermanos Legarda be upheld?
He claims that the payment should be considered as ISSUE: W/N Pacifico has paid at least 2 years of
rent and that the sale should be cancelled? No. installments

HELD: The SC applied the principles of equity and HELD: NO


justice, as correctly held by the CA. considering that The total payments made by Pacifico amounted to
Saldana had already paid the total sum of P3,582.06 P846, 600. What should be used as divisor is the amount
including interests, which is even more than the value of of the installment in the downpayment. The P750, 000
the two lots. downpayment was to be paid in 6 monthly installments
And even if the sum applied to the principal alone therefore deducting it from what he paid, the remaining
were to be considered, which was of the total of balance is P96,600. Pacifico was able to pay the
P1,682.28, the same was already more than the value of downpayment in 11 months after the last monthly
one lot, which is P1,500.00. The only balance due on installment was due. But he failed to pay at least 2
both lots was P1,317.72, which was even less than the years of installments therefore he is not entitled to a
value of one lot. By this, the court ruled that Saldana refund of the cash surrender value of his payments
had already paid for at least one lot. And he is given the under Sec. 3 of RA 6552. What is applicable is Sec 4
choice as to which one. which provides that the buyer should be given a grace
Even considering that Saldana as having defaulted period of not less than 60 days and if he should still fail
after February 1956, when he suspended payments after to do so, the seller may cancel the contract after 30
the 95th installment, he had as of the already paid by days from receipt of the buyer of the notice of
way of principal (P1,682.28) more than the full value of cancellation.
one lot (P1,500.00). Furthermore, regardless of the Pacifico admitted that the under the restructured
propriety of applying Article 1592 thereto, Legarda scheme, the 1st installment on the 70% balance of the
Hermanos was not denied substantial justice. According purchase was due on Jan 5, 1998. Although he issued
to ART. 1234, If the obligation has been substantially checks to cover for them, the 1st 2 were dishonored.
performed in good faith, the obligor may recover as When he was notified of the dishonor, he took no action
though there had been a strict and complete fulfillment, hence the 60 day grace period lapsed. Hence the
less damages suffered by the obligee, and that in the cancellation was justified.
interest of justice and equity, the decision appealed
from may be upheld upon the authority of ART. 1234.
15. MCLAUGHLIN v CA

14. JESTRA DEV AND MANAGEMENT CORP v PACIFICO FACTS: Petitioner Luisa McLaughlin (seller) and private
respondent Ramon Flores (buyer) entered into a
FACTS: Daniel Pacifico signed a Reservation application contract of conditional sale of real property. The total
with Fil-Estate Marketing Assn for the purchase of a purchase price is P140,000. P26,550 should be paid upon
house nad lot and paid the reservation fee. The execution of the deed and the balance not later than
Reservation application contained the amounts to be May 31, 1977 with an interest of 1% per month until fully
paid in installments with interests. Unable to comply paid.
with the schedule of payments, Pacifico requested Flores failed to pay and hence petitioner filed a
Jestra to allow him to make periodic payments which complaint for the rescission of the deed of conditional
the latter granted. They later on executed a contract to sale. Eventually, the parties entered into a compromise
sell when the remaining balance was only P260K. agreement, which was accepted by the court.
Pacifico requested twice for a restructuring of his The parties agreed that Flores shall pay P50,000 upon
unsettled obligation which Jestra granted subject to signing of the agreement and the balance in 2 equal
certain conditions of additional penalties et al. As installments payable on June 30, 1980 and December
compliance to the condition, Pacifico issued 12 post- 31, 1980. Flores also agreed to pay P1,000 monthly
dated checks however he is unable to pay so he rental until the obligation is fully paid for the use of the
requested that he be allowed to dispose the property to subject matter of the deed of conditional sale. They
recover his interest and he could recover the 12 post also agreed that in the event Flores fails to comply with
dated checks, which was this time was denied by Jestra. his obligations, the petitioner will be entitled to the
Jestra then sent a notarial notice of cancellation that issuance of a writ of execution rescinding the deed of
they are giving him until a certain date to pay or else conditional sale and all the payments made will be
the contract will be automatically cancelled. forfeited in favor of the plaintiff.
Pacifico then filed a complaint before the HLURB On October 15, 1980, petitioner wrote to Flores
claiming that despite his full payment of the demanding payment of the balance on or before October
downpayment, Jestra failed to deliver to him the 31. This demand included the installment due on June
property and instead sold it to another buyer. HRLURB 30 and December 31, 1980. On October 30, Flores sent a
Arbiter decided in Pacificos favor finding Jestra liable. letter signifying his willingness and intention to pay the

ANTONIO | HIPOLITO | IMPERIAL | ZARAGOSA 34


%
full balance. at least 2 years of regular installment payments.
On November 7, petitioner filed a motion for writ of But in cases where less than 2 years of installments
execution alleging that Flores failed to pay the were paid, the seller shall give the buyer a grace period
installment due on June 1980 and also failed to pay the of not less than 60 days from the date the installment
monthly rentals from that date. She prayed that the became due. If the buyer fails to pay the installments
deed of conditional sale be rescinded with forfeiture of due at the expiration of the grace period, the seller may
all payments and payment of the monthly rentals and cancel the contract after thirty days from the receipt by
eviction of Flores. The trial court granted the motion. the buyer of the notice of the cancellation or the
On November 17, Flores filed a motion for demand for rescission of the contract by a notarial act.
reconsideration tendering at the same time a certified Assuming that under the terms of agreement the
managers check payable to petitioner and covering the December 31 installment was due when on October 15
entire obligation including the December 1980 petitioner demanded payment of the balance on or
installment. The trial court denied the motion. before October 31, petitioner could cancel the contract
On appeal, the CA ruled in favor of Flores holding that after 30 days from the receipt by Flores of the notice of
the delay in payment was not a violation of an essential cancellation.
condition which would warrant a rescission since On Considering petitioners motion for execution filed on
November 17 or just 17 days from the October 31 November 7 as a notice of cancellation, petitioner could
deadline set by petitioner, Flores tendered the certified cancel the contract after 30 days from the receipt by
managers check and that it was inequitable for Flores Flores of said motion.
to forfeit all the payments made (P101,550). Flores tender of payment together with his motion
for reconsideration on November 17 was well within the
ISSUE: WHETHER it is inequitable to cancel the contract 30 day period granted by law.
and to have the amount paid by Flores be forfeited to The tender made by Flores of a certified bank
petitioner particularly after Flores had tendered the managers check was a valid tender of payment. It
certified managers check in full payment of the covered the full amount of the obligation. However,
obligation. YES. although he had made a valid tender of payment which
preserved his rights as a vendee, he did not follow it
HELD: There is already substantial compliance by Flores with consignation or deposit of the sum due with the
with the compromise agreement. More importantly, the court. Hence he remains liable for the payment of his
Maceda law recognizes the vendors right to cancel the obligation because of his failure to deposit the amount
contract to sell upon the breach and nonpayment of the due with the court.
stipulated installments but requires a grace period after

REMEDY OF RESCISSION IN SALES CONTRACT COVERING IMMOVABLES:


CONTRACT OF SALE VS. CONTRACT TO SELL
for annulment of the deed of sale in favor of Household
1. ADELFA PROPERTIES INC v CA Corporation and recovery of ownership of the property.
As a consequence, Adelfa Properties held payment of
FACTS: Rosario Jimenez-Castaneda, Salud Jimenez and the full purchase price and suggested that they settle
their brothers, Jose and Dominador Jimenez, were the the case with their nephews and nieces.
registered co-owners of a parcel of land in Las Pias. In In 1990, Adelfa Properties caused to be annotated on
1988, Jose and Dominador sold their share consisting of the TCT the exclusive option to purchase. On the same
1/2 of said parcel of land, specifically the eastern day, Rosario and Salud executed a Deed of Conditional
portion thereof, to Adelfa Properties. Subsequently, a Sale in favor of Emylene Chua over the same parcel of
Confirmatory Extrajudicial Partition Agreement was land for P3M, of which P1.5M was paid to the former on
executed by the Jimenezes, wherein the eastern portion said date, with the balance to be paid upon the transfer
of the subject lot, was adjudicated to Jose and of title to the specified 1/2 portion. Atty. Bernardo
Dominador Jimenez, while the western portion was wrote Rosario and Salud informing the latter that in
allocated to Rosario and Salud Jimenez. Thereafter, view of the dismissal of the case against them, Adelfa
Adelfa Properties expressed interest in buying the Properties was willing to pay the purchase price, and he
western portion of the property from Rosario and Salud. requested that the corresponding deed of absolute sale
Accordingly, in 1989, an Exclusive Option to Purchase be executed. This was ignored by Rosario and Salud.
was executed between the parties, with the condition Jimenez counsel sent a letter to Adelfa Properties
that the selling price shall be P2.86M, that the option enclosing therein a check for P25,000.00 representing
money of P50,000 shall be credited as partial payment the refund of 50% of the option money paid under the
upon the consummation of sale, that the balance is to exclusive option to purchase. Rosario and Salud then
be paid on or before 30 November 1989, and that in case requested Adelfa Properties to return the owners
of default by Adelfa Properties to pay the balance, the duplicate copy of the certificate of title of Salud
option is cancelled and 50% of the option money shall be Jimenez. Adelfa Properties failed to surrender the
forfeited and the other 50% refunded upon the sale of certificate of title.
the property to a third party. Rosario and Salud Jimenez filed a petition for the
Meanwhile, a complaint was filed by the nephews and annulment of contract, while Emylene Chua, the
nieces of Rosario and Salud against Jose and Dominador subsequent purchaser of the lot, filed a complaint in
ANTONIO | HIPOLITO | IMPERIAL | ZARAGOSA 35
%
intervention. RTC ruled in favor of the Jimenezes and sale as soon as it has been transferred in their name.
CA affirmed. The balance of P1.19M is due upon the execution of the
deed. When title to the property was finally transferred
ISSUE: W/N the contract between the Jimenezes and to their names, the Coronels sold the property to
Adelfa Properties is an option contract Mabanag for P1.58M after she paid P300K dp. Because of
this, they cancelled and rescinded the contract with
HELD: NO. The alleged option contract is a contract to Alcaraz by returning the P50,00 dp.
sell, rather than a contract of sale. The distinction Alcaraz filed a complaint for specific performance
between the two is important for in contract of sale, against the Coronels and cause the annotation of a
the title passes to the vendee upon the delivery of the notice of lis pendens on the TCT. Mabanag, on the other
thing sold; whereas in a contract to sell, by agreement hand, caused the annotation of a notice of adverse
the ownership is reserved in the vendor and is not to claim with the RD. However, the Coronels executed a
pass until the full payment of the price. In a contract of Deed of Absolute Sale in favor Mabanag. RTC ruled in
sale, the vendor has lost and cannot recover ownership favor of Alcaraz. CA affirmed.
until and unless the contract is resolved or rescinded;
whereas in a contract to sell, title is retained by the ISSUE: Whether the receipt of downpayment serves a
vendor until the full payment of the price, such contract to sell or a conditional contract of sale
payment being a positive suspensive condition and
failure of which is not a breach but an event that HELD: NO. The agreement could not have been a
prevents the obligation of the vendor to convey title contract to sell because the sellers made no express
from becoming effective. Thus, a deed of sale is reservation of ownership or title to the subject parcel of
considered absolute in nature where there is neither a land. Furthermore, the circumstance, which prevented
stipulation in the deed that title to the property sold is the parties from entering into an absolute contract of
reserved in the seller until the full payment of the sale, pertained to the sellers themselves (the certificate
price, nor one giving the vendor the right to unilaterally of title was not in their names) and not the full payment
resolve the contract the moment the buyer fails to pay of the purchase price. Under the established facts and
within a fixed period. circumstances of the case, had the certificate of title
The parties never intended to transfer ownership to been in the names of petitioners-sellers at that time,
Adelfa Properties to completion of payment of the there would have been no reason why an absolute
purchase price, this is inferred by the fact that the contract of sale could not have been executed and
exclusive option to purchase, although it provided for consummated right there and then. Moreover, unlike in
automatic rescission of the contract and partial a contract to sell, petitioners did not merely promise to
forfeiture of the amount already paid in case of default, sell the property to private respondent upon the
does not mention that Adelfa Properties is obliged to fulfillment of the suspensive condition. On the contrary,
return possession or ownership of the property as a having already agreed to sell the subject property, they
consequence of non-payment. There is no stipulation undertook to have the certificate of title changed to
anent reversion or reconveyance of the property in the their names and immediately thereafter, to execute the
event that petitioner does not comply with its written deed of absolute sale. What is clearly
obligation. With the absence of such a stipulation, it established by the plain language of the subject
may legally be inferred that there was an implied document is that when the said Receipt of Down
agreement that ownership shall not pass to the Payment was prepared and signed by petitioners, the
purchaser until he had fully paid the price. Article 1478 parties had agreed to a conditional contract of sale,
of the Civil Code does not require that such a stipulation consummation of which is subject only to the successful
be expressly made. Consequently, an implied stipulation transfer of the certificate of title from the name of
to that effect is considered valid and binding and petitioners father to their names. The suspensive
enforceable between the parties. A contract which condition was fulfilled on 6 February 1985 and thus, the
contains this kind of stipulation is considered a contract conditional contract of sale between the parties became
to sell. Moreover, that the parties really intended to obligatory, the only act required for the consummation
execute a contract to sell is bolstered by the fact that thereof being the delivery of the property by means of
the deed of absolute sale would have been issued only the execution of the deed of absolute sale in a public
upon the payment of the balance of the purchase price, instrument, which petitioners unequivocally committed
as may be gleaned from Adelfa Properties letter dated themselves to do as evidenced by the Receipt of Down
16 April 1990 wherein it informed the vendors that it is Payment.
now ready and willing to pay you simultaneously with
the execution of the corresponding deed of absolute
sale. 3. PNB v CA

FACTS:
2. CORONEL v CA PNB owned a parcel of land which Lapaz Kaw Ngo
offered to buy. Events under the first letter-agreement
FACTS: In 1985, Coronel executed a document entitled PNB accepted Lapazs offer subject to certain
"Receipt of Down Payment" in favor of Alcaraz for stipulations. The important ones are the following:
P50,000 dp of P1.24M as purchase price for an inherited 1. The selling price shall be P5.4million. Lapaz had
house and lot promising to execute a deed of absolute already paid P100,000 as deposit.

ANTONIO | HIPOLITO | IMPERIAL | ZARAGOSA 36


%
2. Upon failure to pay the additional deposit worth the vendors obligation to transfer title is subordinated
P970,000, the P100,000 shall be forfeited and PNB shall to a happening of a future and uncertain event. So that
be authorized to sell the property to another. if the suspensive condition does not take place, the
3. The property shall be cleared of its present tenants at parties would stand as if the conditional obligation
the expense of Lapaz. never existed.
4. Sale was subject to other terms and conditions to be In the instant case, the second letter-agreement was
imposed. replete with conditions that Lapaz had to fulfill before
Lapaz agreed, so she proceeded to clear the lot of its the sale could be executed. The sale was dependent
tenants at her own expense. upon Lapazs compliance with certain conditions (i.e.,
However, due to difficulties in money, she requested payment, eviction of occupants). It was stipulated in
for adjustment of payment proposals, which the bank the contract that her failure to pay the additional
denied. PNB also reminded her that she had not yet sent deposit would allow PNB to forfeit the price and allow
her letter of conformity to the agreement reached and them to sell the property to other parties.
told her to pay the full price of P5.4million. If not, the This stipulation took the nature of a reservation of
lot will be sold to other parties. title in the vendor until full payment of the purchase
Lapaz requested for a reduction of the price as the price, or giving the vendor the right to unilaterally
size of the land was substantially reduced. PNB agreed. rescind the contract the moment the buyer fails to pay
PNB still did not receive payment from Lapaz, and within the fixed period.
gave the latter the last chance to pay the balance of the In addition, Lapazs refusal to evict the occupants on
down payment. If she failed to pay, the sale shall be the ground that she had already done so under the first
cancelled and the P100,000 payment shall be forfeited. agreement was not justified as the two letter-
Lapaz failed to pay, so P100,000 was forfeited and the agreements were different transactions all together.
sale never materialized. PNB leased the premises to a Her fulfillment of the conditions in the first one did not
certain Rivera. carry over to the second one despite the identity of the
Lapaz requested for a refund of her deposit in the stipulation.
total amount of P660,000 and asked that the forfeited The P100,000 and the P200,000 were not also earnest
P100,000 be reduced to P30,000. PNB agreed. Events money. Article 1482, which defines earnest money,
under the second letter-agreement gives only a disputable presumption that prevails in the
Lapaz requested for a revival of the previously absence of rebuttal evidence. In the instant case, the
approved offer to PNB. PNB approved. letter-agreements themselves were the evidence that
All conditions as in the first agreement were the same, proved the intention of the parties to enter into
except for the purchase price and deposit. The price negotiations leading to a contract of sale mutually
was P5.1million, the deposit was P200,000. acceptable to both as to absolutely bind them. The
Lapaz refused, however, to conform to the condition P100,000 and the P200,000 could not have been proof of
of vacating the premises at her expense as she had the perfection of the sale as the letter-agreements were
already done so under the first agreement. (She full of condition precedents before the sale could be
apparently considered this second letter-agreement as a executed. The money thus given could be considered as
continuation of the first so she said that she was no part of the consideration of PNBs promise to reserve
longer required to evict the tenants as she had already the property for Lapaz.
done so.) Besides, according to her, the occupants of
the property were tenants of PNB. PNB refused this
offer. 4. BABASA v CA
To prevent the forfeiture of her P200,000 deposit, she
signed the letter-agreement. She told PNB that she was FACTS: In 1981, a contract of Conditional Sale of
willing to pay the remaining deposit of P800K as long as Registered Lands was executed between the spouses
it was PNB who would clear the property. PNB refused, Vivencio and Elena Babasa as vendors and Tabangao
and forfeited the P200,000 of Lapaz. Realty Inc. (Tabangao) as vendee over 3 parcels of land
PNB informed Lapaz that they had already decided to in Batangas. Since the certificates of title over the lots
sell the property for not less than P7M. were in the name of third persons who had already
executed deeds of reconveyance and disclaimer in favor
ISSUES: of the Babasas, it was agreed that the total purchase
1. Whether or not there was a perfected contract of price of P2,121,920.00 would be paid in the following
sale. No. There was no perfected contract of sale. manner: P300,000.00 upon signing of the contract, and
2. Whether or not the P100,000 or the P200,000 was P1,821,920.00 upon presentation by the Babasas of
earnest money. No. They were not earnest money. transfer certificates of titles in their name, free from all
liens and encumbrances, and delivery of registerable
HELD: documents of sale in favor of Tabangao within 20
It is important to note that the first letter-agreement months from the signing of the contract. In the
was cancelled and thereafter no longer existed. The meantime, the retained balance of the purchase price
second letter-agreement is not a contract of sale but a would earn interest at 17% per annum or P20,648.43
contract to sell whose conditions were not fulfilled, monthly payable to the Babasas until 31 December 1982.
which prevented the obligations therein from obtaining It was expressly stipulated that Tabangao would have
obligatory force. the absolute and unconditional right to take immediate
A contract to sell is one where the obligatory force of possession of the lots as well as introduce any

ANTONIO | HIPOLITO | IMPERIAL | ZARAGOSA 37


%
improvements thereon. On 18 May 1981 Tabangao instant case, ownership over Lots 17827-A, 17827-B and
leased the lots to Shell Gas Philippines, Inc. (SHELL), 17827-C passed to Tabangao both by constructive and
which immediately started the construction thereon of a actual delivery. Constructive delivery was accomplished
Liquefied Petroleum Gas Terminal Project, an approved upon the execution of the contract of 11 April 1981
zone export enterprise of the Export Processing Zone. without any reservation of title on the part of the
Tabangao is the real estate arm of SHELL. The parties Babasas while actual delivery was made when Tabangao
substantially complied with the terms of the contract. took unconditional possession of the lots and leased
Tabangao paid the first installment of P300,000.00 to them to its associate company SHELL which constructed
the Babasas while the latter delivered actual possession its multi-million peso LPG Project thereon.
of the lots to the former. In addition, Tabangao paid In Romero v. Court of Appeals and Lim v. Court of
P379,625.00 to the tenants of the lots as disturbance Appeals, the Court distinguished between a condition
compensation and as payment for existing crops as well imposed on the perfection of a contract and a condition
as P334,700.00 to the owners of the houses standing imposed merely on the performance of an obligation.
thereon in addition to granting them residential lots While failure to comply with the first condition results in
with the total area of 2,800 square meters. Tabangao the failure of a contract, failure to comply with the
likewise paid the stipulated monthly interest for the 20- second merely gives the other party the option to either
month period amounting to P408,580.80. Meanwhile, the refuse to proceed with the sale or to waive the
Babasas filed Civil Case 519 and Petition 373 for the condition. In the present case, the spouses contract
transfer of titles of the lots in their name. However, 2 with Tabangao did not lose its efficacy when the 20-
days prior to the expiration of the 20-month period, month period stipulated therein expired without the
specifically on 31 December 1982, the Babasas asked spouses being able to deliver clean certificates of title
Tabangao for an indefinite extension within which to such that Tabangao may no longer demand performance
deliver clean titles over the lots. They asked that of their obligation.
Tabangao continue paying the monthly interest of
P20,648.43 starting January 1983 on the ground that
Civil Case 519 and Petition 373 had not yet been 5. VALDEZ v CA
resolved with finality in their favor. Tabangao refused
the request. In retaliation the Babasas executed a FACTS: Carlos Valdez Sr. and Josefina Valdez were
notarized unilateral rescission dated 28 February 1983 owners of a parcel of land. When Carlos Sr. died,
to which Tabangao responded by reminding the Babasas Josefina subdivided the property into eight lots. On May
that they were the ones who did not comply with their 1, 1979, she executed a special power attorney,
contractual obligation to deliver clean titles within the authorizing her son Carlos Jr, who was a practicing
stipulated 20-month period, hence, had no right to lawyer, to sell a portion thereof (lots 3-C and 3-D) to
rescind their contract. The Babasas insisted on the Jose Lagon for P80,000. Part of consideration was also
unilateral rescission and demanded that SHELL vacate for Lagon to transfer the Rural Bank of Isulan to the
the lots. subject property, and to construct a commercial
On 19 July 1983 Tabangao instituted an action for building beside the bank. Without knowledge of
specific performance with damages in the RTC Batangas Josefina, Carlos Jr. entered into a different agreement,
City to compel the spouses to comply with their selling the property for P40/square meter, and it was
obligation to deliver clean titles over the properties. indicated in the deed that the P80,000 had already been
The Babasas moved to dismiss the complaint on the paid in cash. A downpayment of P20,000 was paid by the
ground that their contract with Tabangao became null wife of Lagon, to which Josefina issued a receipt. Carlos
and void with the expiration of the 20-month period Jr. prepared an affidavit, signed by Lagon, the transfer
given them within which to deliver clean certificates of of the bank and the construction of commercial building
title. SHELL entered the dispute as intervenor praying as part of the condition, else the deed of absolute sale
that its lease over the premises be respected by the shall be null and void without need of demand. Lagon
Babasas. RTC ruled in favor of Tabangao and Shell. CA failed to comply with the considerations stated in the
affirmed. deed, to which the Valdez refused to deliver the torrens
title. Lagon had Lot 3-C to be subdivided into three
ISSUE: W/N there was a contract of absolute sale separate lots, to which he paid the professional
between the Babasa and Tabagao services. Josefina used the subdivision survey, and sold
Lot 3-C-2 to PCIB, evidenced by a deed of absolute sale,
HELD: YES. Although denominated Conditional Sale of exectued a real mortgage over Lot 3-C-3 to DBP, and
Registered Lands, the contract between the spouses executed a deed of absolute sale in favor of Carlos Jr.
and Tabangao is one of absolute sale. Aside from the over Lot 3-C-1. She also sold lot 3-D to Engr. Rodolfo
terms and stipulations used therein indicating such kind Delfin. Lagon filed a complaint against Josefina and
of sale, there is absolutely no proviso reserving title in Carlos Jr for specific performance and damages. Trial
the Babasas until full payment of the purchase price, Court ruled in favor of Lagon. CA reversed, but reversed
nor any stipulation giving them the right to unilaterally itself, ruling in favor or Lagon.
rescind the contract in case of non-payment. A deed of
sale is absolute in nature although denominated a ISSUE: Whether the agreement was a contract of sale or
conditional sale absent such stipulations. In such cases, contract to sell / Whether the contract was ratified
ownership of the thing sold passes to the vendee upon
the constructive or actual delivery thereof. In the RULING: It is a contract of sale. The nature of the

ANTONIO | HIPOLITO | IMPERIAL | ZARAGOSA 38


%
contract must be inferred from the express terms and question to Jabil as early as 27 March 1965 so that the
agreement and from the contemporaneous and latter constructed thereon Sallys Beach Resort also
subsequent acts of the parties thereto. When Josefina, known as Jabils Beach Resort in March, 1965; Mactan
through her son acting a an attorney-in-fact, executed a White Beach Resort on 15 January 1966 and Bevirlyns
deed of absolute sale in favor of Lagon, she did not Beach Resort on 1 September 1965. Such facts were
reserve the ownership of the property, subject to the admitted by the Dignos spouses.
completion of payment of the consideration. However,
Carlos Jr. exceeded his authority when he entered into a
different agreement with Lagon, making the contract 7. UNIVERSITY OF THE PHILIPPINES v DELOS ANGELES
unenforceable, unless ratified. In this case, it was
ratified when Josefina accepted the downpayment of FACTS:
P20,000 and issued a receipt as a consequence of UP was given a land grant which shall be developed to
ratifying the contract. It must be noted, however, that obtain additional income for its support.
an affidavit was signed by Lagon as part of the UP and ALUMCO entered into a logging agreement
consideration, to transfer the Rural Bank of Isulan as where ALUMCO was granted the exclusive authority for
well as constructing a commercial bank beside the bank, an extendible period of 5 years (by mutual agreement),
both failed to perform by Lagon, making the deed of to cut and remove timber from the land grant
absolute sale null and void. It cannot be considered as inconsideration of royalties and fees to be paid to UP.
an afterthought contrived by Carlos Jr. since Lagon ALUMCO incurred an unpaid amount of P219,363. UP
admitted in court the authenticity of the affidavit, and demanded payment but it failed to pay. ALUMCO
its binding effect against him. There was no need to received a letter that UP would rescind or terminate
rescind the contract because it was clearly stipulated their logging agreement. They executed an instrument
that failure to comply with such obligation makes the Acknowledgement of Debt & Proposed Manner of
deed null and void, though petitioners are obliged to Payment which the UP President approved. ALUMCO
refund the respondent's partial payment of the subject agreed to give their creditor (UP) the right to consider
property. the logging agreement as rescinded without necessity of
any judicial suit and creditor will be entitled to P50,000
for liquidated damages.
6. DIGNOS v CA ALUMCO continued logging but still incurred unpaid
accounts. UP then informed them that as of that date,
FACTS: Dignos is the owner of a parcel of land in Lapu- they considered rescinded the agreement and of no
Lapu City, which they sold to Jabil for P28,000, payable further legal effect. UP then filed for collection of the
in 2 installments and with an assumption of unpaid accounts and the trial court gave them
indebtedness with First Insular Bank of Cebu for preliminary injunction to prevent ALUMCO from
P12,000. However, Dignos also sold the same land in continuing their logging.
favor of Cabigas, who were US citizens, for P35,000. A Through a public bidding, the concession was awarded
Deed of Absolute Sale was executed in favor of the to Sta. Clara Lumber Company and a new agreement
Cabigas spouses. was entered into between them and UP.
Jabil filed a suit against Dignos with CFI of Cebu. RTC ALUMCO tried to enjoin the bidding but the contract
ruled in favor of Jabil and declared the sale to Cabigas was already concluded and Sta. Clara started its
null and void. On appeal, CA affirmed RTC decision with operation.
modification. Upon motion by ALUMCO, UP was declared in
contempt of court for violating the writ of injunction
ISSUE: W/N the contract between Dignos and Jabil is a against them.
contract of sale (as opposed to a contract to sale) ALUMCOs contentions are the following:
a. It blamed its former general manager for their
HELD: YES. A deed of sale is absolute in nature although failure to pay their account.
denominated as a Deed of Conditional Sale where b. Logs cut were rotten; thus, they were unable to
nowhere in the contract in question is a proviso or sell them.
stipulation to the effect that title to the property sold is c. UPs unilateral rescission was invalid without a
reserved in the vendor until full payment of the court order.
purchase price, nor is there a stipulation giving the
vendor the right to unilaterally rescind the contract the ISSUE: W/N UP can validly rescind its agreement with
moment the vendee fails to pay within a fixed period. In ALUMCO even without court order. Yes. UP can
the present case, there is no stipulation reserving the unilaterally rescind the agreement.
title of the property on the vendors nor does it give
them the right to unilaterally rescind the contract upon HELD:
non-payment of the balance thereof within a fixed UP and ALUMCO expressly stipulated in their
period. Acknowledgement of Debt that upon default of
While there was no constructive delivery of the land payment, creditor UP has the right and power to rescind
sold in the present case, as subject Deed of Sale is a their Logging Agreement without the necessity of a
private instrument, it is beyond question that there was judicial suit.
actual delivery thereof. As found by the trial court, the There is nothing in the law that prohibits the parties
Dignos spouses delivered the possession of the land in from entering into agreements that violation of terms of

ANTONIO | HIPOLITO | IMPERIAL | ZARAGOSA 39


%
the contract would cause its cancellation even without P246.42 until fully paid. Par. 6 of the contract provided
court intervention. for automatic extrajudicial rescission upon default in
Act of a party in treating a contract as cancelled on payment of any monthly installment after the lapse of
account of any infraction by the other party must be 90 days from the expiration of the grace period of a
made known to the other and is always provisional, month, without need of notice and forfeiture of all
being subject to scrutiny and review by the proper installments paid.
court. Dumpit was able to pay the dp and several
If the other party deems the rescission unjustified, he installments amounting to P13,722.50, with the last
free to resort to judicial action. The court shall, after payment made on Dec. 5, 1967 for installments up to
due hearing, decide if the rescission was proper, in Sept. 1967.
which case it will be affirmed and if not proper, the In 1973, Dumpit requested Palay Inc to update his
responsible party will be liable for damages. overdue accounts and sought its permission to assign his
A party who deems the contract violated may consider rights to Dizon. However, Palay informed him that his
it rescinded and act accordingly, even without court Contract to Sell had long been rescinded pursuant to
action but it proceeds at its own risk. Only the final Par. 6 and that the lot had already been resold.
judgment of the court will conclusively settle whether Dumpit filed a complaint with the NHA for
the action taken was proper or not. But the law does not reconveyance with an alternative prayer for refund.
prohibit the parties from exercising due diligence to NHA ruled in favor of Dumpit, stating that the rescission
minimize their own damages. is void for lack of either judicial or notarial demand.
UP was able to show a prima facie case of breach of Office of the President affirmed.
contract and default in payment by ALUMCO. Excuses by
ALUMCO are not proper for them to suspend their ISSUE:
payments. 1. W/N notice or demand may be dispensed with by
Thus, the Supreme Court lifted the injunction. stipulation in a contract to sell
2. W/N Palay should be liable for the refund of the
installment payments made by Dumpit
8. PALAY INC v CLAVE
HELD:
FACTS: In 1965, Palay Inc., through its President 1. NO. Although a judicial action for rescission of a
Onstott, executed in favor of Dumpit (respondent) a contract is not necessary where the contract provides
Contract to Sell a parcel of land in Antipolo, RIzal. The for its revocation and cancellation for violation of any of
sale was for P23,300 with 9% interest p.a., payable with its terms and condition, jurisprudence has shown that at
a downpayment of P4,660 and monthly installments of
least, there was a written notice sent to the defaulter including the said land, to Pacifico Banking Corp. (PBC).
informing him of the rescission. Par. 6 cannot be In 1975, the Orlinos, who remained in possession of the
considered a waiver of Dumpit's right to be notified land, made a written offer to PBC to redeem the
because it was a contract of adhesion. A waiver must be property. In response, the bank agreed provided that
certain and unequivocal and intelligently made; such P160K should be paid in full upon signing of the Deed of
waiver follows only where the liberty of choice has been Absolute Sale and that as additional consideration,
fully accorded. Orlinos' share on a property in Caloocan City should be
Moreover, the indispensability of notice of conveyed to the bank.
cancellation to the buyer is protected under RA 6551. It After a year, PBC advised the Orlinos that if the
is a matter of public policy to protect the buyers of real transaction will not be finalized in 30 days, it would be
estate on installment payments against onerous and offered to other buyers. However, negotiations ensued
oppressive conditions. Waiver of notice is one such between them until 2 years after, PBC sold the land to
onerous and oppressive condition to buyers of real spouses Lim for P300K.
estate on installment payments. The Orlinos filed a complaint against PBC and Lim for
the annulment of the deed of sale on the ground that
2. YES. As a consequence of the rescission of the the subject land had bee earlier sold to them. RTC held
contract, right to the lot should be restored to Dumpit that PBC and Lim acted in bad faith knowing that there
or the same should be replaced by another acceptable was a cloud in the status of the property. CA affirmed.
lot. However, considering that the lot had been resold
to a third person, Dumpit is entitled to refund of the ISSUE: Whether the transaction between PBC and the
installments paid plus legal interest of 12%. Orlinos is a contract to sell or a contract of sale

HELD: CONTRACT TO SELL. There was no immediate


9. LIM v CA transfer of title to the Orlinos as what would have
happened if there had been a sale. The supposed sale
FACTS: In 1965, Orlinos (3 co-owners) mortgaged a was never registered and there was no new TCT in favor
parcel of land in Diliman, QC to Progressive Commercial of the Orlinos. They also acknowledged that the title to
Bank as security for a P100K loan. They failed to pay the the property would remain with the bank until their
loan and the mortgage was foreclosed, where the bank transaction shall be finalized. Moreover, the
acquired the property as the highest bidder at the consideration agreed upon was never paid to convert
auction sale. The bank transferred all its assets, the agreement into a contract of sale. As payment of

ANTONIO | HIPOLITO | IMPERIAL | ZARAGOSA 40


%
the consideration was a positive suspensive condition, SC's decision reversing the CA's decision and ordering the
title to the property never passed to the private RD to cancel the notice of lis pendens on the titles
respondents. Thus, the property was legally issued to AFPMBAI, declaring it as buyer in good faith
unencumbered and still belonged to PBC when it was and for value.
sold to Lim. Investco Inc and Solid Homes Inc entered into a
On RESCISSION: Although a contract to sell imposes contract to sell. During this time, the titles to the
reciprocal obligations and cannot be terminated Quezon City and Marikina properties had not been
unilaterally by either party, judicial rescission is transferred in the name of Investco Inc as asignee of the
required under Art. 1911 of the CC. However, this rule is owners, Angela Perez Staley and Antonio Perez. Thus,
not absolute. Jurisprudence has shown that a party may Investco Inc merely agreed to sell and Solid Homes to
take it upon itself to consider the contract rescinded buy the former's rights and interest in the properties.
and act accordingly albeit subject to judicial However, Solid Homes Inc. reneged or defaulted on its
confirmation, which may or may not be given. It is true obligation. Thus, Investco Inc rescinded extra-judicially
that the rescinding party takes a risk that its action may such contract to sell. After such event, AFPMBAI and
not be approved by the court. The Orlinos obligated Investco Inc entered into a contract of absolute sale,
themselves to deliver to PBC P160K and their share on wherein the former paid in full, causing the transfer of
the property in Caloocan City. However, the Orlinos did titles in its name.
not act on their obligations. PBC could not be required
to wait for them forever. Thus, PBC had the right to ISSUE: W/N Investco Inc properly rescinded its contract
consider the contract to sell between them terminated to sell and buy with Solid Homes Inc
for non-payment of the stipulated consideration.
HELD: YES. Upon Solid Homes Inc's failure to comply
with its obligation under the contract, there was no
10. AFP MUTUAL BENEFIT ASSN INC v CA need to judicially rescind the contract. Failure by one of
the parties to abide by the conditions in a contract to
FACTS: This case involved Solid Homes Inc's MR of the sell resulted in the rescission of the contract.

CONDITIONS AND WARRANTIES


1. LA FORTEZA v MACHUCA -30,000 earnest money, to be forfeited in favor of
Lafortezas if the sale is not effected due to the fault of
FACTS: Machuca
* The disputed property in this case consists of a house -600,000 upon issuance of the new certificate of
and lot located at Marcelo Green Village, Paranaque, title in the name of late Francisco Laforteza and upon
which is registered in the name of the late Francisco execution of an extra judicial settlement of the
Laforteza, although it is conjugal in nature decedents estate with sale in favor of Machuca
* Lea Zulueta-Laforteza executed a Special Power of * Paragraph 4 of the Memorandum contained a provision
Attorney in favor of Robert and Gonzalo Laforteza, that: upon issuance of the new title, the Machuca shall
appointing both as her attorney-in-fact authorizing them be notified in writing and he shall have 30 days to
jointly to sell the subject property and sign any produce the balance of 600k which shall be paid to
document for the settlement of the estate of the late Laforteza upon execution of the extrajudicial
Francisco Laforteza settlement
* Michael Laforteza also executed a Special Power of * Machuca paid earnest money of 30k plus rentals for
Attorney in favor of Robert and Gonzalo Laforteza subject property
granting them the same authority. Both agency *Upon failure of Machuca to comply with the payment of
instruments contained a provision that in any document the balance, Lafortezas informed the formed that they
or paper to exercise authority granted, the signature of were canceling the contract
both attorneys-in-fact must be affixed * Machuca requested that he intends to tender payment
* Dennis Laforteza executed an SPA in favor of Robert L. of the balance which was refused by the Lafortezas who
for the purpose of selling the subject property. A year insisted for the rescission of the memorandum.
later, he executed another SPA in favor of Robert and * Machuca filed an action for specific performance
Gonzalo L. naming both attorneys-in-fact for the * TC: ruled in favor of Machuca which the CA affirmed
purpose of selling the subject property and signing any
document for the settlement of the estate of the late ISSUES: W/N THE CONTRACT EXECUTED BY THE PARTIES
Francisco LAforteza. Both agency instruments contained IS A CONTRACT OF SALE OR A CONTRACT TO SELL
same provisions as that mentioned above.
* In the exercise of the above authority, the heirs of the RULING: CONTRACT OF SALE AND LEASE
late Franciso L. represented by Robert and Gonzalo L The Memorandum of Agreement shows that the
entered into a Memorandum of Agreement (Contract to transaction between the petitioners and respondent was
Sell) with Machuca over the subject property for the one of sale and lease
sum of 630,000 payable as follows: A contract of sale is a consensual contract and is
perfected at the moment there is a meeting of the

ANTONIO | HIPOLITO | IMPERIAL | ZARAGOSA 41


%
minds upon the thing which is the object of the contract The Heirs of Escanlar failed to pay the balance of the
and upon the price. From that moment the parties may purchase price, but the Heirs of Cari-an never
reciprocally demand performance subject to the demanded payment and continued to accept belated
provisions of the law governing the form of contracts. payments. They later on sold their interests over the
In this case, there was a perfected agreement same land to the Chuas and assailed the validity of the
between the petitioners and respondent whereby Deed of Sale they executed with the Heirs of Escanlar.
Lafortezas obligated themselves to transfer the The lower courts annulled the contract for not having
ownership of and deliver the house and lot and Machuca the approval of the court as stipulated.
to pay the price amounting to 630k.
All the elements of a contract of sale were thus ISSUE: W/N the Deed of Sale to the Heirs of Escanlar is
present. However, the balance of the purchase price valid
was to be paid only upon the issuance of the new
certificate of title in lieu of the one in the name of the HELD: YES. There is a distinction between the validity
late Francisco Laforteza and upon the execution of an and effectivity. Only the effectivity was made subject to
extrajudicial settlement of his estate. the condition. So long as all the requisites (consent,
Prior to the issuance of the reconstituted title, subject matter, and price) are present, as in this case,
Machuca was already placed in possession of the house the contract is already perfected. Nonetheless, the
and lot as lessee thereof for 6 months at a monthly rate intent of the parties clearly manifests their intention to
of 3,500k. It was stipulated that should the issuance of give efficacy to the contract. In fact, the vendors
the new title and execution of the extrajudicial continued to accept payments. That being the case, the
settlement be completed prior to expiration of 6month sale in favor of the Heirs of Escanlar must be preferred
period, Machuca would be liable only for the rentals as it is a valid and subsisting one.
pertaining to the period commencing from the date of
the execution of the agreement up to the executon of
the extrajudicial settlement. 3. POWER COMMERCIAL AND INDUSTRIAL CORP. v CA
It was also expressly stipulated that if after the
expiration of the 6 month period, the lost title was not FACTS: Power Commercial Corp entered into a contract
yet replaced and the extrajudicial partition was not yet of sale with the Quiambao spouses. It agreed to assume
executed, Machuca would no longer be required to pay the mortgages thereon. A Deed of Absolute Sale with
rentals and would continue to occupy and use the Assumption of Mortgage was executed. Power
premises until the subject condition was complied with Commercial Corp failed to settle the mortgage debt
by Lafortezas. contracted by the spouses, thus it could not undertake
The 6-month period during which Machuca would be in the proper action to evict the lessees on the lot. Power
possession of the property as lessee, was clearly not a Commercial Corp thereafter sought to rescind the
period within which to exercise such option. An option is contract of sale alleging that it failed to take actual and
a contract granting a privilege to buy or sell within an physical possession of the lot.
agreed time and at a determined price.
In this case, the 6-month period merely delayed the ISSUE: W/N there was a breach of warranty on the part
demandability of the contract of sale and did not of the spouses that it would evict the lessees
determine perfection for after the expiration of the 6
month period, there was a absolute obligation on the HELD: NO. First, such condition that the Quiambao
part of Lafortezas and Machuca to comply with the spouses would have to evict the lessees was not
terms of the sale. stipulated in the contract. Thus, it cannot be considered
The fact that after the expiration of the 6-month a condition imposed upon its perfection. In fact, Power
period, Machuca would retain possession of the house Commercial Corp. was well aware of the presence of the
and lot without need of paying rentals for the use tenants therein. It was also given control over the said
therefore, clearly indicated that the parties lot and it endeavored to terminate the occupation of its
contemplated that ownership over the property would actual tenants. Also, since it was Power Commercial
already be transferred by that time. that knowingly undertook the risk of evicting the
What further indicated that this was a contract of sale lessees, it cannot now claim that there was a breach of
was the payment of earnest money. Earnest money is warranty on the part of the vendor.
something of value to show that buyer was really in
earnest, and given to the seller to bind the bargain.
Whenever earnest money is given in a contract of sale,
4. GUINHAWA v PEOPLE
it is considered as part of the purchase price and proof
of the perfection of the contract.
FACTS:
* Jaime Guinhawa was engaged in the business of selling
2. HEIRS OF PEDRO ESCANLAR v CA brand new motor vehicles, including Mitsubishi vans,
under the business name of Guinrox Motor Sales. His
FACTS: The Heirs of Cari-an executed a Deed of Sale of office and display room for cars were located along
Rights, Interests, and Participation over a parcel of Panganiban Avenue, Naga City. He employed Gil Azotea
undivided land in favor of the Heirs of Escanlar. It was as his sales manager.
stipulated that the contract shall become effective
only upon approval of the CFI of Negros Occidental.

ANTONIO | HIPOLITO | IMPERIAL | ZARAGOSA 42


%
* Guinhawa purchased a brand new Mitsubishi L-300 ISSUE: W/N THERE WERE FRAUDULENT
Versa Van from the Union Motors Corporation (UMC) in REPRESENTATIONS MADE BY THE SELLER, GUINHAWA BY
Paco, Manila. VIRTUE OF THE CONTRACT OF SALE EXECUTED BETWEEN
* The van bore Plate no. DLK 406. Guinhawas driver, HIM AND THE COUPLE
Olayan, drove the van from Manila to Naga City.
* However, while the van was traveling along the RULING: YES
highway in Daet, Camarines Norte, Olayan suffered a Article 1389 of NCC provides that failure to disclose
heart attack. The van went out of control, traversed the facts when there is a duty to reveal them constitutes
highway onto the opposite lane, and was ditched into fraud. In a contract of sale, a buyer and seller do not
the canal parallel to the highway. The van was deal from equal bargaining positions when the latter has
damaged, and the left front tire had to be replaced. knowledge, a material fact which, if communicated to
* The van was repaired and later offered for sale in the buyer, would render the grouns unacceptable or, at
Guinhawas showroom. least, substantially less desirable.
* Spouses Ralph and Josephine Silo wanted to buy a new If, in a contract of sale, the vendor knowingly allowed
van for their garment business; they purchased items in the vendee to be deceived as to the thing sold in a
Manila and sold them in Naga City. material matter by failing to disclose an intrinsic
* Unaware that the van had been damaged and repaired circumstance that it vital to the contract, knowing that
on account of the accident in Daet, the couple decided the vendee is acting upon the presumption that no such
to purchase the van for 591k. Azotea, sales manager, fact exists, deceit is accomplished by the suppression of
suggested that the couple make a downpayment of the truth.
118,200, and pay the balance of the purchase price by In this case, Guinhawa and Azotea knew that the van
installments via a loan from the United Coconut Planters had figured in an accident, was damaged and had to be
Bank (UCPB), with the van as collateral. repaired. Nevertheless, the van was placed in the
* Azotea offered to make the necessary arrangements showroom, thus making it appear to the public that it
with UCPB for the consummation of the loan transaction was a brand new unit. Guinhawa was mandated to
wherein the couple agreed. reveal the foregoing facts to Silos but they even
* The spouses executed a Promissory Note for the obdurately declared when they testified that the court
amount of 692,676 as payment of the balance on the did not figure in an accident, nor had it been repaired.
purchase price, and as evidence of the chattel mortgage Even when Guinhawa was apprised that Silos had
over the van in favor of UCPB. discovered the vans defects, the former agreed to
* The couple arrived in Guinhawas office to take replace the van, but changed his mind and insisted that
delivery of the van. The latter executed the deed of it must be first sold.
sale, and the couple paid the 161,470 downpayment, for Guinhawa is not relieved of his criminal liability for
they were issued a receipt. They were furnished a deceitful concealment of material facts, even if Silos
Service Manual which contained the warranty terms and made a visual inspection of the vans interior and
conditions. exterior before she agreed to buy and failed to inspects
* Azotea instructed the couple on how to start the van its under chassis.
and to operate its radio. Ralph Silo no longer conducted
a test drive; he and his wife assumed that there were no
defects in the van as it was brand new. 5. ANG v CA
* Josephine Silo, accompanied by Glenda Pingol, went to
Manila on board the van, with Glendas husband as the FACTS:
driver. On their return trip to Naga from Manila, the * Under a car-swapping scheme, Bruno Soledad sold his
driver heard a squeaking sound, which seemed to be Mitsubishi GSR sedan 1982 model to Jaime Ang by a
coming from underneath the van. The squeaking sound Deed of Absolute Sale
persisted and upon examination at the Shell gasoline * For his part, Ang conveyed to Soledad his Mitsubishi
station, it was found out that some parts underneath Lancer model 1988 also by a Deed of Absolute Sale
the van had been welded. * As Angs car was of a later model, Soledad paid him an
* Guinhawa insisted that the defects were mere factory additional 55,000
defects. As the defects persisted, the spouses requested * Ang, a buyer and seller of used vehicles, later offered
that Guinhawa replace the van with 2 Charade-Daihatsu the Mitsubishi GSR for sale through Far Eastern Motors, a
vehicles within a week or two, with the additional costs second hand auto display center. The car was even sold
to be taken from their downpayment. to a certain Paul Bugash for 225k.
* The spouses brought the car to Rx Auto Clinic for * Before the Deed could be registered in Bugashs name,
examination wherein the mechanic discovered that it however, the vehicle was seized by virtue of a writ of
was the left front stabilizer that was producing the replevin on account of the alleged failure of Ronaldo
annoying sound, and that it had been repaired. Panes, the owner of the car prior to Soledad, to pay the
* Josephine Silo filed for rescission of the sale and mortgage debt constituted thereon.
refund of their money. * To secure the release of the vehicle, Ang paid BA
* They instituted also a criminal complaint for other Finance the amount of 62,038.47. Soledad refused to
deceits made by Guinhawa by making fraudulent reimburse the said amount, despite repeated demands,
representations about the car being brand new and that drawing Ang to charge him for estafa with abuse of
it never encountered an accident. confidence.

ANTONIO | HIPOLITO | IMPERIAL | ZARAGOSA 43


%
ISSUE: W/N THE COMPLAINT HAD PRESCRIBED HINGES ON action is reckoned from the date, Ang instituted his first
A DETERMINATION OF WHAT KIND OF WARRANTY IS complaint for damages and not when filed the complaint
PROVIDED IN THE DEED OF ABSOLUTE SALE subject of this case, the action just the same had
prescribed, it having been filed 16 months after the
RULING: YES date of delivery of the vehicle.
A warranty is a statement or representation made by On the basis of breach of warranty against eviction,
the seller of goods, contemporaneously and as part of essential requisites thereof were not met. For one,
the contract of sale, having reference to the character, there is no judgment, which deprived Ang of the
quality or title of the goods, and by which he promises vehicle. For another, there was no suit for eviction
or undertakes to insure that certain facts are or shall be which Soledad as seller was impleaded as co-defendant
as he then represents them. at the instance of the vendee.
Warranties by the seller may be express or implied. Even under the principle of solutio indebiti, Ang
In declaring that Soledad owned and had clean title to cannot recover from Soledad the amount he paid BA
the vehicle at the time of the deed of absolute sale was Finance. For, Ang settled the mortgage debt on his own
forged, he gave an implied warranty of title. In pledging volition under the supposition that he would resell the
that he will defend the same from all claims or any car. It turned out that he did pay BA Finance in order to
claim whatsoever and will save the vendee from any suit avoid returning the payment made by the ultimate
by the government of the Republic of the Phils, Soledad buyer Bugash.
gave a warranty against eviction.
Given Angs business of buying and selling used
vehicles, he could not have merely relied on Soledads 6. NUTRIMIX FEEDS CORP v CA
affirmation that the car was free from liens and
encumbrances. He was expected to have thoroughly FACTS: Evangelista spouses purchased feeds from
verified the cars registration and related documents. Nutrimix. They refused to pay their unsettled debt
Since what Soledad, as seller, gave was an implied claiming that thousands of their livestock were poisoned
warranty, the prescriptive period to file a breach by the Nutrimix feeds. Nutrimix sued them for collection
thereof is 6 months after the delivery of the vehicle, of money. The spouses countered with a suit for
following Art. 1571. But even if the date of filing of the
damages. Various expert witnesses were presented that time, they may have already been contaminated.
during the trial. They failed to prove that the feeds delivered to be
tested were the same feeds that allegedly poisoned the
ISSUE: W/N Nutrimix should be held liable for the death animals. It is also common practice for them to mix
of the livestock different kinds of feeds. The mere death of the animals
does not raise a prima facie case of breach of warranty.
HELD: NO. In alleging that there was a violation of In this case, the evidence presented by the spouses are
warranty against hidden defects, the spouses assumed only circumstantial.
the burden of proof. However, this they failed to The remedies of breach of warranty against hidden
overcome. Under the law, the defect must exist at the defects are either withdrawal from the contract or to
time the sale was made and at the time the product left demand a proportionate reduction of the price plus
the hands of the seller, which the spouses failed to damages in either case. In this case, though the spouses
prove. The feeds were belatedly tested3 months after failed to make out their case, hence they should be
the death of the broilers and hogs. This means that at liable for their debt.

EXTINGUISHMENT OF SALE
1. ROBERTS v PAPIO obligation, the corporation returned the owners
duplicate TCT which was then delivered to Amelia
FACTS: Roberts.
* The Spouses Papio were the owners of a 274 sqm * The parties (A. Roberts as lessor and Martin Papio as
residential lot located in Makati. In order to secure a lessee) executed a 2-year contract of lease. The
59k loan from the Amparo Investments Corp, they contract was subject to renewal or extension for a like
executed a real estate mortgage on the property. Upon period at the option of the lessor, the lessee waiving
Papios failure to pay the loan, the corporation filed a thereby the benefits of an implied new lease. The lessee
petition for the extrajudicial foreclosure of the was obliged to pay monthly rentals of 800 to be
mortgage. deposited in the lessors account.
* Since the couple needed money to redeem the * A new TCT was issued in the name of Amelia Roberts
property and to prevent the foreclosure of the real as owner. Martin Papio paid the rentals and thereafter
estate mortgage, they executed a Deed of Absolute Sale for another year. He then failed to pay rentals, but he
over the property in favor of Martin Papios cousin, and his family nevertheless remained in possession of
Amelia Roberts. the property for almost 13 years.
* Of the 95k purchase price, 59k was paid to the Amparo * A. Roberts reminded Papio that he failed to pay
Investments Corp, while the 26k difference was retained monthly rentals amounting to a total liability of 410k.
by the spouses. As soon as the spouses had settled their She demanded that Papio vacate the property within 15
ANTONIO | HIPOLITO | IMPERIAL | ZARAGOSA 44
%
days from receipt of the letter in case he failed to settle HELD: NO. Their right has already prescribed.
the amount. Considering that no period for redemption was agreed
* A. Roberts filed a complaint for unlawful detainer and upon, the law imposes a 4-year limitation. This means
damages against Martin Papio that from the time the school was merged to Cebu State
College, they had 4 years, or until June 1987 to redeem
ISSUE: W/N THE DEED OF ABSOLUTE SALE AND the property. However, they failed to do so within the
CONTRACT OF LEASE EXECUTED BY THE PARTIES IS AN period. Failure to redeem automatically consolidates
EQUITABLE MORTGAGE OVER THE PROPERTY ownership in favor of the vendee. The fact that the right
to redeem was annotated does not make it
RULING: NO imprescriptible, it only serves to notify third persons.
An equitable mortgage is one that, although lacking in
some formality, form or words, or other requisites
demanded by a statute, nevertheless reveals the 3. SOLID HOMES INC v CA
intention of the parties to charge a real property as
security for a debt and contain nothing impossible or FACTS:
contrary to law. A contract between the parties is an * Solid Homes executed in favor of State Financing
equitable mortgage if the following requisites are Center a Real Estate Mortgage on its properties
present: a. the parties entered into a contract embraced in the TCT, in order to secure the payment of
denominated as a contract of sale and b. the intention a loan of 10M which the former obtained from the
was to secure an existing debt by way of mortgage. The latter.
decisive factor is the intention of the parties. * A year later, Solid Homes applied for and was
In an equitable mortgage, the mortgagor retains granted an additional loan of 1, 511,270.03 by State
ownership over the property but subject to foreclosure Financing, and to secure its payment, Solid executed an
and sale at public auction upon failure of the mortgagor amendment to real estate mortgage whereby the credits
to pay his obligation. secured by the first mortgage on the abovementioned
In contrast, in a pacto de retro sale, ownership of the properties were increased from 10M to 11,511,270.03.
property sold is immediately transferred to the vendee a * Solid homes obtained additional credits and
retro subject only to the right of the vendor a retro to financing facilities from State Financing in the sum of
repurchase the property upon compliance with legal 1,499,811.97 and to secure its payment, the former
requirements for the repurchase. The failure of the executed the amendment to real estate mortgage
vendor a retro to exercise the right to repurchase within whereby the mortgage executed on its properties was
the agreed time vests upon the vendee a retro, by again amended so that the loans or credits secured
operation of law, absolute title over the property. thereby were further increased from 11,511, 270.03 to
One repurchases only what one has previously sold. 13,011,082.00
The right to repurchase presupposes a valid contract of * When the obligations became due and payable, State
sale between same parties. By insisting that he had Financing made repeated demands upon Solid homes for
repurchased the property, Papio thereby admitted that the payment thereof, but the latter failed to do so.
the deed of absolute sale executed by him and Roberts * State Financing filed a petition for extrajudicial
was in fact and in law a deed of absolute sale and not an foreclosure of the mortgages who in pursuance of the
equitable mortgage; he had acquired ownership over the petition, issued a notice of sheriffs sale whereby the
property based on said deed. mortgaged properties of Solid homes and the
Respondent, is thus estopped from asserting that the improvements existing thereon, including the V.V.
contract under the deed of absolute sale is an equitable Soliven Towers II Building were set for public auction
mortgage unless there is an allegation and evidence of sale in order to satisfy the full amount of Solid homes
palpable mistake on the part of respondent, or a fraud mortgage indebtedness, the interest thereon, and the
on the part of Roberts. fees and expenses incidental to the foreclosure
proceedings.
* Before the scheduled public auction sale, the
2. MISTERIO v CEBU STATE COLLEGE OF SCIENCE AND mortgagor Solid homes made representations and
TECHNOLOGY induced State Financing to forego with the foreclosure
of the real estate mortgage. By reason thereof, State
FACTS: Asuncion sold to Sudlon Agricultural High School Financing agreed to suspend the foreclosure of
(SAHS) a parcel of land, reserving the right to mortgaged properties, subject to the terms and
repurchase the same in case (1) the school ceases to conditions they agreed upon, and in pursuance of the
exist, or (2) the school transfers location. She had her said agreement, they executed a document entitled
right annotated. She died. By virtue of BP 412, SAHS was MEMORANDUM OF AGREEMENT/DACION EN PAGO.
merged with the Cebu State College, effective June
1983. In 1990, the heirs of Asuncion sought to exercise ISSUE:
their right to redeem, claiming that school has ceased to
exist. 1. W/N THE MEMORANDUM OF AGREEMENT/
DACION EN PAGO EXECUTED BY THE PARTIES IS
ISSUE: W/N the heirs of Asuncion may still exercise their VALID AND BINDING
right to redeem the property 2. W/N SOLID HOMES CAN CLAIM DAMAGES
ARISING FROM THE NON-ANNOTATION OF ITS

ANTONIO | HIPOLITO | IMPERIAL | ZARAGOSA 45


%
RIGHT OF REPURCHASE IN THE CONSOLIDATED ownership, b) the alleged new liability of the spouses c)
TITLES the alleged continuing liability of the spouses. It is clear
that the petitioners had other issues which involve more
RULING: 1. YES | 2. NO than just a simple claim of of immediate possession, and
The Memorandum of Agreement/Dacion En Pago was thus the RTC had jurisdiction over the case.
valid and binding, and that the registration of said However, the transfer was in the nature of pactum
instrument in the Register of Deeds was in accordance commissorium, since the sale was really considered as
with law and the agreement of the parties. an equitable mortgage. It was really intended by the
Solid homes utterly failed to prove that respondent spouses to make such undated deed of sale a security.
corporation had maliciously and in bad faith caused the Also, when petitioners transferred the title in its name,
non-annotation of petitioners right of repurchase so as the spouses was never informed of such action. Such
to prevent the latter from exercising such right. transfer was therefore void, making the TCT held by
On the contrary, it is admitted by both parties that petitioners null and void as well.
State Financing informed Solid homes of the registration
with the register of deeds of their memorandum of
agreement/dacion en pago and the issuance of the new 5. ABILLA v GOBONSENG
certificates of title in the name of State Financing.
Clearly, petitioner was not prejudiced by the non- FACTS: Spouses Abilla instituted against Spouses
annotation of such right in the certificates of title issued Gobonseng an action for specific performance, recovery
in the name of State Financing. Also, it was not the of sum of money and damages, seeking the
function of the corporation to cause said annotation. It reimbursement of the expenses they incurred in the
was equally the responsibility of petitioner to protect its preparation and registration of 2 public instruments--
own rights by making sure that its right of repurchase Deed of Sale and Option to Buy. As a defense, Spouses
was indeed annotated in the consolidated titles of State Gobonseng contended that the transaction covered by
Financing. these instruments was a mortgage. RTC ruled in favor of
The only legal transgression of State was its failure to Spouses Abilla, stating that it was a sale giving Spouses
observe the proper procedure in effecting the Gobonseng until Aug. 31, 1983 within which to buy back
consolidation of the titles in its name. But this does not the 17 lots subject of the sale. CA affirmed and held
automatically entitle the petitioner to damages absent that the transaction was a pacto de retro sale, and not
convincing proof of malice and bad faith on the part of an equitable mortgage.
private respondent-corporation In 1999, Spouses Gobonseng filed with the RTC an
urgent motion to repuchase the lots with tender of
payment, which was denied. However, after the judge
4. A. FRANCISCO REALTY v CA inhibited himself from the case, it was reraffled to a
different branch, which granted the motion to
FACTS: A. Francisco Realty and Development Corp. repurchase.
granted a loan worth P7.5M in favor of spouses
Javillonar, to which the latter executed three ISSUE: W/N Spouses Gobonseng may exercise the right
documents: a) a promissory note containing the interest to repurchase, as stipulated in Art. 1606 (3)
charge of 4% monthly, b) a deed of mortgage over the
subject property, c) an undated deed of sale of the HELD: NO. Sellers in a sale judicially-declared as pacto
mortgaged property. Since the spouses allegedly failed de retro may NOT exercise the right to repurchase
to comply with the payments, petitioner registered the within the 30-day period provided under Art. 1606,
sale in its favor, getting a TCT issued in its name although they have taken the position that the same was
without knowledge by the spouses. Subsequently, the an equitable mortgage, if it shown that there was no
spouses obtained another loan worth P2.5M, signing honest belief thereof since: (a) none of the
another promissory note in favor of petitioner. circumstances under Art. 1602 were shown to exist to
Petitioner demanded the possession of the property, as warrant a conclusion that the transaction was an
well as the interest payments, to which the spouses equitable mortgage; and (b) that if they truly believed
refused to comply. Petitioner filed an action for the sale to be an equitable mortgage, as a sign of good
possession in the RTC. RTC ruled in favor of petitioner, faith, they should have consigned with the trial court
but CA reversed. the amount representing their alleged loan, on or before
the expiration of the right to repurchase.
ISSUE:

1. Whether the RTC had jurisdiction over the case 6. FRANCISCO v BOISER
(property issue)
2. Whether the sale was considered as an FACTS:
equitable mortgage Petitioner Adalia Francisco and three of her sisters,
Ester, Elizabeth, and Adeluisa, were co-owners of four
RULING: Even though the case was filed less than one parcels of registered land in Caloocan City
year after the demand to vacate, making it an action of On August 1979, they sold 1/5 of their undivided share
unlawful detainer, there were other issues to be to their mother, Adela Blas, for PhP10,000, making her a
considered such as: a) the validity of the transfer of co-owner of the real property to that extent
ANTONIO | HIPOLITO | IMPERIAL | ZARAGOSA 46
%
7 years later, in 1986, however, Adela sold her 1/5 under the law must be notified of the sale
share for PhP10,000 to respondent Zenaida Boiser, Notice by the co-owner likewise removes all doubt as
another sister of petitioner to the fact of the sale, its perfection, and its validity by
In 1992 or 6 years after the sale, Adalia received not immediately notifying, or not notifying at all, a co-
summons with a copy of a complaint by Zenaida owner, the vendor can delay or even effectively prevent
demanding her share in the rentals being collected from the meaningful exercise of the right of redemption
the tenants of the Ten Commandments Building, which However, it would be unjust in the case at bar to
stands on the co-owned property require the vendor Adela to serve notice of the sale,
Adalia then informs Zenaida that she was exercising when the fact has already been established in both
her right of redemption as co-owner of the subject lower courts Adalia has effectively exercised her right
property, depositing for that purpose PhP10,000 with when she deposited the PhP10,000 redemption price 7
the Clerk of Court days after receiving the summons
The case was however dismissed after Zenaida was
declared non-suited, and Adalias counterclaim was thus Fallo
dismissed as well Petition granted, decision of the CA reversed
3 years after, Adalia institutes a complaint demanding The decision in Etcuban v. CA is abandoned, and the
the redemption of the property, contending that the 30- one in Butte v. Manuel Uy and Sons, Inc., as affirmed in
day period for redemption under Art. 1623 had not Salatandol v. Retes, upheld
begun to run against her or any of the other co-owners,
since the vendor Adela did not inform them about the NOTE
sale, which fact they only came to know of when Adalia The Court failed to negate or possibly appreciate the
received the summons in 1992 fact of Adalias knowledge of the sale prior to the
Zenaida on the other hand contends that Adalia summons, as proven her letter-advise to the tenants of
already knew of the sale even before she received the the building
summons since Zenaida had informed Adalia by letter of The period given by the Court to Adalia was 30 days
the sale with a demand for her share of the rentals after the receipt of the summons on 5 August 1992,
three months before filing suit, attaching to it a copy of which is 4 September 1992
the deed of sale
Adalias receipt of the said letter is proven by the fact
that within a week, she advised the tenants of the 7. SORIANO v BAUTISTA
building to disregard Zenaidas letter-demand
The trial court dismissed the complaint for legal FACTS: Bautista spouses mortgaged their lot to Soriano,
redemption, holding that Art. 1623 does not prescribe who took possession thereof and cultivated the same.
any particular form of notifying co-owners on appeal, Pursuant to Par. 5 of their agreement, Soriano decided
the CA affirmed to buy the lot. Bautista refused to sell claiming that
being mortgagors, they cannot be deprived of their right
ISSUE: Whether the letter-demand by Zenaida to Adalia, to redeem the property.
to which the deed of sale was attached, can be
considered as sufficient compliance with the notice ISSUE: W/N Soriano may buy the mortgaged property of
requirement of Art. 1623 for the purpose of legal Bautista
redemption
HELD: YES. True that the transaction is a mortgage,
HELD: which carried with it a customary right of redemption.
The petitioner points out that the case does not However, the mortgagors right to redeem was rendered
concern the particular form in which such notice must defeasible at the election of the mortgagees by virtue of
be given, but rather the sufficiency of notice given by a Par. 5, allowing them the option to purchase the said
vendee in lieu of the required notice to be given by the lot. There is nothing immoral or illegal about such
vendor or prospective vendor stipulation. It was supported by the same consideration
The text of Art. 1623 clearly and expressly prescribes as the mortgage contract and constituted an irrevocable
that the 30 days for making the redemption shall be continuing offer within the time stipulated. That being
counted from notice in writing by the vendor it makes the case, Bautista spouses must be compelled to honor
sense to require that notice be given by the vendor and the sale.
nobody else, since the vendor of an undivided interest is
in the best position to know who are his co-owners, who

ANTONIO | HIPOLITO | IMPERIAL | ZARAGOSA 47


%

ASSIGNMENT
1. NYCO SALES CORP v BA FINANCE made by Licaros with the Anglo-Asean Bank at the
former's expense. When Gatmaitan contacted the
FACTS: NYCO Sales Corp extended a credit foreign bank, it said they will look into it, but it didn't
accommodation to the Fernandez Brothers. The prospered. Because of the inability to collect,
brothers, acting in behalf of Sanshell Corp, discounted a Gatmaitan did not bother to pay Licaros the value of the
BPI check for P60,000 with NYCO, which then indorsed promissory note. Licaros, however, believing that he had
the said check to BA Finance accompanied by a Deed of a right to collect from Gatmaitan regardless of the
Assignment. BA Finance, in turn, released the funds, outcome, demanded payment, but was ignore. Licaros
which were used by the brothers. The BPI check was filed a complaint against Gatmaitan for the collection of
dishonored. The brothers issued a substitute check, the note. The trial court ruled in favor of Licaros, but
which was also dishonored. Now BA Finance goes after CA reversed.
NYCO, which disclaims liability.
ISSUE: Whether the memorandum of agreement
ISSUE: W/N NYCO, as the assignor, is liable for breach of between petitioner and respondent is one of assignment
warranties of credit or one of conventional subrogation

HELD: YES. The assignor (NYCO) warrants both the RULING: It is a conventional subrogation. An assignment
existence and legality of the credit, as well as the of credit has been defined as the process of transferring
solvency of the debtor. If there is a breach of any of the the right of the assignor to the assignee who would then
2 warranties, the assignor is liable to the assignee. That have a right to proceed against the debtor. Consent of
being the case, NYCO cannot evade liability. So long as the debtor is not required is not necessary to product its
the credit remains unpaid, the assignor remains liable legal effects, since notice of the assignment would be
notwithstanding failure to give notice of dishonor that is enough. On the other hand, subrogation of credit has
because the liability of NYCO stems form the been defined as the transfer of all the rights of the
assignment, not on the checks alone. creditor to a third person, who substitutes him in all his
rights. It requires that all the related parties thereto,
the original creditor, the new creditor and the debtor,
2. LICAROS v GATMAITAN enter into a new agreement, requiring the consent of
the debtor of such transfer of rights. In the case at
FACTS: Abelardo Licaros invested his money worth hand, it was clearly stipulated by the parties in the
$150,000 with Anglo-Asean Bank, a money market memorandum of agreement that the express conformity
placement by way of deposit, based in the Republic of of the third party (debtor) is needed. The memorandum
Venatu. Unexpectedly, he had a hard time getting back contains a space for the signature of the Anglo-Asean
his investments as well as the interest earned. He then Bank written therein "with our conforme". Without such
sought the counsel of Antonio Gatmaitan, a reputable signature, there was no transfer of rights. The usage of
banker and investor. They entered into an agreement, the word "Assignment" was used as a general term, since
where a non-negotiable promissory note was to be Gatmaitan was not a lawyer, and therefore was not
executed in favor of Licaros worth $150,000, and that well-versed with the language of the law.
Gatmaitan would take over the value of the investment

BULK SALES LAW


1. CHIN v UY: CA case contained in O.G. DBP in a "deed of cession of property in payment of
obligation" or dacion en pago. In turn, DBP sold these
DOCTRINE: A sale made of all the effects in the vendor's assets to Union Glass that same year.
store without the buyer being furnished a sworn list of In 1983, Yu instituted an action against Pioneer Glass,
creditors as required by Sec 3, is null and void DBP, and Union Glass, asserting that the transfer of the
irrespective of the good or bad faith of the buyer, and assets to DBP was void by reason of fraud.
judgment creditors may treat such sale as never having
been made and proceed to have execution levied on the Pioneer Glass: denied liability to Yu on the
properties thus sold. ground that by virtue of the dacion en pago in
favor of DBP, the bank assumed liability to its
creditors including Yu under a payment
2. DBP v HON JUDGE OF RTC OF MANILA scheme, which is under pending
implementation
FACTS: In 1978, Pioneer Glass Manufacturing DBP: denied liability to Yu on the ground that
Corp.purchased from Yu (under Ancar Equipment Parts there being no proof that the unpaid
and Tonicar) equipment parts worth P7,000. However, merchandise purchased by Pioneer Glass were
Pioneer failed or refused to pay upon demand. Without among those transferred to it
informing Yu, Pioneer Glass transferred all its assets to Union Glass: denied liability to Yu on the

ANTONIO | HIPOLITO | IMPERIAL | ZARAGOSA 48


%
ground that there was no privity of contract them in order to store, handle and display their goods
between them, or assuming applicability of the and wares. The technicality of these terms conveys the
Bulk Sales Law, no liability attached to Union intention of the law to apply it to merchants who are in
Glass. the business of selling goods and wares and similar
merchandise.
MTC denied the motions to dismiss filed by Union In this case, Pioneer Glass manufactured glass only on
Glass and DBP and ruled in favor of Yu. RTC affirmed specific orders and it did not sell directly to consumers
MTC's decision. but manufactured its products only for particular
clients. Thus, Pioneer Glass was NOT a merchandiser.
ISSUE: W/N the Bulk Sales Law covers the conveyance in Moreover, the dacion en pago between Pioneer and DBP
question (its violation would make DBP, Union Glass, and transferred and conveyed the bulk of its corporate
Pioneer Glass liable to Yu) assets to extinguish its outstanding debts to DBP. Thus,
the subject matter of the deed of cession was the
HELD: NO. Under the Bulk Sales Law, the terms "goods" assets, not stock-in-trade. Such conveyance was clearly
and "merchandise," having acquired a fixed meaning, outside the ambit of the Bulk Sales Law.
refer to things and articles, which are kept for sale by a SC ordered Pioneer Glass, not DBP and Union Glass, to
merchant. Likewise, the term "fixtures" has been pay Yu the price of the equipment purchased plus
interpreted to mean the chattels, which the merchants interest.
usually possess and annex to the premises occupied by

RETAIL TRADE LIBERALIZATION ACT OF 2000


AND RELATED PROVISIONS OF THE ANTI-DUMMY LAW
1. KING v HERNAEZ consumption goods. They are sold to manufacturers
and industries as raw materials. They are intermediate
FACTS: Macario King, a naturalized Filipino, owned the goods, not consumption goods.
grocery store Import Meat & Produce. He employed 3
Chinamen, one as purchaser and 2 others as salesmen.
He sought the permission of the President to retain the 3. GOODYEAR TIRE v REYES SR
services of the 3, but was denied based on the Retail
Trade Law and the Anti-Dummy Law, which prohibit FACTS: Goodyear, a corporation not wholly owned by
aliens from interfering in the management and Filipinos, was engaged in the manufacturing and sale of
operation of retail establishments. King contends that rubber products such as tires, batteries, conveyor belts,
the 3 aliens are employed in non-control positions and soles of shoes, etc.
do not participate in the management, thus, they are
not covered by the Anti-Dummy Law. ISSUE: W/N Goodyear is covered by the Retail Trade Law
insofar as the prohibition against aliens from engaging in
ISSUE: W/N the employment of the 3 Chinamen is retail trade is concerned.
covered under the Anti-Dummy Law
HELD: NO. Retail pertains to the direct selling to the
HELD: YES. The prohibition covers the entire range of general public of merchandise of goods for consumption.
employment, regardless of whether they are control or They pertain to goods for personal, family and
non-control positions. Thus, employment of aliens for household consumption. A manufacturer who sells his
evening clerical positions is prohibited. The reason is products to industrial and commercial users so that the
obvious: to plug any loopholes that unscrupulous aliens latter may use the same to render some general service
may exploit for the purpose of circumventing the law. to the public is clearly not covered by the prohibition.
The enterprise of Goodyear clearly falls within this
category. The sale to proprietary planters and persons
2. BALMACEDA v UNION CARBIDE PHILIPPINES INC engaged in the exploration of natural resources is also
included in the said classification and cannot be
FACTS: Union Carbide was a manufacturer having 2 considered retail as to come within the ambit of the
divisions: the Consumer Products Division and the prohibition. But insofar as sale to employees and
Industrial Products Division. officers is concerned, this may be considered retail
and comes under the prohibition.
ISSUE: W/N the Industrial Products Division is engaged in
the retail business
4. DBP v HON JUDGE OF RTC OF MANILA
HELD: NO. Retail pertains to the direct selling to the
general public of merchandise of goods for consumption. FACTS: In 1978, Pioneer Glass Manufacturing
They pertain to goods for personal, family and Corp.purchased from Yu (under Ancar Equipment Parts
household consumption. The products sold under this and Tonicar) equipment parts worth P7,000. However,
division are clearly not covered by the term Pioneer failed or refused to pay upon demand. Without
ANTONIO | HIPOLITO | IMPERIAL | ZARAGOSA 49
%
informing Yu, Pioneer Glass transferred all its assets to assuming applicability of the Bulk Sales Law, no liability
DBP in a "deed of cession of property in payment of attached to Union Glass.
obligation" or dacion en pago. In turn, DBP sold these
assets to Union Glass that same year. MTC denied the motions to dismiss filed by Union
In 1983, Yu instituted an action against Pioneer Glass, Glass and DBP and ruled in favor of Yu. RTC affirmed
DBP, and Union Glass, asserting that the transfer of the MTC's decision.
assets to DBP was void by reason of fraud.
ISSUE: W/N the Pioneer Glass is a merchandiser, covered
Pioneer Glass: denied liability to Yu on the ground that under the Retail Trade Act
by virtue of the dacion en pago in favor of DBP, the
bank assumed liability to its creditors including Yu under HELD: NO. There was an undisputed evidence that
a payment scheme, which is under pending Pioneer Glass manufactures glass only on specific orders
implementation and does not sell directly to consumers but
DBP: denied liability to Yu on the ground that there manufactures its products only for particular clients. As
being no proof that the unpaid merchandise purchased such, it cannot be said the Pioneer Glass is a
by Pioneer Glass were among those transferred to it merchandiser within the meaning of the Retail Trade
Union Glass: denied liability to Yu on the ground that Act.
there was no privity of contract between them, or

ANTONIO | HIPOLITO | IMPERIAL | ZARAGOSA 50

Das könnte Ihnen auch gefallen